Exam 2 Flashcards

1
Q
  1. Question: Which is not is a benefit of POCUS?
    a. Increases procure accuracy
    b. Can be done inpatient & outpatient
    c. There is prompt screening
    d. There is radiation, which allows for accuracy
A

Answer: D
Rationale for why the answer is correct: There is no ionizing radiation, which decreases the amount of radiation to the patient
Site for question/rationale: radiation presentation time 5:45

How well did you know this?
1
Not at all
2
3
4
5
Perfectly
2
Q
  1. Question: Why would the provider choose to eFast in the trauma center?
    a. Because it is fast and easy
    b. To check for fluid in the abdomen/chest
    c. Pt can receive definite care like chest tubes
    d. All the of answers above
A

Answer: D
Rationale for why the answer is correct: Efast can be used at bedside without interrupting ongoing care, thus making it easy and fast to use
Site for question/rationale: radiation presentation time 18:26

How well did you know this?
1
Not at all
2
3
4
5
Perfectly
3
Q
  1. Question: Upon your eFast, you notice there is possible injury to the liver. What should the provider do next?
    a. Diagnose patient based of the findings
    b. Order CT scan of the liver
    c. Repeat the eFast in one hour
    d. Nothing else needs to be done
A

Answer: B
Rationale for why the answer is correct: cannot define whether there is solid organ injury. You can only check for fluids or not with eFast. Therefore, pt needs to have a CT scan of the liver.
Site for question/rationale: Radiology time 28:45

How well did you know this?
1
Not at all
2
3
4
5
Perfectly
4
Q
  1. Question: Your 65-year patient presents to the clinic with erythematous plaque with greasy looking, yellow scales to his scalp. He states, “I think it’s dandruff, but nothing I’ve tried in the store has helped.” He asked what is causing this?
    a. Because he lacks vitamin E
    b. Because he wears hats all the time
    c. Cause is unknown
    d. Because he eats too much greasy foods
A

Answer: C
Rationale for why the answer is correct: While the exact cause is unknown, it is thought that the sebaceous glands are involved.
Site for question/rationale: Dermatology presentation 13:13

How well did you know this?
1
Not at all
2
3
4
5
Perfectly
5
Q
  1. Question: Your patient c/o skin tags to his neck and would like to have them removed. What kind of biopsy would be best for removal of the skin tags?
    a. Shave biopsy
    b. Punch biopsy
    c. Excisional biopsy
    d. Puncture biopsy
A

Answer: A
Rationale for why the answer is correct: The shave biopsy is quick and requires no sutures
Site for question/rationale: dermatology presentation, Dr. Day time 14:12

How well did you know this?
1
Not at all
2
3
4
5
Perfectly
6
Q
  1. Question: A 15-year female admits to you that she is having thoughts of killing herself right now because her boyfriend broke up with her? What should the provider do next?
    a. Keep patient in sight
    b. Since she is with her parents, allow for patient to go home with them
    c. Just document in the chart, no further action is needed
    d. Do not take patient seriously, because she doesn’t mean it at her age
A

Answer: A
Rationale for why the answer is correct: Suicide is the 2nd leading cause of death in ages 10-24. Always take them seriously, stay with them, remove sharp objects, and get immediate assistance
Site: suicide sheet pages 1-2

How well did you know this?
1
Not at all
2
3
4
5
Perfectly
7
Q
  1. Question: What screening tool is to assess for alcohol abuse?
    a. PHQ-9
    b. CAGE
    c. SOAP
    d. Mini mental exam
A

Answer: C
Rationale for why the answer is correct: CAGE is the correct screening tool. Cut down, annoyed, guilty, eye opener
Site for question/rationale: Cage scale-alcohol screening tool

How well did you know this?
1
Not at all
2
3
4
5
Perfectly
8
Q
  1. Question: A 33 y/o male patient c/o a severe, throbbing left side of his head pain with sensitivity to light that started about 3 days ago. He is most likely experiencing which type of headache?
    a. Tension headache
    b. migraine
    c. cluster headache
    d. exercise headache
A

Answer: A
Rationale for why the answer is correct: usually occur on one side, often associated with N/V, extreme sensitivity to light and sound
Site for question/rationale: Headaches PDF

How well did you know this?
1
Not at all
2
3
4
5
Perfectly
9
Q
  1. Question: A 25 y/o female c/o runny nose, clear eye drainage, that only seems to happen in the spring time. She lives near a lot of ragwood trees. This patient more than likely has:
    a. Seasonal allergic rhinitis
    b. sinusitis
    c. perennial allergic rhinitis
    d. non-allergic rhinitis
A

Answer: A
Rationale for why the answer is correct: occurs at various times of the year caused by hypersensitivity to ragwood
Site for question/rationale: Nose PDF

How well did you know this?
1
Not at all
2
3
4
5
Perfectly
10
Q
  1. Question: A 43 y/o male c/o having the worst headache in his life that started about an hour ago. He has tried ibuprofen and resting, but nothing is helping. What should the provider recommend?
    a. Prescribe Tylenol extra strength Q4 hours
    b. Send pt home with a work excuse for 5 days off of work
    c. Refer to the ER for immediate evaluation
    d. Have patient scheduled for a MRI in one week if his headache has not subsided
A

Answer: C
Rationale for why the answer is correct: should consider a sentinel headache of an intracranial aneurysm
Site for question/rationale: to scan or not to scan in headaches

How well did you know this?
1
Not at all
2
3
4
5
Perfectly
11
Q
  1. Question: In order to accurately depict the size of the heart in an x-ray, the view should be _.
    a. PA
    b. AP
    c. Lateral
    d. The view does not make a difference
A

Answer: A
Rationale for why the answer is correct: In order to accurately estimate the size of the heart in a chest x-ray, you should obtain a PA view. This means that the x-ray is being taken from the posterior to the anterior of the patient. The patient’s heart is then closer to the film; therefore, it is a more accurate representation of the heart’s size.
Site for question/rationale: Basic radiology pdf, page 7, slide 1

How well did you know this?
1
Not at all
2
3
4
5
Perfectly
12
Q
  1. Question: A chest x-ray shows a heart silhouette that is greater than ½ the total width of the chest. Potential differential diagnoses for this condition are _ (select all that apply).
    a. Acute myocardial infarction
    b. Pericardial effusion
    c. Congestive heart failure
    d. Pneumothorax
    e. Cardiomyopathy
A

Answer: B, C, and E
Rationale for why the answer is correct: If the cardiac silhouette is >1/2 the total width of the chest, the muscle has clearly hypertrophied, or there is an effusion. This does not occur in an acute myocardial infarction or pneumothorax.
Site for question/rationale: Basic radiology pdf, page 12, slide 1

How well did you know this?
1
Not at all
2
3
4
5
Perfectly
13
Q
  1. Question: Which skin lesion occurs usually on the face due to solar radiation?
    a. Squamous cell carcinoma
    b. Basal cell carcinoma
    c. Pityriasis Rosea
    d. Scarlatina
A

Answer: B
Rationale for why the answer is correct: Squamous cell carcinoma is commonly occurring on the head due to sun exposure, basal cell carcinoma usually occurs on the face due to sun exposure, pityriasis rosea occurs on the trunk and then spreads in a Christmas tree pattern, and scarlatina usually starts on the trunk and then spreads to the extremities.
Site for question/rationale: Dr. Day’s Dermatology lecture; starting at 24 minutes and ending at 35 minutes.

How well did you know this?
1
Not at all
2
3
4
5
Perfectly
14
Q
  1. Question: Which of the following is incorrect for the ABCDE mnemonic for assessing skin lesions?
    a. Color variation
    b. Diameter >2mm
    c. Evolution
    d. Asymmetry
A

Answer: B
Rationale for why the answer is correct: The ABCDE mnemonic stands for: Asymmetry, Border irregularities, Color variegation, Diameter >6mm, and Evolution. Diameter of >2mm is incorrect.
Site for question/rationale: Dr. Day’s Dermatology lecture, around 1 minute

How well did you know this?
1
Not at all
2
3
4
5
Perfectly
15
Q
  1. Question: A patient presents with painful yellow blisters in a dermatomal pattern. Your top differential diagnosis is _.
    a. Herpes simplex virus
    b. Chicken pox
    c. Impetigo
    d. Herpes zoster
A

Answer: D
Rationale for why the answer is correct: Herpes zoster virus presents with yellow, painful blisters in a dermatomal distribution.
Site for question/rationale: Dr. Moore and Dr. Sun’s Skin Lesion presentation video at 4-9 minutes.

How well did you know this?
1
Not at all
2
3
4
5
Perfectly
16
Q
  1. Question: When using the Ask Suicide Screening Questions (asQ) tool for adolescents, which question may be included?
    a. Have you spoken to your parents about this?
    b. Do you have support at home?
    c. Are you having thoughts of killing yourself right now?
    d. Do you have a plan for killing yourself?
A
Answer: C
Rationale for why the answer is correct: After asking the first four questions of the asQ tool (In the past few weeks, have you wished you were dead?, In the past few weeks, have you felt that you or your family would be better off if you were dead?, In the past week, have you been having thoughts about killing yourself?, and Have you ever tried to kill yourself?), you should continue to question number 5 if any of the above questions resulted in a “yes” answer. The fifth questions that should be asked is “Are you having thoughts of killing yourself right now?”.
Site for question/rationale: Depression Screening Information for Youth in module 4, page 2.
How well did you know this?
1
Not at all
2
3
4
5
Perfectly
17
Q
  1. Question: The CAGE questionnaire includes each of the following questions EXCEPT _.
    a. Have you ever felt Guilty about drinking?
    b. Have you ever been Angry when you didn’t have alcohol to drink?
    c. Have you ever felt the need to Cut down on your drinking?
    d. Have you ever felt you needed a drink first thing in the morning (Eye-opener) to steady your nerves or to get rid of a hangover?
A
Answer: B
Rationale for why the answer is correct: Each of the questions above is included in the CAGE questionnaire except for answer choice B. The correct question for the A in CAGE is: “Have people Annoyed you by criticizing your drinking?”. 
Site for question/rationale: CAGE Scale – Alcohol Screening Tool in module 4
How well did you know this?
1
Not at all
2
3
4
5
Perfectly
18
Q
  1. Question: A 21-year-old female presents with a headache that she describes as pulsating only on the left side of her head. She has nausea without vomiting. She also complains that the light makes the headache worse. You know that this is a __.
    a. Migraine
    b. Migraine with aura
    c. Tension headache
    d. Cluster headache
A
Answer: A
Rationale for why the answer is correct: A migraine headache is severe, unilateral, pulsatile, and is often accompanied by nausea, vomiting, and photophobia. Migraines with auras are migraines that strike after sensory disturbances (flashes of light, blind spots, vision changes, tingling of face). Cluster headaches are localized on one eye, awaken you in the middle of the night, and come in cyclical patterns or clusters. Tension headaches are common and are described as mild to moderate pain in a tight band around the head. Tension headaches are the most common headaches. 
Site for question/rationale: Headaches PDF in module 4
How well did you know this?
1
Not at all
2
3
4
5
Perfectly
19
Q
  1. Question: A correct description of proptosis is __.
    a. An infection of the retina that is common in patients with AIDS
    b. Protrusion of the eye(s) from the eye socket(s) due to swelling of tissue behind the eye
    c. Malalignment of the eyes on the same point at the same time
    d. Thinned and cone shaped cornea
A
Answer: B
Rationale for why the answer is correct: CMB retinitis is a serious infection of the retina that is common in patients with AIDS, proptosis is when one or both eyes protrude from the eye sockets due to swelling behind the eye, strabismus (or crossed eyes) is when the eyes cannot align on the same point at the same time, and keratoconus is when the cornea becomes thin and cone shaped. 
Site for question/rationale: Eyes PDF in module 4
How well did you know this?
1
Not at all
2
3
4
5
Perfectly
20
Q
  1. Question: A patient presents to the clinic with complaints of a runny and itchy nose and frequent sneezing. When reviewing the history of present illness, the patient reports that she has been using nasal decongestants for 2 weeks due to a head cold. The top differential diagnosis based on this information is __.
    a. Allergic rhinitis
    b. Sinusitis
    c. Common cold with nasal congestion
    d. Non-allergic rhinitis
A
Answer: D
Rationale for why the answer is correct: Common causes of non-allergic rhinitis include cigarette smoke, pollutants, strong odors, alcoholic beverages, cold temperatures, blockages in the nose, deviated septum, infections, and over-use of nasal decongestants. Allergic rhinitis is due to non-infectious particles that cause an immune response, such as pets, molds, dust mites, foods, medicines, and insect venom. This patient has non-allergic rhinitis due to overuse of decongestants. 
Site for question/rationale: Nose PDF in module 4
How well did you know this?
1
Not at all
2
3
4
5
Perfectly
21
Q
  1. Question: What can a POCUS be used to help diagnose/treat a patient? (select all that apply)
    a. Lumbar puncture
    b. Thoracentesis
    c. Paracentesis
    d. Pneumothorax
A

Answer: A, B, C, D
Rationale for why the answer is correct: ultrasounds can be helpful in guiding the physician/practitioner in all of these procedures where to place the needle
Site for question/rationale: Ultrasound presentation by Dr. Steve Branham, slide 6

How well did you know this?
1
Not at all
2
3
4
5
Perfectly
22
Q
  1. Question: The curve linear probe is used to view these structures EXCEPT:
    a. Liver
    b. Breast
    c. Gallbladder
    d. Bladder
A

Answer: B
Rationale for why the answer is correct: The curve linear probe is used to visualize the liver, gallbladder, and bladder. The linear probe is used to visualize the breast.
Site for question/rationale: Ultrasound presentation by Dr. Steve Branham, slide 16

How well did you know this?
1
Not at all
2
3
4
5
Perfectly
23
Q
  1. Question: Which is the most serious form of skin cancer?
    a. Basal cell carcinoma
    b. Melanoma
    c. Squamous cell carcinoma
    d. Merkel cell carcinoma
A

Answer: B
Rationale for why the answer is correct: Melanoma is the most serious form of skin cancer due to its’ ability to spread to other tissues in the body
Site for question/rationale: Dermatology lecture by Dr. Day (timestamp 0:30)

How well did you know this?
1
Not at all
2
3
4
5
Perfectly
24
Q

uestion: Which herpes virus is associated with Kaposi sarcoma?
a. HSV ll
b. HSV l
c. HSV Vlll
d. HSV Vl

A

Answer: C
Rationale for why the answer is correct: Kaposi sarcoma is also known as Kaposi sarcoma-associated Herpes virus (KSHV) because it requires infection by herpes virus Vlll.
Site for question/rationale: Dermatology lecture by Dr. Day (timestamp 10:30)

How well did you know this?
1
Not at all
2
3
4
5
Perfectly
25
5. Question: Which are three contraindications for cryosurgery? a. Malignant melanoma b. Granulation tissue c. Recurrent basal cell carcinoma d. Chronic severe hepatitis B
Answer: A, C, D Rationale for why the answer is correct: Malignant melanoma and recurrent basal cell carcinoma are ABSOLUTE contraindications and chronic severe hepatitis B is a RELATIVE contraindication. Granulation tissue is an indication for cryosurgery. Site for question/rationale: Clinical Key Cryosurgery procedural video information
26
6. Question: Which test can help identify alcohol abuse? a. PHQ-9 b. CAGE c. MoCA test d. DSM-5
Answer: B Rationale for why the answer is correct: PHQ-9 tests for depression, CAGE test for alcohol abuse, MoCA test for memory/cognitive impairment, and DSM-5 is a manual of mental disorders Site for question/rationale: https://psychology-tools.com/cage-alcohol-questionnaire/
27
7. Question: A patient must exhibit symptoms of generalized anxiety disorder for how long in order to receive a diagnosis: a. 2 weeks b. 6 months c. 2 months d. 3 months
Answer: B Rationale for why the answer is correct: The patient must show s/s of generalized anxiety disorder for at least 6 months before a diagnosis can occur. Site for question/rationale: Mental Health Disorders lecture by Dr. Day (timestamp 2:00)
28
8. Question: A secondary headache can be caused by: a. Chiari malformation b. Brain tumor c. Sinus infection d. All of the above
Answer: D Rationale for why the answer is correct: There are several causes for secondary headaches. Chiari malformation, sinus infection, and brain tumor can all activate the pain-sensitive nerves of the head. Site for question/rationale: Headache PDF
29
9. Question: What occurs when a build-up of fluid in the eye creates pressure, damaging the optic nerve? a. Cataracts b. Glaucoma c. Strabismus d. Ocular hypertension
Answer: B Rationale for why the answer is correct: Glaucoma is the cause of fluid build-up creating pressure and damaging the optic nerve. Cataracts are a degenerative form of eye disease in which the lens gradually becomes opaque and vision ‘mists’ over. Strabismus is just another name for “crossed eyes”. Ocular hypertension is just increased pressure in the eye. Site for question/rationale: Eye PDF handout
30
10. Question: What are 3 signs and symptoms of cerumen impaction? a. Tinnitus b. Pruritus c. Hearing loss d. Ear drainage
Answer: A, B, C Rationale for why the answer is correct: Signs and symptoms of cerumen impaction are: hearing loss (conductive), especially unilateral, foreign body sensation; ear fullness, tinnitus (uncommon), vertigo or “dizziness” (uncommon), mild pain or discomfort (earache), pruritus, reflex cough Site for question/rationale: Removal of Impacted Cerumen Clinical Key Todd Thomsen, MD
31
1. In which of the following scenarios would an eFAST exam be used? (Select all that apply) a. Pregnancy b. Blood in Chest c. Blood in Abdomen d. Blood in Brain
Answer: B & C Rationale for why the answer is correct: Used to determine is chest tube is needed due to fluid build up and useful to identify possible liver lacerations causing internal abdominal bleeding, prior to CT or in unstable patients who could not be taken to CT. Site for question/rationale: Ultrasound Lecture at 18:45
32
2. The Nurse Practioner is seeing a patient who presents with signs and symptoms of a pericardial effusion. The Nurse Practioner performs an eFAST on the patient. Which view would the Nurse Practioner use on the patient? a. Left and Right Thoracic views b. Longitudinal View of RUQ c. Subxiphoid Longitudinal view d. Subxiphoid Transverse view
Answer: D Rationale for why the answer is correct: Subxiphoid transverse views are used to assess for pericardial effusion and left lobe liver injuries Site for question/rationale: US lecture at 18:55
33
3. During the skin assessment the Nurse practioner notes a skin lesion with a “shiny and pearly” appearance. The Nurse practioner suspects this could be which of the following? a. Wart b. Squamous cell c. Melanoma d. Basal cell
Answer: D Rationale for why the answer is correct: Shiny pearly appearance is the classic text book appearance of this type of lesion Site for question/rationale: Skin Disorders presentation at 3:00 by Dr Moore and Dr Sun
34
4. The Nurse Practioner needs to anesthetize a large area for a short period of time, for a procedure they will perform. Which of the following would be appropriate to use? a. 0.5% Lidocaine b. 2% Lidocaine c. 6 mg/kg as needed d. 0.5% Lidocaine with Epinephrine
Answer: A Rationale for why the answer is correct: For short term anesthetic effects 0.5% Lidocaine is used for large areas, epinephrine would be added when the block needs to be extended 4-6 hours, 2% is used for smaller areas and the max dose that can be used is 4.5mg/kg Site for question/rationale: Basics of local. Anesthetics by Dr Moore at 1:51
35
5. Question: When assessing a possible melanoma of the skin the Nurse Practioner uses the ABCDE method. Which of the following is NOT part of the ABCDE a. Asymmetry b. Color c. Density >6mm d. Evolution
Answer: C Rationale for why the answer is correct: Site for question/rationale: Dermatology at 1:17 by Dr Day
36
6. Question: A Mother brings in her 12 year old daughter to be seen in the clinic. The mother states that she has noticed a behavior change in her daughter. In the last 12 months her daughter has been bullying kids at school, broke the neighbors car windows, stole some magazines from the store and last month she ran way from home for a day. Of the following diagnosis which is the most likely? a. Bipolar II disorder b. Bipolar I disorder c. Oppositional Defiant disorder d. Conduct Disorder
Answer: D Rationale for why the answer is correct: Patient is fits the following criteria for Conduct disorder, 3 or more criteria within 12 months and 1 within 6 months, Criteria: Aggression to people, Destruction of property, theft and serious violations of rules Site for question/rationale: Common Mental Health Disorders at 7:35 by Dr Day
37
7. Question: The Nurse practioner suspects the patient might have a drinking problem, which assessment tool could the Nurse Practioner use? a. ASq b. CAGE c. NIH d. PHQ-9
Answer: B Rationale for why the answer is correct: The CAGE tool is uses four questions to determine if the patient has a possible substance abuse problem. 1 point is scored for yes and 0 for no, a score of 2 or more is considered clinically significant. Site for question/rationale: CAGE TOOL
38
8. Question: A male patient presents to the clinic with complaints of a headache. They state if feels like they have a tight band across their head and it seems to last for hours. They have recently been under a lot of pressure in their new job. Which of the following is the most likey diagnosis. a. Migraine b. Tension c. Cluster d. Stroke
Answer: A Rationale for why the answer is correct: Tension headaches tend to last from mins to hours, no visual auras are seen by the patient and can be brought on by stress. Site for question/rationale: Headaches presentation by Dr Moore and Dr Hillard at 1:55
39
9. Question: A 55 year old patient presents to the clinic with complaints of headache. The patient states this is the “worst headache” of my life, feels like thunder has clapped all around them. What would the priority intervention the Nurse Practioner to implement? a. Ibuprofen 800mg Q4h b. Icepack around the neck c. Send to ER for CT d. Send home to rest
Answer: C Rationale for why the answer is correct: Must rule out possible brain bleed, patients often describe headache as worst headache they have had, patient older than 55 should rule out brain bleed via CT Site for question/rationale: Headaches by Dr Moore and Dr Hillard at 7:45
40
10. Question: A 31-year old female patient presents with an abrupt onset of sore throat, painful swallowing, high fever, no nasal congestion, no postnasal drip, no cough, generalized malaise and headache. On assessment the patient has swollen and tender lymph nodes on the sides of their neck. a. Mono b. Postnasal Drip c. Strep d. Allergic Rhinitis
Answer: C Rationale for why the answer is correct: Patient is presenting with classic symptoms of Strep throat such as abrupt onset, painful swallowing, fever and swollen neck lympnodes. Site for question/rationale: ENT presentation by Dr Sun and Dr Moore at 15:49
41
1. Question: Which of the following would appear darker (black) on an x-ray? a) Air b) Fat c) Muscle d) Bone
Answer: Air Rationale for why the answer is correct: The less dense an object allows more x-ray to penetrate it so more of the film is exposed Site for question/rationale: Understanding radiology PowerPoint slide 3&4
42
2. Question: Which of the following is NOT one of the four basic views on x-rays? a) PosteroAnterior (PA) b) AnteroPosterior (AP) c) Oblique d) Horizontal
Rationale for why the answer is correct: The four basic views include PosteroAnterior (PA), AnteroPosterior (AP), Lateral (Lat), and Oblique. Site for question/rationale: Understanding radiology PowerPoint slide 7
43
3. Question: What is one way you can rule out a differential diagnosis of melanoma? a) Lesion is not asymmetrical b) Has irregular borders c) Has variegated colors d) It is a recent change/development of the lesion
Answer: It is not Asymmetrical Rationale for why the answer is correct: Lack of asymmetry, irregular border, variegated colors, and being a recent change in lesion along with having the ugly duckling sign would not rule out diagnosis of melanoma, but instead would rule in. Site for question/rationale: Dermatology minute 24:13
44
4. Question: How can you rule in Basal cell carcinoma? a) Lesion on face b) Lesion on body area not exposed to solar radiation c) Blue lesion d) Dry patch and oily area surrounding lesion
Answer: A lesion on face Rationale for why the answer is correct: A lesion on the face could rule in Basal cell carcinoma since It usually occurs on areas exposed to solar radiation like the face. Site for question/rationale: Dermatology Minute 24:38
45
5. Question: The characteristics for diagnosing melanoma are ABCDE, which of the following is not one of these characteristics? a) Asymmetry b) Border irregularities c) Cancer d) Diameter >6mm e) Evolution
Answer: Cancer Rationale for why the answer is correct: the characteristics to look for melanoma include asymmetry, border irregularities, color variegation, diameter >6mm, and evolution Site for question/rationale: Dermatology lecture slide 3
46
6. Question: Which of the following is not one of the 2 symptoms for a person to be diagnosed with a substance use disorder? a) Consuming more than planned b) Repeated use in dangerous situations. c) Failure to fulfill other obligations d) Drink 1-2 drinks on special occasions.
Answer: Drink 1-2 drinks on special occasions. Rationale for why the answer is correct: For someone to be diagnosed with substance abuse disorder the patient needs to meet 2 of the following symptoms within the past 12 months: getting sloppy drunk because they consume more than planned, they are worried about getting their next fix and worry or fail at controlling use, they fail to fulfill other obligations, and they spend significant amount of time using or obtaining drugs, have cravings, continued use, repeated use in dangerous situations, don’t do activities they would normally enjoy, build up tolerance, and have withdrawal symptoms if they try to stop using the drug. Site for question/rationale: Mental Health Disorder lecture Minute 0:50
47
7. Question: How long should a patient be experiencing symptoms in order to be diagnosed with generalized anxiety disorder? a) At least 6 months b) 1 month c) 2 weeks d) 4 months
Answer: At least 6 months Rationale for why the answer is correct: symptoms should be present for at least 6 months to make a diagnosis of Generalized Anxiety Disorder Site for question/rationale: Mental Health Disorder lecture Minute 2:00
48
8. Question: Which of the following nasal disease is affected by triggers? a) Acute Viral Rhinosinusitis b) Acute bacterial Rhinosinusitis c) Allergic Rhinitis d) Non-allergic Rhinitis
Answer: Allergic Rhinitis Rationale for why the answer is correct: The only disease process that is affected by triggers is Allergic Rhinitis. Site for question/rationale: HEENT lecture Minute 19:24
49
9. Question: Which of the following may present with nasal pruritis? a) Acute Viral Rhinosinusitis b) Acute bacterial Rhinosinusitis c) Allergic Rhinitis d) Non-allergic Rhinitis e) Both C and D
Answer: Both C and D Rationale for why the answer is correct: Both allergic rhinitis and non-allergic rhinitis may present with nasal pruritus. Site for question/rationale: HEENT lecture Minute 21:40
50
10. Question: Which of the following is not a cause of secondary headaches? a) Acute Sinusitis b) Arterial tears c) Brain tumor d) Exercise
Answer: Exercise Rationale for why the answer is correct: Exercise headaches are considered primary headaches as opposed to the other options. Site for question/rationale: Headache’s handout pg. 1-2
51
1. A 14 yo female presents after a fall from her bike while riding at a slow speed. She was attempting to break her fall by extending her arms and landed on her left shoulder. Findings on x-ray reveal a fracture through most of the physis and into the metaphysis. This is which type of Salter-Harris fracture? a. Type 1 b. Type 2 c. Type 3 d. Type 4
Answer: b. Type 2 Rationale: type 2 is across the physis and into the metaphysis; type 1 is through the physis only; Type 3 is through part of the physis and into the epiphysis; Type 4 is through all levels, and type 5 is a crush type that damages the physis by direct compression. Site for question/rationale: Radiology review Part 2 – Dr. Stritto; Slide 48
52
2. A 52 yo male patient arrives at the clinic complaining of RUQ abdominal pain after a fall from his backyard deck. He landed, abdomen first, on a short railing below the deck. The patient is tachycardic, pale, and diaphoretic. You are concerned about a liver laceration and want to perform a FAST exam. When looking at the liver via ultrasound, which of the following probes would be appropriate for viewing free fluid around the liver? a. Linear probe b. Curvilinear probe c. Phased array probe d. Both b & c
Answer: Both b & c Rationale: Even though the curvilinear probe may be best for viewing the liver, both it and the phased array are useful in abdominal exploration. The linear probe does not have the depth necessary to view the liver in detail. Site for question/rationale: Ultrasound lecture – Dr. Branham, slides 14-18 and beginning at 11:12 on video.
53
3. A patient presents with a pink papule on their nose. The periphery of the papule is raised more than the middle, and it has a translucent quality. The patient has a history of significant unprotected sun exposure as a child. You tell the patient they have the most common type of malignancy in Caucasians. What type of malignancy have you diagnosed? a. Squamous Cell Carcinoma b. Melanoma c. Basal Cell Carcinoma d. Merkel Cell Carcinoma
Answer: c. Basal Cell Carcinoma Rationale: Basal Cell Carcinomas are flesh to pink colored papules with rolled borders, a pearlescent or translucent appearance, and they are caused by UV exposure, especially in childhood. These are the most common type of malignancy in Caucasians. Site for question/rationale: Dermatology lecture – Dr. Day; Slide 5-6, 3:08 on video.
54
4. Which of the following biopsy types typically requires little training, no sutures to close the wound, and is used for lesions confined to the epidermis, such as warts? a. Punch biopsy b. Shave biopsy c. Excision d. None of the above
Answer: b. Shave biopsy Rationale: The shave biopsy is used for small lesions involving only the epidermis, whereas punch biopsies and excisions are useful for dermal layers and typically require at least one suture to close. Site for question/rationale: Dermatology lecture – Dr Day; Slide 18, 14:15 on video.
55
5. Which of the following is NOT characteristic of Non-Hodgkin Lymphoma of the skin? a. Brownish red to deep red skin nodules b. Ulcerations c. Derived from B-cell and T-cell progenitors. d. Erythematous plaques/Yellowing scales
Answer: d. Erythematous plaques/Yellowing scales Rationale: Non-Hodgkin Lymphoma of the skin is non-scaling. All the others are characteristic of Non-Hodgkin Lymphoma of the skin. Site for question/rationale: Dermatology lecture – Dr Day; Slide 11, 8:48 on video.
56
6. Daniel, a 13 yo male, presents to the clinic with his mother, who complains that he has been getting into trouble in the last 3-4 months. Based on her descriptions of the events, you believe the teen to be displaying conduct disorder. Which of the following is a symptom of conduct disorder? a. He was found kicking a dog repeatedly. b. He threw a classmate's phone on the ground and stomped on it, trying to break it. c. He took a hundred-dollar bill out of his mother's purse. d. All the above
Answer: d. all the above Rationale: Conduct disorder is characterized by school-aged children or teenagers violating the basic rights of others. All the above situations are violations. Site for question/rationale: Mental Health Disorders – Dr. Day; Video 6:11
57
7. A 22 yo male presents with his mother with a complaint of "People are trying to poison me." The mother states that he has increasing paranoia for the past few weeks, and he believes his friends are spies for the Russian government. The signs/symptoms of schizophrenia include hallucinations, delusions, incoherent speech, behavior problems, lack of expression, or purposeful activity. For a diagnosis of schizophrenia, the patient must exhibit which of the following? a. At least 2 signs/symptoms for at least 1 month b. At least 1 signs/symptoms for at least 2 months c. At least 3 signs/symptoms for at least 2 months d. At least 1 signs/symptoms for at least 1 month
Answer: a. At least 2 signs/symptoms for at least 1 month Rationale: The requirement for a diagnosis of schizophrenia is a least 2 of the above symptoms for at least 1 month. Site for question/rationale: Mental Health Disorders – Dr. Day; Video 15:33
58
8. A 32 yo female presents with a complaint of a migraine headache. This would be considered a primary headache as it stems from overactivity or problems with the head's pain-sensitive structure. Which of the following would be considered a secondary headache, a symptom of a disease that activates the head's pain-sensitive nerves? a. Cluster headache b. Hangover headache c. Tension headache
Answer: c. Hangover headache Rationale: The hangover headache is stimulated due to the alcohol, so secondarily stimulating the head's pain-sensitive nerves. Cluster and tension headaches are both primary headaches. Site for question/rationale: Headaches Handout
59
9. A 34 yo female presents with fever for the past three days, congestion, post-nasal drainage, unilateral sinus pain, and the patient states symptoms have become progressively worse. The physical exam is notable for purulent nasal secretions and erythematous nasal mucosa. Which of the following is the most likely differential? a. Allergic Rhinitis b. Non-allergic Rhinitis c. Acute Viral Rhinosinusitis d. Acute Bacterial Rhinosinusitis
Answer: d. Acute Bacterial Rhinosinusitis Rationale: The key is the unilateral sinus pain and purulent sinus drainage. Sinus pain is typically bilateral in viral rhinosinusitis and allergic rhinitis and absent in non-allergic rhinitis. Site for question/rationale: HEENT presentation – Drs Moore and Sun; Video 16:33
60
10. Your clinic patient complains of hearing problems on the right side and feels pressure on the right tympanic membrane. The physical exam is notable for cerumen impaction, for which you will attempt removal. During the irrigation procedure, the patient becomes extremely nauseous, dizzy, and you note nystagmus. What is the likely reason for these symptoms? a. You used a cold irrigation solution b. You used a warm irrigation solution c. You used saline instead of water d. You used water instead of saline
Answer: a. You used a cold irrigation solution Rationale: Irrigating the ear canal with cold fluid may cause a cold-caloric response with the above symptoms. The solution can be either saline or water and should be warmed to body temperature. Site for question/rationale: Removal of Impacted Cerumen Clinical Key Todd Thomsen, MD; Irrigation and syringing section (Clinical Pearls)
61
1. Question: On an X-ray film, when something appears white it is… a. Radiolucent b. Radiodense c. Air d. Low density
Answer: B Rationale for why the answer is correct: A white appearance on an xray is radiodense (opaque), high density, usually bone or barium. Site for question/rationale: Basic Radiology, page 2
62
2. Question: When using an ultrasound to perform a FAST exam at the bedside, you would be looking for all of the following EXCEPT: a. Free fluid b. Pneumothorax c. Organ status/injury d. Fractures
Answer: D Rationale for why the answer is correct: Ideally, the FAST exam is used on truma patients to quickly assess for free fluid or other issues in the chest/abd. Site for question/rationale: Ultrasound presentation, slide 22-28
63
3. Question: What are the key characteristics of melanoma? a. Asymmetry, regular border, one color, small in diameter, flat b. Asymmetry, Border irregularity, color variation, diameter > 5mm, evolution c. Symmetric, regular border, one color, small in diameter, flat d. Symmetric, border irregularity, color variation, diameter > 5mm, evolution
Answer: B a. Rationale for why the answer is correct: The key characterstics to assess in melanoma are Asymmetry, Border irregularity, color variation, diameter > 5mm, evolution. Site for question/rationale: Dr Mercedes Derm lecture. 00:48
64
4. Question: A 1 year old patient presents with fever, rose macules and papules on the truck and extremities. What diagnosis would you expect for this patient? a. Scarlitina b. Roseola c. Squamous Cell Carcinoma d. Melenoma
Answer: B Rationale for why the answer is correct: Roseola presents with fever followed by asymptomatic rash is children between 6 months to 1 year. This type of rash bothers parents more than the child. Site for question/rationale: Dr Mercedes Derm Lecture, 39:21
65
5. Question: What is the best way to rule in/out a melanoma or carcinoma? a. US b. Cultures c. Biopsy d. Blood test
Answer: C Rationale for why the answer is correct: While we use our full skin assessment to rule in and out different types of skin issues, a biopsy is the best way to find the specific diagnosis. Site for question/rationale: Skin lesion presentation, 13:00
66
6. Question: All of the following EXCEPT are questions you would see on a suicide risk screening. a. Have you ever tried to kill yourself? b. Do you feel sad or depressed? c. In the past few weeks, have you felt you would be better off dead? d. Are you having thoughts of hurting or killing yourself right now?
Answer: B Rationale for why the answer is correct: Suicide screening questions ask specifics about thoughts of suicide and history of suicide. Site for question/rationale: NIMH Suicide Risk Screening Tool
67
7. Question: The mother of your 16 year old female patient states that her daughter has been having hallucination and intermittent incomprehensible speech over the past few months. Which diagnosis would you consider for this patient? a. Schizophrenia b. Bipolar c. Obsessive Compulsive Disorder d. Depression
Answer: A Rationale for why the answer is correct: Schizophrenia is classified as having 2 or more of the following for over a month. These may include delusions or hallucinations, incoherent speech, catatonic states, or lack of emotion. Site for question/rationale: Mental Health Disorder Presentation, 16:02
68
Which of the following would be a negative finding in your review of symptoms in a patient diagnosed with mononucleosis? a. Painful swallowing b. Fatigue/ malaise c. Boggy lymph nodes d. Fever
Answer: C Rationale for why the answer is correct: The posterior lymph nodes would be enlarged for a patient with mono. Boggy lymph nodes are found in viral and post nasal drip issues. Site for question/rationale: ENT presentation, 15:00
69
9. Question: A patient presents to office with a history of congestion, clear nasal drainage, eyes tearing up and seems to by triggers when outdoors in the spring. What diagnosis would you give this patient? a. Acute Viral Rhinosinusitis b. Acute Bacterial Rhinosinusitis c. Allergic Rhinitis d. Nonallergic Rhinitis
Answer: C Rationale for why the answer is correct: The above symptoms coincide with allergic rhinitis. Site for question/rationale: ENT presentation, 26:34
70
10. Question: A female patient is recently diagnosed with migraines. What symptoms would you expect for her to complain of? a. “The worst headache of my life” b. “It just kinda hurts all over” c. “I threw up and the right side of my head hurt” d. “The headache started after I worked out”
Answer: C Rationale for why the answer is correct: Migraines are typically unilateral and can be accompanied by an aura, N/V, and can be worsened with excessive light or sound. Site for question/rationale: Headache pdf.
71
``` 1. Question: On a PA/LAT CXR, the major fissure is seen in the left lung. What is indicative of this finding? A. Normal finding B. CHF C. Excessive fluid D. Pneumonia ```
Answer: C. Excessive fluid. Rationale for why the answer is correct: On a CXR, fissures are not normally visualized, if fissures are seen, this is sign of excessive fluid. CHF can be seen with cardiomegaly and pleural effusion, and pneumonia is seen as a “Silhouette Sign” in lung field. Site for question/rationale: Dr. Stritto presentation “Understanding Radiology” slides #34, 41, 45, and 76.
72
2. Question: On a Lat C-Spine X-ray of an adult, what is the disc space for C1-C4? a. < 17 mm b. < 7mm c. < 21mm d. < 13mm
Answer: b. <7mm. Rationale for why the answer is correct: The disc space for C1-C4 should be <7mm in an adult. Kids should be ½ the size of the vertebral body. Site for question/rationale: Dr. Stritto presentation “Understanding Radiology” slide #107.
73
3. Question: A Caucasian patient comes into the clinic expressing concern for a growth on the side of his nose. The patient states that his growth has increased in size for the past week. The growth appears round, shiny, and erythematous. He states that it is painless but is concerned due to size. From just initial exam of growth, what skin disorder is suspected. a. Basal cell carcinoma b. Melanoma c. Merkel cell carcinoma d. Squamous Cell Carcinoma
Answer: C. Merkel Cell Carcinoma. Rationale for why the answer is correct: This is classic presentation for Merkel Cell Carcinoma. Basal cell carcinoma is very similar but is slow growing and usually has telangiectatic vessels, melanoma is not round but asymmetric with irregular borders, and Squamous Cell Carcinoma is not round but coming in papules, plaques, or nodules and usually has cauliflower appearance. Site for question/rationale: Dermatology presentation by Dr. Day at minute 7:18
74
4. Question: When using a local anesthetic on a patient with an abscess on their face, what is the correct procedure. a. Inject while pulling out on either side of abscess on one side and repeat on other side. b. Inject while pulling out under abscess. c. Refer the patient to a surgeon due to location of abscess. d. Inject while pulling out in center of abscess.
Answer: C. Refer the patient to a surgeon due to location of abscess. Rationale for why the answer is correct: If the patient has an abscess on the face, palm, soles, or elbow, a referral to surgery should be used. If the abscess were anywhere else, the correct answer would be “Inject while pulling out on either side of abscess on one side and repeat on other side.” You should never inject under or in center of abscess because it does no good to put medication directly in the lesion.
75
5. Question: A four-year-old boy comes into the clinic with complaints of rash to torso and extremities. Upon examination, patient presents with a rash that is erythematous with a rough texture. Patient also is complaining of a sore throat and has a tongue that has a white patch with a beefy red appearance. a. Roseola b. Scarlatina c. Fifth disease d. Pityriasis Rosea
Answer: B. Scarlatina Rationale for why the answer is correct: This patient presents with the classic signs of Scarlatina which are a sandpaper texture to rash, red tonsils with exudate, submandibular adenopathy, strawberry tongue, and flush cheeks. Site for question/rationale: Dermatology presentation by Dr. Day at minute 26:53.
76
6. Question: Which of the following is NOT characteristic of Bipolar disorder? a. A manic episode that lasts for a week b. Extreme sadness or lack of energy c. Difficulty concentrating d. Serious mood swings
Answer: C. Difficulty concentrating Rationale for why the answer is correct: Difficulty concentrating is not a symptom of bipolar disorder; however, it is a symptom of ADHD. The other three are all symptoms of bipolar disorder. Site for question/rationale: Common Mental Health Disorders presentation by Dr. Day at minute 17:19.
77
7. Question: CAGE is a questionnaire used to determine if a patient has an alcohol use problem, what does the “A” in CAGE stand for? a. Do you have any Apprehension concerning your alcohol use? b. Do you get Angry with yourself when you drink? c. When you drink, do you show Aggressive behavior? d. Have people Annoyed you by criticizing your drinking?
Answer: D. Have people Annoyed you by criticizing your drinking? Rationale for why the answer is correct: The CAGE questionnaire consists of 4 questions, one for each letter in acronym. “A” is question 2, if people annoy you for drinking? (CAGE Alcohol Questionaire (CAGE), n.d.) Site for question/rationale: CAGE Alcohol Questionnaire (CAGE) website.
78
8. Question: Which of the following is NOT a cause of Secondary Headaches. a. Brain Tumor b. Chemical Activity in your Brain c. Carbon Monoxide Poisoning d. Sinus Infection
Answer: B. Chemical Activity in your Brain Rationale for why the answer is correct: In primary headaches, there are issues with the pain-sensitive areas in the head. Other possible causes of primary headaches are the chemical activity in the brain, the nerves and blood vessels within the skull, and muscular structures in the head and neck area. Site for question/rationale: “Headaches” pdf file on pages 1 and 2.
79
9. Question: What is a symptom of Acute Bacterial Rhinosinusitis? a. Bilateral sinus pain b. Fever lasting longer than 48 hours. c. Clear nasal discharge. d. Nasal pruritis
Answer: B. Fever lasting longer than 48 hours. Rationale for why the answer is correct: Patient’s with Acute Bacteria Rhinosinusitis have a fever that lasts longer than 48 hours, these patients are sicker longer than patients with Acute Viral Rhinosinusitis who have a fever than lasts from 24 to 48 hours. Patient with Acute Bacteria Rhinosinusitis will also have a purulent discharge, unilateral sinus pain, and will not have any pruritis. Site for question/rationale: Ear, Nose and Throat presentation by Dr. Amy Moore and Dr. Grace Sun at minute 17:03
80
10. Question: When applying the fluorescein dye to a patient’s eye, you must: a. Raise the lower lid and put the tip of the wet fluorescein strip under the conjunctiva of the lower eye. b. Wet the fluorescein strip and allow a drop of the dye to drip in the lower lid of the eye. c. Avoid having the patient blink for 1 minute after instillation of dye. d. Raise the upper lid and put the tip of the wet fluorescein strip under the conjunctiva of the upper eye.
Answer: A. Raise the lower lid and put the tip of the wet fluorescein strip under the conjunctiva of the lower eye. Rationale for why the answer is correct: The correct procedure is to apply wet strip to lower eye and have the patient blink to distribute the dye of the entire cornea until the surface is uniformly green. Site for question/rationale: “Instilling the Fluorescein Dye in the Eye” video at minute 0:11.
81
1. Question: As an APRN you know that when ordering X-rays, its best to order more than one view because: A. Insurance doesn't cover X-rays if only one view is ordered because one view can’t be used to definitively diagnose the patient B. More than one X-ray view has an increased sensitivity therefore an increased chance of identifying a problem C. X-rays can not be read and interpreted by radiologists without two or more views D. Ordering two or more X-ray views increases your chance of being sued for malpractice
Answer: B. More than one X-ray view has an increased sensitivity therefore an increased chance of identifying a problem Rationale for why the answer is correct: More than one X-ray view allows you to see things that could be covered up or hidden in a previous X-ray view. You have an increased chance of identifying a problem and correctly diagnosing a patient and therefore treating there correctly. Two views are better than one and the body is 3 dimensional, an x-ray is not. Site for question/rationale: Radiology lecture and PowerPoint from our mandatory “on campus” day by Dr. Rita Dello Stritto
82
``` 2. Question: The heart on an AP chest X-ray view when compared to a PA chest x-ray view appears A. Larger B. Smaller C. Tilted to the right D. Tilted to the left ```
Answer: A. Larger Rationale for why the answer is correct: The AP view the heart is further away from the x ray film while on the PA view the heart is closer to the film. This distorts the AP view of the heart because the heart is further away. An AP view is taken from the front and the film is placed behind the patient’s back, so the “picture” of the heart is projected onto the film (kind of like a shadow) larger than a PA view. Site for question/rationale: Radiology lecture and PowerPoint from our mandatory “on campus” day by Dr. Rita Dello Stritto
83
``` 3. Question: All of the following are apart of the ABCDE criteria for melanoma EXCEPT A. Asymmetry B. Boarder irregularities C. Depth D. Evolution ```
Answer: C. Depth Rationale for why the answer is correct: Asymmetry, boarder irregularities, color variegation, diameter >6mm, and evolution are the most important factors in examining a lesion for melanoma factors. Depth is not a defining criteria for melanoma. Site for question/rationale: Dermatology lecture by Dr. Mercedes Day
84
4. Question: Which population is at highest risk of developing melanoma A. Asian males greater than 50 years of age B. Family history of melanoma C. Occasional sun exposure as a child/adolescent/young adult D. Total nevus count above 10 but below 20
Answer: B. Family history of melanoma Rationale for why the answer is correct: Most at risk populations for developing melanoma are white males over the age of 50, family history of melanoma, total nevus count above 25, one of more atypical nevi, significant childhood, adolescent, or young adult sun exposure including one painful or blistering sunburn under 30 years of age, chronic outdoor activities without photo protection, indoor tanning beds, immunocompromising conditions, and red hair phenotype. Site for question/rationale: Dermatology lecture by Dr. Mercedes Day
85
``` 5. Question: Squamous cell carcinoma is most commonly found A. On the back B. On the tops of the hands C. On the scalp D. On the face and neck ```
Answer: D. On the face and neck Rationale for why the answer is correct: Lesions are most commonly found on the head and neck (more than 50% of cases). Squamous cell carcinoma can also be found on the dorsum of the hands and forearms, legs, arms, shoulder, or back, chest or abdomen. Site for question/rationale: Dermatology lecture by Dr. Mercedes Day
86
6. Question: Which of the following is TRUE regarding postpartum depression? A. Although they may feel down, moms with postpartum depression don't have difficulty bonding with their baby B. Postpartum depression can only occur after delivery, not during pregnancy C. Postpartum depression is a serious debilitating illness lasting longer than 2 weeks D. Mothers with postpartum depression typically have increased energy levels
Answer: C. Postpartum depression is a serious debilitating illness lasting longer than 2 weeks Rationale for why the answer is correct: Mothers diagnosed with postpartum depression typically have difficulty bonding with their baby, loss of energy, and depressed mood or severe mood swings. Additionally, postpartum depression can occur during pregnancy and at any point up to 6 months after delivery.
87
``` 7. Question: All of the following mental health disorders can accompany PTSD EXCEPT: A. Depression B. Substance abuse C. Anxiety D. ADHD ```
Answer: D. ADHD Rationale for why the answer is correct: Depression, substance abuse and anxiety can all be present with PTSD. They use substances to cope and may feel depressed and anxious due to avoidance behaviors, re-experiencing trauma, or fear about encountering triggers. They often feel detached and isolated from others. ADHD does not accompany PTSD. Site for question/rationale: Dr. Mercedes Day PowerPoint on Common Mental Health Disorders
88
``` 8. Question: A 14 y/o patient comes in with complaints of a painful unilateral red eye with a sudden onset. She has no other symptoms and reports no recent injury to her eye. You suspect that she has: A. Viral conjunctivitis B. Foreign object injury C. Bacterial conjunctivitis D. Eye floaters ```
Answer: C. Bacterial conjunctivitis Rationale for why the answer is correct: Viral conjunctivitis usually had a gradual onset that ends up in both eyes with accompanies symptoms (like the common cold). The patient states she did not sustain any injury to her eye so we known its not a foreign object injury, and eye floaters are not accompanied by pain or redness of the eye. Bacterial conjunctivitis most commonly occurs in one eye with a rapid onset and no other viral symptoms. Site for question/rationale: Ear, nose, and throat presentation by Dr. Amy Moore and Dr. Grace Sun
89
``` 9. Question: A patient you are assessing has swollen lymph nodes in their neck. You know you can safely rule out which of the following as a possible cause for the lymphadenopathy? A. Trauma B. Bacterial infection C. Allergies D. Neuropathic causes ```
Answer: A. Trauma Rationale for why the answer is correct: Trauma cannot cause lymphadenopathy. Swollen lymph nodes are an immune/lymphatic response to an infection that is draining down. Site for question/rationale: Ear, nose, and throat presentation by Dr. Amy Moore and Dr. Grace Sun
90
``` 10. Question: You have a patient that's monospot test just came back positive. As an APRN you know that which of the following causes mono? A. Adenovirus B. Epstein-Barr Virus C. Streptococcus pyrogenes D. Cytomegalovirus ```
Answer: B. Epstein-Barr virus Rationale for why the answer is correct: Streptococcus pyrogenes is the bacterium that causes pharyngitis and while the adenovirus and cytomegalovirus cause similar symptoms to mono, they are not the cause. Site for question/rationale: Ear, nose, and throat presentation by Dr. Amy Moore and Dr. Grace Sun
91
1. Question: An APRN is asked to perform a F.A.S.T. exam on a trauma patient that has just arrived. The APRN knows a FAST is a a. Focused abdominal scan for trauma utilizing an ultra sound device b. Focused abdominal scan for trauma utilizing CT imaging c. Focused aortic scan for trauma utilizing ultra sound d. Focused abdominal scan for trauma that is utilized in the operating room primarily
Answer: a Rationale for why the answer is correct: The FAST exam is performed with an ultra sound to identify abnormalities in trauma patient’s abdominal cavity. These tests are performed at bedside and are highly effective without the delay of CT. Site for question/rationale: Dr. Branham’s “Ultrasound” lecture (18:23)
92
2. Question: What are the first 3 steps that should be performed when reading an X-Ray? a. Assess for fractures, free fluid, and pneumothoracies b. Check the name, date, and previous studies for comparison c. Count the ribs, measure heart size, and identify borders d. None of the above- wait for the radiologist report then review the X-Ray
Answer: b Rationale for why the answer is correct: Dr. Stritto impresses the importance of ensuring you are reviewing the correct patient and time the X-ray was performed. Reviewing previous studies can show how something has improved or declined since the last exam. Site for question/rationale: Dr. Stritto’s lecture “Basic Radiology” (Slide 22)
93
3. Question: A parent arrives to your clinic with an 8-month-old daughter stating she has been running a fever of 102 degrees for three days. Upon examination, you notice blanchable rose-pink macules and papules on her chest and arms. Otherwise, the exam in normal. With this information, what is the most likely diagnosis for this patient? a. Scarlatina b. Molluscum c. Roseola d. Eczema Herpeticum
Rationale for why the answer is correct: Roseola’s onset usually begins with a fever that lasts approximately four days. The fever is followed by a blanchable rose-pink rash that starts on the trunk and may spread to the extremities. Site for question/rationale: Dr. Day’s “Dermatology” lecture (29:08)
94
4. Question: An absolute contraindication of cryosurgery is a. Actinic keratoses b. Molluscum Contagiosum c. Skin tags d. Malignant melanoma
Answer: d Rationale for why the answer is correct: Malignant melanoma is an absolute contraindication of cryosurgery Site for question/rationale: (Tuggy et al., 2020)
95
5. Question: When you are using a punch biopsy, it is important to note that a. It will remove the full thickness of the skin b. It is a shallow instrument that only removes the epidermal layer c. The biopsy must always be a smaller diameter than the suspected cancer d. It is used to excise a large (>3 cm) area of skin
Answer: a Rationale for why the answer is correct: The punch biopsy cuts through and removes all layers of skin in order to get a deep sample of tissue. It is used for smaller diameter biopsies (up to approximately 1 cm)and should be a larger diameter than the borders of suspected lesion. Site for question/rationale: Dr. Sun and Dr. Moore’s “Skin Disorders” lecture (11:19)
96
6. Question: When diagnosing a patient with schizophrenia, the APRN knows that 2 or more of the following symptoms must be present for at least month. These symptoms include incoherent speech, behavior problems, lack of emotional expression/lack of purposeful activity, as well as a. Flashbacks of traumatic event b. Harm to animals and people c. Lack of interest in normally pleasurable activities d. Delusions and hallucinations
Answer: d Rationale for why the answer is correct: Delusions and hallucinations are part of the diagnostic criteria for schizophrenia as well as the symptoms present in question proposed. Site for question/rationale: Dr. Day’s lecture “Common Mental Health Disorders” (16:26)
97
7. Question: Regarding the CAGE Alcohol Questionnaire, what score indicates the individual may have a problem with alcohol a. 0 b. 1 c. >2 d. >5
Answer: c Rationale for why the answer is correct: The CAGE questionnaire is scored 0-4. A score of 2 or greater indicates a likelihood of an alcohol problem.
98
8. Question: You are an APRN seeing a female adult patient who is complaining of left sided headache. She states it started yesterday. She feels nauseous, and described an “aura” preceding the headache. She states she feels better laying down in a dark quiet room. Based on this information, your most likely diagnosis is? a. Cluster headache b. Tension headache c. Migraine headache d. Dysmenorrhea
Answer: c Rationale for why the answer is correct: Common findings with a migraine headache include: unilateral pain, aura, nausea, female, hormonal changes, and relief in quiet dark spaces. Site for question/rationale: Dr. Moore and Dr. Hilliard’s lecture “Headaches” (4:10)
99
9. Question: You are a FNP working at a clinic when a patient arrives in moderate distress. He states, “I have the worst headache of my entire life”. He can hardly sit still during a brief exam and his blood pressure is 208/103. He is a long-time smoker and has a history of hypertension. Your priority diagnosis and action for this patient is a. Intracerebral hemorrhage- Send to ER immediately b. Migraine headache- Write for Sumatriptan c. Ischemic stroke- Administer aspirin d. Hypertension- Finish your full history and physical exam before proceeding with further action
Answer: a Rationale for why the answer is correct: This patient is likely experiencing a significant intracranial event with a high potential of mortality such as an intracerebral hemorrhage or ischemic event. Priority action for this patient is transfer to higher acuity center with a CT capabilities.
100
10. Question: A patient arrives to your clinic with complaints of left eye redness, drainage, and itchiness, that occurred suddenly. With this information, your most likely diagnosis for this patient is a. Viral conjunctivitis b. Macular degeneration c. Bacterial conjunctivitis d. Glaucoma
Answer: c Rationale for why the answer is correct: Viral conjunctivitis usually is gradual onset with associated cough/cold symptoms. Bacterial is typically sudden and unilateral with symptoms of redness, itchiness, and drainage. Site for question/rationale: Dr. Sun and Dr. Moore’s lecture “Ear, Nose, and Throat” (1:35)
101
1. Question: Why is it best for the APRN to order more than one view X-Ray? A. To bill the insurance B. Because it’s standard practice C. There is an increased chance of finding a problem D. Because of Medicare/Medicaid rules and regulations
Answer: C Rationale for why the answer is correct: Two views have increased sensitivity and an increased chance of finding a problem. Two views are better than one view. Site for question/rationale: Basic Radiology slide 12
102
2. Question When should the APRN order an AP view chest X-ray? A. When the patient can stand B. When you need an oblique view C. When you want an accurate view of the heart size D. When the patient is unable to stand
Answer: D Rationale for why the answer is correct: A PA and Lat view is for patients that can stand. An AP view is for patients who cannot stand. With the AP view, the heart is further away, therefore distorted and unable to estimate the heart size. Site for question/rationale: Basic radiology slide 18-19
103
``` 3. Question: What is the most severe form of skin cancer? A. Melanoma B. Basal cell cancer C. Squamous cell cancer D. Merkel cell cancer ```
Answer: A Rationale for why the answer is correct: Melanoma is the most serious type of skin cancer. Most arise as superficial tumors confined to the epidermis. Site for question/rationale: Dr. Day dermatology lecture 0.35
104
``` 4. Question: When diagnosing a skin lesion using ABCDE, what does the A stand for? A. Angle B. A lot of sun to the area C. Asymmetry D. Area ```
Answer: C Rationale for why the answer is correct: A stands for asymmetry. Look to see if the lesion is the same all the way around. Is it irregular. B stands for border. C stands for color. D stands for diameter. E stands for evolving. Site for question/rationale: Skin Lesion presentation 0.45
105
5. Question: Adding epinephrine to lidocaine will extend the anesthetic effect to A. 1-2 hours B. 7-9 hours C. 4-6 hours D. Epinephrine will not extend the anesthetic effect
Answer: C Rationale for why the answer is correct: Epinephrine can be added to lidocaine and it will extend the anesthetic effect of lidocaine by about 4-6 hours. The APRN must be careful where he/she injects epinephrine. There is a lot of debate about using lidocaine with epinephrine in areas such as in the digits, nose, ears, toes, fingers. Site for question/rationale: Overview of local anesthetics with punch biopsy lecture 0.31
106
6. Question: What are the criteria to make a diagnosis of substance use disorder? A. The patient must abuse illicit drugs only to have a substance use disorder. B. If the patient has two symptoms, such as craving and failure to fulfill other obligations within the past 12 months. C. If a patient has one symptom, such as withdrawal symptoms within the past 12 months. D. When a patient has two symptoms such as craving and failure to fulfill other obligations
Answer: B Rationale for why the answer is correct: Substance use disorder involves alcohol or drug abuse that impairs daily life or causes noticeable distress. Diagnosis criteria includes 2 of the following symptoms (consuming more than planned, worrying about failed efforts to control use, time using or obtaining drugs, failure to fulfill other obligations, craving, continued use, repeated use in dangerous situations, not doing activities they usually would enjoy, building up a tolerance, withdrawal symptoms) within the past 12 months. Site for question/rationale: Mental Health Disorders lecture 1:27
107
7. Question: What is the asQ screening tool? A. Suicide screening tool for geriatric patients B. Alcohol screening tool for adolescents C. STD screening tool D. Suicide screening tool for ages 10-24 years of age
Answer: D Rationale for why the answer is correct: The asQ screening tool was developed in 3 pediatric emergency departments for non-psychiatric clinicians to screen for suicide in patients 10-25 years of age. Any “yes” answer to the first four questions is a positive screen. Site for question/rationale: Depression screening information for youth
108
8. Question: What is the difference between viral and bacterial conjunctivitis? A. Viral has a sudden onset B. The patient will have a cough for a few days then have red eyes on the third day with bacterial conjunctivitis C. Bacterial has a sudden onset D. There is no difference between viral and bacterial conjunctivitis
Answer: C Rationale for why the answer is correct: Viral conjunctivitis will usually be preceded by a cough, while bacterial conjunctivitis will have a sudden onset. Bacterial conjunctivitis usually involves one eye, while viral conjunctivitis may start in one eye, but it will move to the other eye with cross-contamination. Site for question/rationale: HEENT presentation 1:40
109
9. Question: How can the APRN differentiate between headaches? A. Muscle tension headaches are usually on one side of the head B. Migraine headaches occur more in women, and the patient may be more sensitive to light C. Cluster headaches usually occur in the middle of the day, and the patient wants to sit still D. Cluster headaches occur more in females
Answer: B Rationale for why the answer is correct: With muscle tension headaches, the patient feels like a band is wrapped around his/her head. The headache can last several minutes to several hours. Migraine headaches occur more in females, and the pain is usually on one side. Migraine headaches typically last 4-72 hours. The patient may have an aura, see flashes, be sensitive to light, sensitive to odor, have nausea, usually wants to sit still. Cluster headaches usually occur in the middle of the night. The patient usually wakes up with pain in the periorbital area with tears in the affected eye, nasal drainage out of that one side, and the patient does not want to sit still. Cluster headaches are more common in male patients. Site for question/rationale: Headache differential lecture
110
``` 10. Question: How will the APRN treat a patient who states, “This is the worst headache of my life”! A. Refer the patient to a neurologist B. Prescribe a calcium channel blocker C. Prescribe an NSAID D. Send the patient to the ER ```
Answer: D Rationale for why the answer is correct: If a patient states, “this is the worst headache of my life”, it’s a red flag for the APRN. This type of headache is also described as a thunderclap headache. It’s best to send the patient to the emergency room because they could have a brain bleed. A CAT scan is the recommended test for the patient. Site for question/rationale: Headache differential lecture
111
1. Question: What chest x-ray views would be ordered to examine the heart? a) Lateral and oblique b) AP and oblique c) PA and lateral d) Oblique and PA
Rationale for why the answer is correct: The PA view is preferred due to the heart being closer to the film. The AP view distorts the image due to the location of the heart. Lateral shows the side of the heart. More than 1 view increases the chances of finding the problem. Site for question/rationale: Dr Rita Stritto’s Understanding Radiology lecture Slide 18 and 19.
112
3. Question: Which part of the body is most affected by basal cell carcinoma? a) Back b) Face c) Chest d) Lower limbs
Answer: b) Face Rationale for why the answer is correct: Basal cell carcinoma is directly linked to exposed of ultraviolet light. The face is most often exposed to the sun causing an increased risk. 70% of Basal cell carcinomas occur on the face and head. Site for question/rationale: Dr Day’s Dermatology Lecture 4:10 slide 4
113
4. Question: A 48-year-old male presents with a greasy yellowish rash that involves his chest, nasolabial area, and eyebrows. Which of the following is most likely the diagnosis? a) Seborrheic Dermatitis b) Basal Cell Carcinoma c) Kaposi Carcoma d) Psoriasis
Answer: a) Seborrheic Dermatitis Rationale for why the answer is correct: Seborrheic Dermatitis presents as well demarcated, erythematous plaques with greasy-looking yellow scales. Site for question/rationale: Dr Day’s Dermatology Lecture 12:30
114
5. Question: A 5-year-old male presents with a sandpaper- like rash on the trunk and groin area, enlarged tonsils, and a strawberry like tongue. Which of the following is most likely the diagnosis? a) Fifth Disease b) Roseola c) Ezcema Herpeticum d) Scarlatina
Answer: d) Scarlatina Rationale for why the answer is correct: Scarlatina presents with Sandpaper like rash, enlarged tonsils and strawberry line tongue. Site for question/rationale: Dr Day’s Dermatology lecture 27:00
115
6. Question: Which of the Following is NOT a symptom of ADHD? a) Poor organization skills b) Struggling with listening in class c) Obsessions d) Fidgeting or squirming
Answer: c) Obsessions Rationale for why the answer is correct: Obsessions are a symptom of OCD not ADHD Site for question/rationale: Dr Day’s Mental Health Lecture Slide 4 2:39
116
7. Question: Which of the following describes someone with Generalized Anxiety Disorder? a) Mary is not sleeping well and having nightmares after being robbed 2 days ago. b) Beth has been anxious and worried for the past year. Most of the day and almost every day. c) Barry has been having flashbacks from his time in the war. d) Mel has severe anxiety when she needs to give an oral presentation in front of the class.
Answer: b) Beth has been anxious and worried for the past year. Most of the day and almost everyday Rationale for why the answer is correct: Generalized Anxiety disorder is excessive anxiety or worry with symptoms lasting at least 6 months. Site for question/rationale: Dr Day’s Mental Health lecture slide 3 1:55
117
8. Question: A 15-year-old female presents with abrupt onset ear pain rating 7/10, fever of 100.6, some nasal discharge, and reactive lymph nodes. Which of the following would most likely be the diagnosis? a) Bacterial ear infection b) Seasonal Allergies c) Ear Trauma d) Neuropathic Ear pain
Answer: a) Bacterial ear infection Rationale for why the answer is correct: Bacterial ear infection was ruled in due to abrupt onset of ear pain. All others were ruled out for the same reason. Site for question/rationale: HEENT Lecture 6:40
118
9. Question: A 14-year-old male presents to the clinic with a sore throat. He rates his pain at 6/10 with a gradual onset. He admits to problems swallowing. He stated he has had nasal congestion and postnasal drip for approximately 5 days. He denies fever or fatigue. Which of the following is most likely the cause of his sore throat? a) Postnasal drainage b) Viral illness c) Mono d) Strep
Answer: a) Postnasal Drainage Rationale for why the answer is correct: Postnasal Drainage was ruled in due to the lack of fever and fatigue, the presence of nasal congestion and postnasal drip, and the gradual onset of sore throat. Site for question/rationale: HEENT lecture 12:18
119
10. Question: A 34-year-old male presents with facial pain 7/10 increasing when bent over. He stated the pain is only on my left cheek! He admits to purulent nasal drainage only from the left nostril. He stated he has had a fever for over 2 days! Which of the following is most likely the diagnosis? a) Acute Bacterial Rhinosinusitis b) Allergic Rhinitis c) Nonallergic Rhinitis d) Acute Viral Rhinosinusitis
Answer: a) Acute Bacterial Rhinosinusitis Rationale for why the answer is correct: Acute bacterial rhinosinusitis was ruled in due to fever lasting more than 48 hours, unilateral purulent nasal drainage, and unilateral facial pain. Site for question/rationale: HEENT Lecture 17:00
120
1. Question: What does the acronym eFAST stand for A. Extended Focused Abdominal Scan for Trauma B. Electronic Focal Abdominal Scan for Trauma C. Electronic Field Abdominal Scan for Trauma D. Electromagnetic Field Abdominal Scan for Trauma
Answer: A Rationale for why the answer is correct: eFAST is a fast and easy way to check for blood in the chest and abdomen, especially useful in trauma situations Site for question/rationale: Principles of Ultrasound /Point of care ultrasound power point.
121
2. Question: Describe two benefits for using ultrasound over conventional radiographic procedures. A. Very good through bone – cannot be used for fractures B. No ionizing radiation C. Ability to directly see your findings verses inferred by physical exam and historical findings. D. Can be more costly than conventional radiology, CT, MRI.
Answer: B, C Rationale for why the answer is correct: Ultrasound uses high frequency sound waves to view inside the body. Site for question/rationale: Principles of Ultrasound /Point of care ultrasound power point
122
3. Question: What are the ABCDE designations given to Melanoma A. Asymmetry, Border irregularities, Color variegation, Diameter, Evolution B. Asymmetry, Border, Color, Depth, Evolution C. Anamorphism, Boundary, Complexion, Development, Expansion D. Anatomic, borderline, cellular, disease, evolving
Answer: A Rationale for why the answer is correct: Warning signs of Melanoma. Site for question/rationale: Dr. Mercedes video on dermatology. .58 on video
123
``` 4. Question: What is the most common malignancy in Caucasians A. Squamous cell carcinoma B. Basal cell carcinoma C. Malignant Melanoma D. Merkel cell carcinoma ```
Answer: B Rationale for why the answer is correct: Basal cell carcinoma is the most common type of skin cancer, starting in the basal cell layer of the skin, usually occurs on the face, scalp or anywhere that is exposed to continuous sunlight. Site for question/rationale: Dr. Mercedes Day video on dermatology. 4.27 on video
124
5. Question: What are the four types of infection associated with Kaposi sarcoma-associated Herpes virus (KSHV). Select the four that apply A. Extragenic B. Classic C. Endemic D. Iatrogenic E. Acquired immunodeficiency syndrome (AIDS) associated
Answer: B, C, D, E Rationale for why the answer is correct: This form of cancer causes lesions sometimes patches to grow in or on the skin, lymph nodes, mucous membranes lining of the mouth, nose, and throat. Often affects people with HIV, AIDS. Differentiates in color from purple, brown, red, blotchy skin, tumors may develop in other areas of the body. Site for question/rationale: Dr. Mercedes Day video on dermatology. 11.19 on video
125
``` 6. Question: What is the name for a 4-item questionnaire that can indicate potential problems with alcohol abuse. A. RACE B. DAST C. CAGE D. NIDA ```
Answer: C Rationale for why the answer is correct: Four clinical interview questions, the CAGE questions, have proved useful in helping to make a diagnosis of alcoholism. The questions focus on Cutting down, Annoyance by criticism, Guilty feeling, and Eye-openers. The acronym "CAGE" helps the physician to recall the questions. How these questions were identified and their use in clinical and research studies are described. Site for question/rationale: CAGE scale questionnaire / Alcohol Screening Tool.
126
``` 7. Question: Conduct disorder is a mental health disorder that typically presents with at least 3 or more of the following criteria within the past 12 months and at least 1 of the criteria listed within the past 6 months. Select all that apply. A. Aggression to people and animals B. Been arrested multiple times C. Destruction of property D. On probation E. Deceitfulness or theft F. Serious violations of rules ```
Answer: A, C, E, F Rationale for why the answer is correct: Conduct disorder is a serious behavioral and emotional disorder that is usually diagnosed as school aged children or teenagers where societal norms or rules are violated. Site for question/rationale: Common Mental Health Disorder video. Dr Mercedes Day. 6.29 on video
127
``` 8. Question: This emergent situation usually occurs when the light sensitive membrane in the back of the eye separates from the nerve tissue and blood supply underneath it. A. Ocular hypertension B. Glaucoma C. Eye floater or flash D. Retinal Detachment ```
Answer: D Rationale for why the answer is correct: An emergency when part of the eye (retina) pulls away from supporting tissue. Site for question/rationale: PDF for EYES
128
``` 9. Question: The term “Proptosis” is also known as A. Lazy eye B. Cross Eye C. Bulging eye D. Retinal detachment ```
D. Retinal detachment Answer: C Rationale for why the answer is correct: Bulging eyes or proptosis according to the PDF Site for question/rationale: PDF for EYES
129
`10. Question: A secondary headache is a symptom of a disease that can activate the pain-sensitive nerves of the head. Any number of conditions — varying greatly in severity — may cause secondary headaches. Select all that apply. A. Acute sinusitis (sinus infection) B. Certain foods, such as processed meats that contain nitrates C. Brain tumor D. Dehydration E. Trigeminal neuralgia
Answer: A, C, D, E Rationale for why the answer is correct: Secondary headache according to the PDF are due to an underlying medical condition such as sinus infection, or injury. Site for question/rationale: PDF for HEADACHES
130
1. Question: Which of the following is considered radiodense or opaque? a. Air b. Fat c. Muscle d. Bone
Answer: Bone Rationale for why the answer is correct: Bone or barium are considered radiodense or opaque. Site for question/rationale: Basic Radiology Lecture Page 2 Slide 3
131
2. Question: When looking at a Chest X-ray in the PA view, the cardiac silhouette should not exceed more than ___ the total width of the chest? a. 1/3 b. 1/4 c. 1/2 d. 3/4
Answer: 1/2 Rationale for why the answer is correct: Cardiac silhouette should not exceed >1/2 total width of the chest. If so, you should consider it to be caused by CHF, cardiomyopathy, or pericardial effusion. Site for question/rationale: Basic Radiology Lecture Page 12 Slide 1
132
3. Question: Brenda, a 36-year old patient, presents to your clinic today. She states she noticed several dull red lesions with pustules a few days ago. What differential diagnosis would you give her? a. Actinic Keratoses b. Seborrhea c. Rosacea d. Cancer
Answer: Rosacea Rationale for why the answer is correct: Rosacea is considered to be a dull red skin lesion with pustules and smoothness of skin present. Site for question/rationale: Differential Diagnosis Textbook Page 259
133
4. Question: When assessing a skin lesion for possible melanoma, what is NOT one of the factors to look for? a. Color variegation b. Asymmetry c. Diameter more than 6 mm d. Smooth, even borders e. Evolution
Answer: Smooth, even borders Rationale for why the answer is correct: When assessing a skin lesion for melanoma, you should look for asymmetry, border irregularities, color variegation, a diameter more than 6 mm, and evolution. Site for question/rationale: Dermatology Lecture Melanoma Slide
134
5. Question: A 52-year old farmer presents to clinic with concerns of a skin lesion on his nose. He described it as pink and translucent in color with a telangiectatic vessel inside. What would be your leading differential diagnosis? a. Melanoma b. Basal Cell Carcinoma c. Merkel Cell Carcinoma d. Squamous Cell Carcinoma
Answer: Basal Cell Carcinoma Rationale for why the answer is correct: Basal Cell Carcinomas appear as a pink or flesh colored papule (rolled border where periphery is more raised than the middle), pearly or translucent quality, telangiectatic vessel within the papule, and may also appear as an ulceration Site for question/rationale: Dermatology Lecture Basal Cell Carcinoma Slide
135
6. Question: An 18-year old college student presents to clinic complaining of having less energy than normal, a poor appetite, and trouble sleeping. What would be an appropriate assessment tool to help diagnose the patient of depression? a. PHQ-9 b. CAGE Scale c. Hamilton d. Beck Scale
Answer: PHQ-9 Rationale for why the answer is correct: PHQ-9 is an assessment tool used to diagnose depression ranging from minimal depression to severe depression. Site for question/rationale: PHQ-9 Presentation
136
7. Question: In order to be diagnosed with a substance use disorder, the patient must display __ of the following symptoms within the past 12 months: consuming more than planned, worrying or failed efforts to control use, time using or obtaining drugs, failure to fulfill other obligations, craving, continued use, repeated use in dangerous situations, not doing activities they normally would enjoy, building up a tolerance, or having withdrawal symptoms. a. 1 b. 2 c. 3 d. 4
Answer: 2 Rationale for why the answer is correct: 2 of the following symptoms listed above must be present within the past 12 months to be diagnosed with a substance use disorder. Site for question/rationale: Substance Use Disorder slide on the Mental Health Disorders Lecture
137
8. Question: Which of the following is NOT considered a primary headache? a. Migraine b. Cluster headache c. Concussion d. Tension headache
Answer: Concussion Rationale for why the answer is correct: The most common primary headaches include a cluster headache, migraine, migraine with aura, and tension headaches. Site for question/rationale: Headache handout
138
9. Question: Keratoconus, a common eye disorder, is defined as? a. An infection of the retina b. A build-up of fluid in the eye that creates eye pressure, damaging the optic nerve c. The cornea becomes thin and cone shaped d. A fluid accumulation in the macula
Answer: The cornea becomes thin and cone shaped Rationale for why the answer is correct: Keratoconus is when the cornea, which is normally round, becomes thin and cone shaped. Site for question/rationale: Eyes Handout
139
10. Question: Seasonal allergic rhinitis is caused by a hypersensitivity to what? a. Cigarette smoke b. Ragweed c. Plant pollen d. Seaweed
Answer: Ragweed Rationale for why the answer is correct: Seasonal allergic rhinitis may occur in various times of the year and caused by a hypersensitivity to ragweed. Site for question/rationale: Nose Handout
140
2. Question: Which of the following are characteristics of radiodense materials? a. They have a high density and appear white. Bone is a good example. b. They have a high density and appear black. Air is a good example. c. They have a low density and appear black. Air is a good example. d. They have a low density and appear white. Bone is a good example.
Answer: A Rationale for why the answer is correct: Radiodense materials reflect x rays, so they appear white on film. They are dense materials. Bone, barium, and fluid are good examples of radiodense materials. Site for question/rationale: Rita Stritto Radiology from Novice to Expert Part 1 Lecture Slide 6.
141
3. Question: Which of the following lesions is most concerning for cancer? a. A 2 mm, round, one colored lesion b. A 6 mm, asymmetrical, multicolored new lesion c. A 1 mm well defined lesion old lesion d. A 4 mm, symmetrical lesion
Answer: B Rationale for why the answer is correct: according to the ABCDE acronym. Asymmetry, Border irregular, Color variation, Diameter > 6mm, Evolution/changing in size/shape/color/new lesion. Site for question/rationale: Mercedes Day Dermatology lecture, Module 3
142
4. Question: Which virus is associated with 5th’s disease? a. HIV b. Influenza c. Parvovirus d. Herpes Simplex 1
Answer: C Rationale for why the answer is correct: Parvovirus is the causative organism for 5th disease. Site for question/rationale: Mercedes Day Dermatology Lecture, Module 3
143
5. Question: Which of the following patients is most at risk for developing melanoma? a. George, an 82-year-old, red haired Caucasian male who worked as a brick mason for 40 years before retiring whose sister had a melanoma lesion removed ten years ago. b. Tammy a 20-year-old African American female who religiously wears sunscreen. c. Fred a 70-year-old African American male who worked indoors most of his life. d. Alicia an 18-year-old Asian American who likes to frequent the tanning bed.
``` Answer: A Rationale for why the answer is correct: George has more risk factors for developing melanoma than the other patients. i. Caucasian males >50 ii. Family history melanoma 1. Three melanoma on each side of family 2. Have lots of nevi = super high risk iii. 1+ nevi iv. significant sun exposure as adolescent/kiddos 1. 1 blistering/painful sunburn < 30 years old v. indoor tanning beds vi. immunocompromised vii. red headed ``` Site for question/rationale: Mercedes Day Dermatology Lecture, Module 3
144
6. Question: Lilly has come to your clinic complaining of the following symptoms: having flashbacks of her family drowning, inability to concentrate, inability to sleep, having a sense of blame, and becoming detached from her social group. Which differential is most likely Lilly’s diagnosis? a. Post-Partum Depression b. Substance Abuse Disorder c. Schizophrenia d. Post-Traumatic Stress Disorder
Answer: D Rationale for why the answer is correct: Lilly is exhibiting signs most closely related to PTSD. Site for question/rationale: Mercedes Day’s Mental Health Disorders Video, Module 4
145
7. Question: Susie has brought her baby in for her two- month wellness visit. As you are chatting, Susie confides in you that she is having severe mood swings, panic attacks, does not feel bonded to her baby, thinks she is a bad mother, and is more withdrawn from family and friends. Which mental health disorder is most likely the cause of Susie’s feelings? a. Schizophrenia b. Depression c. Bipolar 1 disorder d. Post-Partum Depression
Answer: D Rationale for why the answer is correct: All these concerns are indicative of Post-Partum Depression. Site for question/rationale: Mercedes Day’s Mental Health Disorders Video, Module 4
146
8. Question: The following patients present to your clinic. Which clinical picture is the most concerning? a. James, a 20-year-old male who has woken up in excruciating unilateral periorbital pain. b. Tera, who is complaining of a severe headache. She also presents with photophobia and nausea. c. Calvin, who claims he is having the “worst headache of his life”. d. Gloria, who has been more stressed with her job and family life lately. She complains of a band like headache.
Answer: C Rationale for why the answer is correct: This patient could be having a brain bleed and must immediately go to the ER for further evaluation. Site for question/rationale: Headache Differentials video by Dr. Moore and Dr. Hillard, Module 3
147
9. Question: Trevor, an 18-year-old male who has been practicing for the upcoming soccer season visits your clinic with the following complaints: Clear nasal drainage often times alternating with nasal congestion, sinus pain, nasal puritis, and post nasal drip. Which is the most likely diagnosis? a. Bacterial Rhinosinusitis b. Viral Rhinosinusitis c. Allergic Rhinitis d. Non-Allergic Rhinitis
Answer: C Rationale for why the answer is correct: He is showing signs of allergic rhinitis. Site for question/rationale: HEENT presentation nasal differentials. Dr. Moore and Dr. Sun, Module 4
148
10. Question: In differentiating types of headaches what diagnostic skills are most important? a. Labs like CBC or CMP b. Imaging like a Cat scan or an MRI c. Obtaining a family history d. Thorough HPI, ROS, & PE
Answer: D Rationale for why the answer is correct: Headaches can be differentiated and diagnosed by obtaining a thorough history, ROS, and PE. Imaging/labs are not usually useful in diagnosis unless an emergency or a red flag is found like a patient who is having the “worst headache of their life”. Site for question/rationale: Headache Differentials video by Dr. Moore and Dr. Hillard, Module 3
149
1. Question: What does the eFAST exam not show? a. Free fluid in the peritoneal cavity b. Pneumothorax c. Excessive free fluid in the pericardial sac d. Edema
Answer: D. Edema Rationale for why the answer is correct: The eFast exam shows free fluid in the pericardial cavity, peritoneal cavity, chest cavity and if there is a pneumothorax. Site for question/rationale: Ultrasound presentation by Steve Branham. Time 19:20
150
2. Question: Which view would you us during an eFast exam to visualize the pericardial space? a. Left thoracic b. Left longitudinal c. Subxiphoid transverse d. Transverse and longitudinal
Answer: C Rationale for why the answer is correct: The Subxiphoid transverse view assess for pericardial effusion and left lobe liver injuries. Site for question/rationale: Ultrasound presentation by Steve Branham. Time 20:30
151
3. Question: Which of the following would you see for a seborrheic lesion? a. Dull red, smooth skin, pustules, no borders b. Pale color, no scaling, no pustules, irregular boarders c. Red edematous, scaling, smooth skin d. Pale skin, no pustules, scarring, irregular boarders
Answer: C. Red edematous, scaling, smooth skin Rationale for why the answer is correct: Seborrhea is characterized by lesions with red edematous color, scaling, smooth skin, no pustules, no scaring and no boarders of the lesion. Site for question/rationale: Differential Diagnosis for the advanced practice nurse by Jacqueline Rhodes and Marilee Murphy Jensen. Pg 259
152
4. Question: Which diagnostic test would you order to assess if the patient has an allergic reaction to common allergens? a. Scraping b. Cultures c. Patch test d. Biopsy
Answer: C. Patch test Rationale for why the answer is correct: Patch tests expose the patient to common allergens to look for positive reactions. Site for question/rationale: Differential Diagnosis for the advanced practice nurse by Jacqueline Rhodes and Marilee Murphy Jensen. Pg 260
153
5. Question: Which is not a factor that contributes to the development of Merkel Cell Carcinoma? a. Immunosuppression b. At least one blistering sunburn under the age 30 c. Merkel Cell polyomavirus d. UV radiation exposure
Answer: B. At least one blistering sunburn under the age 30. Rationale for why the answer is correct: One blistering sunburn under the age 30 is a risk factor of Basal Cell Carcinoma. All the others are factors that can contribute to the development of Merkel Cell Carcinoma. Site for question/rationale: Dermatology lecture by Dr. Mercedes Day. Time 7:39
154
6. Question: Which of the following is not a characteristic of Schizophrenia? a. Ritualistic behavior b. Lack of emotional expression and/or lack of purposeful activity c. Hallucinations d. Delusions
Answer: A. Ritualistic behavior Rationale for why the answer is correct: Ritualistic behavior is descriptive of obsesibe compulsive disorder, not schizophrenia. Site for question/rationale: Mental Health lecture by Dr. Mercedes Day. Time 15:38
155
7. Question: Generalized Anxiety Disorder is described as a. Engaging in avoidance and experiencing flashbacks after experiencing a traumatic event b. Excessive anxiety or worry for at least 6 months that causes problems in social interactions c. Lack of energy and loss of interest for at least 2 weeks affecting functioning in daily life d. Significant mood swings consisting of mania and depression
Answer: B. Excessive anxiety or worry for at least 6 months that causes problems in social interactions. Rationale for why the answer is correct: Excessive anxiety or worry for 6 months that causes problems in social interaction, school or work is the criteria for generalized anxiety disorder. Site for question/rationale: Mental Health lecture by Dr. Mercedes Day. Time 1:56
156
8. Question: A patient with a fever for the past 4 days, purulent nasal discharge, post nasal drainage and unilateral sinus pain. Which is the leading differential? a. Acute Viral Rhinosinusitis b. Acute Bacterial Rhinosinusitis c. Allergic Rhinitis d. Non Allergic Rhinitis
Answer: B. Acute Bacterial Rhinosinusitis Rationale for why the answer is correct: A fever of greater than 48 hours indicates bacterial infection. Site for question/rationale: HEENT presentation by Dr. Amy Moore and Dr. Grace Sun. Time 25:26
157
9. Question: A 16 year old high school student presents with painful swallowing , fever, chills, fatigue and malaise. Physical exam is significant for posterior lymphadenopathy. Which is the leading diagnosis? a. Strep Throat b. Mononucleosis c. Viral pharyngitis d. Postnasal drip
Answer: B. Mononucleosis Rationale for why the answer is correct: Sore throat, posterior lymphadenopathy, fever, fatigue and malaise are all characteristics of mononucleosis. Site for question
158
10. Question: A 35 year old male presents with ear pain. He denies fever, chills, sore throat, nasal congestion or discharge. Which of the following diagnoses would not be added to the differential? a. Bacterial infection b. Referred c. Trauma d. Neuropathic
Answer: A. Bacterial infection Rationale for why the answer is correct: Bacterial infection of the ear may have symptoms of abrupt onset, fever/chills, sore throat, nasal congestion cough and fatigue. Site for question/rationale: HEENT presentation by Dr. Amy Moore and Dr. Grace Sun. Time 9:22
159
1. In looking at a PA x ray view of the heart, what is the most important question to consider when assessing heart size? a) Does the patient smoke? b) Does the patient have hypertension? c) How old is the patient? d) How big are the lungs?
Answer: c. How old is the patient? Rationale for why the answer is correct: Pediatric heart size is approximately ½ width of the chest. With age/ growth the adult heart would be abnormally large if it took up same percentage or more than half of the chest (cardiomegaly). Site for question/rationale: Dr. Stritto’s Understanding Radiology lecture, Slide 35.
160
2. An eFAST ultrasound is primarily used to assess ________________ ? a) Fluids in the chest and abdomen b) Bone fractures c) Carotid pulses d) Gestational age
Answer: a. Fluids in the chest and abdomen Rationale for why the answer is correct: eFast (extended focused abdominal scan for trauma) is utilized to assess fluids in pericardial, peritoneal, chest cavity and rule out pneumothorax after a trauma. Site for question/rationale: Dr. Branham’s Principles of Ultrasound for the ACNP Student lecture, slide 26.
161
3. Typical Pneumonia presents on a PA/LAT chest x ray in which lobes? a) Upper b) Middle c) Lower d) All lobe involvement
Answer: c. lower Rationale for why the answer is correct: Upper lobe pneumonias often caused by strep, middle lobe pneumonia is usually aspiration and typical pneumonia is usually in lower lobes. Site for question/rationale: Dr. Stritto’s Understanding Radiology lecture, Slide 88
162
4. Painful, yellow, fluid-filled blisters on only one side of the body almost creating a stripe would be indicative of what skin infection? a) Dermatitis b) Psoriasis c) Impetigo d) Herpes Zoster
Answer: d. Herpes Zoster. Rationale for why the answer is correct: Herpes Zoster is a painful skin condition leading to fluid filled blisters usually only effecting one side of the body in a stripe like pattern. Site for question/rationale: Dr. Sun & Dr. Moore’s Skin Disorders Video, time stamp 9:02/ 13:39
163
5. 40-year female presents to the clinic with complaints of small non-draining abscess to the sole of her foot that is not improving. The room is set-up for the incision and drainage. What would the APRN tell this patient? a) That the area will need to be injected with lidocaine before I&D is performed. b) That they will set up a referral to a surgeon because of the location of the abscess. c) That they will need to be injected with bupivacaine before I&D is performed. d) APRNs cannot perform I&D.
Answer: b. That they will set up a referral to a surgeon because of the location of the abscess. Rationale for why the answer is correct: Any abscess on the face, palms of the hands, elbows or soles of feet should be referred to a surgeon for I&D. Site for question/rationale: Dr. Moore’s The Basics of Local Anesthetics lecture, time stamp 6:26/11:26
164
6. 45-year-old female presents to the clinic with chief complaint of weight gain. In completing her history, the provider notes her youngest son has recently left for college and her husband encouraged her to see someone because she is not acting like her usual self for the last month. The patient reports always “feeling tired” and irritable. She does not enjoy playing tennis anymore and avoids friends. She also reports insatiable hunger. What would your list of differential diagnoses include? a) PTSD b) PPD c) Depression d) OCD
Answer: c. depression. Rationale for why the answer is correct: Depression includes symptoms of persistent loneliness/ sadness, lack of energy, feelings of hopelessness, sleep and eating changes, loss of interest in things you used to enjoy, difficulty concentrating, and thoughts of death or suicide. Can be to varying degrees and occurs most days over at least two weeks. PTSD occurs after a traumatic event and includes symptoms of reliving the event. PPD occurs during pregnancy or the 6-mo. following. OCD is related to uncontrollable reoccurring thoughts and actions. Site for question/rationale: Dr. Day’s Common Mental Health Disorders lecture, 8:07/17:19
165
7. To evaluate a 22-year-old male’s drinking habits, what questionnaire would you complete during your assessment? a) PHQ-9 b) CAGE c) ASQ d) All the above
Answer: b. CAGE assessment Rationale for why the answer is correct: CAGE is a 4-question tool used to evaluate potential of alcohol abuse. PHQ-9 is for depression. ASQ evaluates suicide risk. Site for question/rationale: (https://psychology-tools.com/cage-alcohol-questionnaire/ , 2021); required Mental Health module resources.
166
8. 12-year-old patient presents to clinic with his mother. He reports fever 100.8 F and right sided sinus pain/ white drainage for greater than a week. He has been taking daily Allegra po and Tylenol po for fever, but no significant improvement noted. What is your leading differential diagnosis? a) Viral Rhinosinusitis b) Bacterial Rhinosinusitis c) Allergic Sinusitis d) Non- allergic Sinusitis
Answer: b. Bacterial Rhinosinusitis Rationale for why the answer is correct: Bacterial rhinosinusitis usually includes unilateral purulent drainage and sinus pain lasting greater than or equal to 10 days with fever. Bacterial usually presents with bilateral sinus pain and drainage with fever that dissipates after 48 hours. Drainage for allergic and non-allergic sinusitis is usually clear. Site for question/rationale: Dr. Sun and Dr. Moore’s Ear, Nose and Throat lecture, time stamp explanation between 16:51-20:36/ 27:00
167
9. 55-year-old female patient presents to the clinic for check-up and refills of her Topamax for chronic migraines. In her history, she mentions this week she has been vomiting early in the mornings, having severe pain she has never experienced before on one side of her head and trouble finding words. What would your next action as the provider be? a) Increase her dose of Topamax. b) Switch to a different migraine medication. c) Send her for head CT immediately. d) Send her for MRI immediately.
Answer: c. Send her for head CT immediately. Rationale for why the answer is correct: With any acute change in signs and symptoms it is important to rule out brain bleeding or clot. Particularly in instances of new early morning vomiting with headaches, different/ worse new pain, having trouble thinking or those patients greater than 55 years old. Site for question/rationale: Dr. Moore and Dr. Hilliard’s Headache Lecture, time stamp 7:06/ 8:52
168
10. Fluorescein dye is insisted in the eye to better visualize: a) Macular degeneration b) Corneal abrasion/ damage c) Glaucoma d) Cataracts
Answer: b. corneal abrasion/ damage Rationale for why the answer is correct: Fluorescein dye is applied to the eye to highlight corneal defects. When blue light is applied to eye after instillation of dye, abrasions to cornea or other defects are easily visible. It cannot diagnose the other listed conditions. Site for question/rationale: Instilling Fluorescein dye video, required HEENT resources, time stamp 0:36/ 0:46
169
1. Question: An eFAST is used for all EXCEPT? A. Diagnosing thyroid disease B. Checking for blood in the chest and abdomen C. Trauma patients D. Unstable patients when needing to decide about chest tube placement without the delay of waiting for CT
Answer: A. Diagnosing thyroid disease Rationale for why the answer is correct: An eFAST is an acronym for extended Focused Abdominal Scan for Trauma. It can be used at the bedside on trauma patients and can check for blood in the chest and abdomen. An eFAST can also be used on unstable patients who need definitive care like chest tube placement without delay in waiting for CT. Thyroid disease is NOT diagnosed by performing an eFAST. Site for question/rationale: Principles of Ultrasound for the ACNP Student Presentation by Dr. Steve Branham; Module 3; slide 25; timestamp 18:07-18:50.
170
``` 2. Question: With what ultrasound probe are you able to assess the gallbladder, liver, and bladder? A. Liner probe B. Intracavitary probe C. Phased array probe D. Curve linear probe ```
Answer: D. Curve linear probe Rationale for why the answer is correct: The curve linear probe has a maximum depth of 30 centimeters. It’s suitable for assessing the gallbladder, liver, bladder, abdominal aorta, abdominal free fluid, uterus, and ovaries. Site for question/rationale: Principles of Ultrasound for the ACNP Student Presentation by Dr. Steve Branham; Module 3; timestamp at 12:18-12:35.
171
``` 3. Question: A 6-year-old female presents with a complaint of yellow vesicles around her mouth and nose that leaks pus. Based on the description, what skin condition can this patient have? A. Nevi B. Actinic keratosis C. Basal cell D. Impetigo ```
Answer: D. Impetigo Rationale for why the answer is correct: Basal cell carcinoma presents as shiny and pearly growth. A nevus is smooth, round moles that are one color. Actinic keratosis is a thick, scaly, or crusty patch on the skin that occurs mostly in older adults. Impetigo is a very itchy, yellow pus-filled vesicle that is around the mouth and nose. Site for question/rationale: Skin Lesion presentation by Dr. Amy Moore and Dr. Grace Sun; timestamp at 02:55-3:05, 4:35-9:10.
172
``` 4. Question: When doing a comprehensive exam on a patient who comes in with a skin lesion or a rash, what is one area as a provider you need to make sure to assess? A. Up both nostrils B. Patient’s back C. Pharynx D. Pupils ```
Answer: B. Patient’s back Rationale for why the answer is correct: When a patient comes in with a skin condition, makes sure to not just look at the rash or lesion, but look at the rest of the body. It is crucial to assess the patient's back because patients cannot look at their backs. As a provider, if you do not look at their backs, you have missed a lot of the patient. Site for question/rationale: Skin Lesion presentation by Dr. Amy Moore and Dr. Grace Sun; timestamp at 12:10-12:15.
173
5. Question: When stapling a wound, what is the proper way to staple a wound? A. Use pressure with wound edges inverted. B. Do not use pressure, and wound edges are properly inverted. C. Do not use pressure, and wound edges must be properly everted. D. Use pressure and evert wound edges.
Answer: C. Do not use pressure, and wound edges must be properly everted. Rationale for why the answer is correct: Wound edges must be everted appropriately using forceps for proper healing and best cosmetic results. Excessive pressure must be avoided, which may place the staple too deeply and can cause local ischemia. Site for question/rationale: Stapling Devices video; Module 3; timestamp 1:22-1:33, 2:02.
174
6. Question: Which of the following symptoms must be present within the last 12 months for a person to be diagnosed with a disorder due to a substance? A. Craving and hallucination B. Depersonalization and derealization C. Building up tolerance and failure to fulfill other obligations D. All of the above
Answer: C. Building up tolerance and failure to fulfill other obligations Rationale for why the answer is correct: Hallucinations, depersonalization and derealization are not symptoms that help diagnose substance disorder. To be diagnosed with a disorder due to a substance, two of the following symptoms must be present within the last 12 months: consuming more than planned, worrying or failed efforts to control use, time using or obtaining drugs, failure to fulfill other obligations, craving, continued use, repeated use in dangerous situations, not doing activities they usually would enjoy, building up a tolerance, and withdrawal symptoms. Site for question/rationale: Common Mental Health Disorder presentation by Dr. Mercedes Day at timestamp 0:50.
175
``` 7. Question: Blain is a 6-year-old male who his mother accompanies with complaints of doing poorly in school. The mother states the teacher has complained of him getting out of his seat when required to be sitting, blurting out answers without raising his hand, being unable to stay quiet, interrupting others in the class, and not paying attention in lectures. What disorder may Blain have? A. Depression B. ADHD C. Substance use disorder D. Conduct disorder ```
Answer: B. ADHD Rationale for why the answer is correct: ADHD is a brain disorder marked by an ongoing pattern of inattention and hyperactivity-impulsivity that interferes with functioning or development. Getting out of a seat when required to sit, blurting out answers when it is not your turn, unable to stay quiet, and inattention are all signs of ADHD. Site for question/rationale: Common Mental Health Disorder presentation by Dr. Mercedes Day at 4:00-4:22 time.
176
``` 8. Question: A 28-year-old male presents with a complaint of pain in his head. He rates the pain 6/10 on the pain scale and states, “It feels like I have a tight band around my head.” What type of headache is the patient having? A. Primary headache; Migraine B. Secondary headache; Tension headache C. Primary headache; Tension headache D. Secondary headache; Cluster headache ```
``` Answer: C. Primary headache; Tension headache Rationale for why the answer is correct: Tension headache is a primary headache that is often mild to moderate in severity and is described as feeling like a tight band around the head. A tension headache is the most common type of headache; however, the etiology is unknown. Site for question/rationale: Headaches document in module 4; Page 1. Also mentioned in Headache Differential presentation by Dr. Amy Moore and Dr. Tara Hillard at 0:13-0:35 time. ```
177
9. Question: A 68-year-old female presents to the clinic with complaints of a severe headache. The patient states the pain began suddenly and "is the worst headache she has ever had." What should you do? A. Tell her to take Tylenol every four hours for the headache. B. Send her to the ER because she needs a CT scan. C. Send her home D. Inform her this is a cluster headache, and it alleviates with a glass of wine.
Answer: B. Send her to the ER because she needs a CT scan. Rationale for why the answer is correct: The patient has an acute headache. When the patient states their headaches are the worst headaches they have ever had, the patient must be directed to the ER for further evaluation. The patient may need a CT scan of the brain to rule out a blood clot or brain bleed. Site for question/rationale: Headache Differential presentation by Dr. Amy Moore and Dr. Tara Hillard at 4:20-4:45.
178
``` 10. Question: Non-allergic rhinitis can be caused by? A. Cigarette smoke B. Strong odors C. Alcoholic beverages D. All of the above ```
Answer: D. All of the above Rationale for why the answer is correct: Non-allergic rhinitis may not depend on IgE's presence and is not due to an allergic reaction. It can be caused by cigarette smoke, strong odors, alcoholic beverages, and cold temperatures. Site for question/rationale: Nose document in Module 4; Page 1.
179
``` Radiology 1. Question: What x-ray view do you use if the patient is unable to stand? A. AP B. PA C. Lateral D. PA and Lateral ```
Answer: AP Rationale for why the answer is correct: AP view is used if patient is unable to stand Site for question/rationale: Basic Radiology PowerPoint page 6 slide 18
180
2. Question: When reading x-rays it is important for the clinician to take these steps first A. Check the name on the film B. Check the date of the film C. Check for previous studies for comparison D. All of the above
Answer: all of the above Rationale for why the answer is correct: It is prudent to take these steps before reading an x-ray to ensure you have the correct and most current information on the patient. Site for question/rationale: Basic Radiology PowerPoint page 8 slide 22
181
3. Question: High risk patients for developing melanoma include: A. Chronic outdoor activities without adequate photoprotection B. Family history of melanoma C. One or more atypical nevi D. All of the above
Answer: D. All of the above Rationale for why the answer is correct: High risk patients for developing melanoma include: white males greater than 50 years of age, family history of melanoma, total nevi count above 25, one or more atypical nevi, significant childhood, adolescent or young adult sun exposure, which includes at least one blistering or painful sunburn when under 30 years of age, chronic outdoor activities without adequate photoprotection, indoor tanning beds, immunocompromising conditions and red hair phenotype. Site for question/rationale: Dermatology lecture at 1 minute 58 seconds
182
4. Question: When diagnosing a skin lesions what does ABCDE stand for? A. asymmetry, border, color, depth, evolving B. asymmetry, border, cancer, diameter, evolving C. asymmetry, border, color, diameter, evolving D. asymmetry, border, color, diameter, excoriation
Answer: C. asymmetry, border, color, diameter, evolving Rationale for why the answer is correct: The American Academy of Dermatology use the first 5 letters of the alphabet to guide in diagnosing and classifying melanomas. Site for question/rationale: Skin Lesion Presentation at 2 minute 38 seconds
183
``` 5. Question: Which of the following is NOT a contraindication for stapling a wound? A. Deep wounds B. Inadequate hemostasis C. Plan for CT scanning D. The wound has straight, clean edges ```
Answer: D. The wound has straight, clean edges Rationale for why the answer is correct: Contraindications for stappling a wound include: deep wounds, inadequate hemostasis, plan for CT scanning, MRI expected or lacerations on face, hands, feet. Site for question/rationale: Clinical key stapling wound instructions
184
``` 6. Question: What questionnaire can indicate potential problems with alcohol abuse? A. CAGE scale B. PHQ-9 C. DIG FAST D. ECAQ ```
Answer: A. CAGE scale Rationale for why the answer is correct: The CAGE questionnaire is a series of four questions that clinicians use to check for signs of possible alcohol dependency. Site for question/rationale: Module 2 resources for assessing mental health
185
7. Question: Which of the following is true regarding OCD? A. OCD is an anxiety disorder characterized by obsessions and compulsions B. People with OCD can never have obsessions or compulsions alone they have to have both C. The obsessions help to alleviate anxiety D. The person with OCD doesn’t know that their thoughts or obsessions are senseless
Answer: A. OCD is an anxiety disorder characterized by obsessions and compulsions Rationale for why the answer is correct: OCD is an anxiety disorder characterized by obsessions and compulsions. People with OCD can have obsessions without compulsions. The compulsions are what help alleviate anxiety from the obsessions. The hallmark of OCD is that the person understands that their thoughts or behaviors are senseless or excessive. Site for question/rationale: Common Mental Health Disorders presentations 9 minutes and 49 seconds
186
``` 8. Question: Possible causes of a secondary headache include A. concussion B. poor posture C. stress D. skipped meals ```
Answer: A. concussion Rationale for why the answer is correct: A secondary headache is a symptom of a disease that can activate the pain-sensitive nerves of the head. Poor posture, stress, and skipped meals are examples of common primary headaches. Site for question/rationale: Module 4 HEENT Headaches handout
187
``` 9. Question: ____________ are small specks or clouds that move across your field of vision. A. Glaucoma B. Eye Floaters C. Uveitis D. Cataracts ```
Answer: B. Eye Floaters Rationale for why the answer is correct: Eye floaters are small specks or clouds that move across your field of vision. Glaucoma is when a build-up of fluid in the eye creates pressure, damaging the optic nerve. Uveitis is the inflammation of one or more parts of the eye that make up the uvea. Cataracts are a degenerative form of eye disease in which the lens gradually becomes opaque and vision ‘mists’ over. Site for question/rationale: Module 4 HEENT Eyes handout
188
``` 10. Question: Seasonal allergic rhinitis (hay fever) may occur in various times of the year. It is caused by a hypersensitivity to A. Pollen B. Dust C. Ragweed D. Tumbleweeds ```
Answer: C. Ragweed Rationale for why the answer is correct: Seasonal allergic rhinitis (hay fever) may occur in various times of the year. It is caused by a hypersensitivity to ragweed. Site for question/rationale: Module 4 HEENT Nose handout
189
``` 1. Question: Which chest x-ray views are the best technique for visualizing and examining the hearts size? Answer: A. AP and PA B. PA and Lateral C. Lateral Decubitus and Oblique D. Oblique and AP ```
Rationale for why the answer is correct: • The AP chest x-ray view shoots anteriorly to posterior. This view along with the PA view is best for determining the heart’s size and its silhouette. Site for question/rationale: • Found in the Understanding Radiology power point slides by Dr. Stritto on page 7.
190
``` 2. Question: A 22- year- old male presents to the ER with a distended belly after falling off a roof and landing flat on his abdomen. The patient is complaining of pain and tenderness over his entire abdomen. He quickly becomes obtunded and his blood pressure begin to drop. What specialized radiographical procedure can be done for a trauma patient? Answer: A. KUB X-Ray B. Chest Including Abdomen X-Ray C. eFAST D. RUQ Ultrasound ```
Rationale for why the answer is correct: • A fast exam is an easy way to check for blood in the chest and abdomen. It stands for extended Focused Abdominal Scan for Trauma. Site for question/rationale: • Found in the Ultrasound Handouts by Dr. Branham under Module 3 on slide 25.
191
3. Question: A 20- month old male presents to the ED with rash to the nose and mouth. The patient has numerous red sores that have ruptured and leaked “white stuff” and now has yellowish-brown crusts covering them. The APRN diagnoses him with impetigo and starts him on topical antibiotics. Which statement made by the mother indicates an understanding of the infection? Answer: A. “This rash can spread but doesn’t normally” B. “This diagnosis is a lifelong diagnosis that may flair up occasionally” C. “This is extremely contagious and should refrain from daycare until it is gone” D. “He has a virus that he acquired from daycare”
Rationale for why the answer is correct: • Impetigo is found mostly surrounding the face, nose, and mouth. It is found to have red lesions that eventually leak and ooze yellow pus and crust over. This is a bacterial infection caused by either Group A Strep or Staph aureus and is treated with topical antibiotics and is extremely contagious to the touch. Site for question/rationale: • Found under Module 3 in the Skin Lesions/Disorders video completed by Dr. Moore and Dr. Sun starting at minute 4:11.
192
4. Question: When diagnosing Melanoma compared to other skin cancers you would see which characteristics related to the ABCDE assessment? Answer: A. Symmetrical, regular boarders, pink in color, and a diameter > 6 mm B. Asymmetrical, irregular boarders, color variegation, and diameter >6 mm C. Asymmetrical, regular boarders, color variegation, and a diameter <6 mm D. Symmetrical, irregular boarders, pink in color, and a diameter <6 mm
Rationale for why the answer is correct: • Melanoma is the most serious form of skin cancer that can occur anywhere on the body. Normal skin lesions should be uniform in color, with distinct borders, (round or oval, and usually smaller than 6 mm. A mole that is unusual and could indicate melanoma will include asymmetry, irregular borders, changes in color, diameter is greater than 6mm, that is also evolving over time. Site for question/rationale: • Found in the Dermatology video presentation completed by Dr. Mercedes Day under the Module 3 around 0:28 seconds.
193
``` 5. Question: A 10-week-old baby presents with fever, irritability, cutaneous tenderness, and diffuse erythema. What dermatological condition are you most worried about? Answer: A. Staphylococcal Scalded Skin Syndrome B. Pityriasis Rosea C. Eczema Herpeticum D. Scarlatina ```
Rationale for why the answer is correct: • Staph Scalded Skin develops with the same symptoms as described above. This diagnosis is caused from a bacterial pathogen and is considered a life-threatening infection and requires immediate treatment thus it is vital to diagnosis early and treat accordingly. Site for question/rationale: • Found in the Dermatology video presentation completed by Dr. Mercedes Day under Module 3 around minute 37:14.
194
``` 6. Question: A 30-year-old patient presents to your clinic with new onset of fatigue, difficulty sleeping, sadness, and she recently stopped going to her weekly book club with her friends. Which mental health screening questionnaire will help you determine her diagnosis and the cause of her symptoms? Answer: A. CAGE B. MoCA C. PHQ-9 D. MMSE ```
Rationale for why the answer is correct: • This patient is showing symptoms of depression and the appropriate screening assessment tool is the Patient Health Questionnaire (PHQ-9) for a patient this age. The other assessments are not used for screening depression, they are useful for substance abuse, dementia, and delirium respectively. Site for question/rationale: • Found in the Common Mental Health Disorders video completed by Dr. Mercedes Day in the Module 4 Mental Health section around minute 7:47. The mental health screenings were found under this tab as well.
195
``` 7. Question: A 7-year-old boy was brought in by his mother after having difficulty at home and in school with his behavior. Per mom the patient has been arguing with adults, refusing to complete daily tasks, and purposefully doing things to annoy and upset his older sister. The APRN should consider which as their lead diagnosis: Answer: A. Oppositional Defiance Disorder B. Bipolar Disorder C. ADHD D. Conduct Disorder ```
Rationale for why the answer is correct: • Oppositional Defiance Disorder is characterized by frequent temper tantrums, arguing with adults, refusing to complete tasks, questioning rules, doing things purposefully to annoy or upset others, blaming others for their actions, angry attitude, and or seeking revenge. Some of the symptoms of ODD can be like each of the other listed distractors but there are some differences in the diagnosing symptoms. Site for question/rationale: • Found in the Common Mental Health Disorders Video by Dr. Mercedes Day in the Module 4 Mental Health module around minute 10:39.
196
8. Question: What symptoms will be associated with a 24-year-old patient presenting with a migraine with aura? Answer: A. Throbbing behind the eye, nausea, and vomiting B. Flashes of light, vision changes, and tingling sensations in hands and feet C. Photophobia, photophonia, nausea D. Intense pain around the eye, flashes of light, and photophobia
Rationale for why the answer is correct: • Migraines with aura are considered headaches that present with sensory disturbances known as an aura. These disturbances can include flashes of light, blind spots, other vision changes, and or tingling in hands or feet. Site for question/rationale: • Found in the Headaches document page 1 posted by faculty under the Module 4 HEENT sections.
197
``` 9. Question: A patient presents with a lens that has gradually become opaque in character the patient is complaining of “misty vision”. Your leading diagnoses should include: Answer: A. Age-Related Macular Degeneration B. Eye Floaters C. Glaucoma D. Cataracts ```
Rationale for why the answer is correct: • Cataracts is a degenerative eye disease characterized by a gradual lens opacification that patients can describe as their vision “mists” over. Site for question/rationale: • Found in the Eyes document page 1 posted by faculty under the Module 4 HEENT section.
198
10. Question: A 10-year-old male patient arrives to your clinic with sneezing, rhinorrhea, and an itchy nose after encountering cold temperatures yesterday during recess. You diagnose him with non-allergic rhinitis. Which statement by the patient’s mom indicates understanding of this diagnosis? Answer: A. “This diagnosis needs to be treated with antivirals” B. “This diagnosis is not related to an allergic reaction but can be triggered by pollutants” C. “He will have to carry an epi pen wherever he goes” D. “This diagnosis can be caused by bacterial pathogens”
Rationale for why the answer is correct: • Non-allergic rhinitis does not always rely on the presence of IgE and is not due to an allergic reaction. It can be caused by numerous different factors but normally presents with inflammation of nasal mucosa, sneezing, rhinorrhea, itchy nose, and congestion. Site for question/rationale: • Found in the Nose document page 1 posted by faculty under the Module 4 HEENT section.
199
1. Question: When assessing heart size via a chest x-ray, which view can be used to estimate the heart size? a. PA view b. AP view c. Lateral view d. Oblique view
Answer: A Rationale for why the answer is correct: In PA view, the heart is closer to the film and, therefore, the size is not distorted as much. In AP view, the heart is further away, and the size is distorted which makes the image unreliable to estimate heart size. Lateral view can be used to assess which chamber is enlarged. Site for question/rationale: Radiology Review Part 1, slide 19, 20 and 36
200
2. Question: All of the following are findings on an x-ray that are consistent with COPD EXCEPT: a. A flattened diaphragm b. Kerley B lines c. Increased AP diameter of chest d. 10 or more ribs
Answer: B. Rationale for why the answer is correct: A flattened diaphragm, increased AP diameter of the chest (barrel chest), and the ability to see/count 10 or more ribs are findings consistent with COPD; Kerley B lines are consistent with severe heart failure. Site for question/rationale: Radiology Review Part 2, slide 15.
201
3. Question: A concerned mother brings in her 9 month old boy because he has a widespread rash on his trunk, legs and arms. The little boy is afebrile, smiling and playful. The mother states that she had been alternating Motrin and Tylenol for fever for a few days (the highest temperature she got was 101.6 F), but now that the fever is gone, he has this rash. He has been eating/drinking well. Upon examination, you find a rose-pink blanchable, painless rash on the trunk and extremities. What do you do next? a. Order amoxicillin per weight BID for 10 days for the baby and have them follow up with you in 5 days. b. Prepare for the child to be taken to the ED immediately. c. Get a culture of the rash, prescribe a steroid cream and have them follow up in a week. d. Inform the mother that this rash should resolve within a few days on its own. Have her continue to make sure that the baby continues to eat and drink well and have them follow up with you in 2-3 days.
Answer: D Rationale for why the answer is correct: The HPI and presentation of the child and rash make it highly likely that this is Roseola. Site for question/rationale: Dr. Mercedes Day recording on dermatology; time stamp 28:00.
202
4. Question: If you are describing a rash/lesion that is not palpable (there are no elevation or depressions of the skin, it is “flush” with the skin), what you could call it? a. Plaques b. Pustules c. Macules d. Papules
Answer: C Rationale for why the answer is correct: The term macule is used to describe a non-palpable lesion/rash that varies in color from the surrounding skin. There are no elevations or depressions of the skin. Site for question/rationale: Differential Diagnosis for the Advanced Practice Nurse textbook p. 263.
203
5. Question: Which if the following is TRUE about an incision and drainage (I&D) procedure for an abscess? a. Pack the wound as tightly as possible with iodoform gauze b. Antibiotics are usually unnecessary if I&D is done properly, unless there is a comorbid disease such as diabetes c. Inject in the abscess with lidocaine to help prolong anesthetic benefits d. Antibiotics are always given after an I&D
Answer: B Rationale for why the answer is correct: Do not pack the wound too tight with iodoform; avoid infiltration of the abscess with lidocaine, antibiotics are rarely needed, unless there is a comorbid disease Site for question/rationale: Incision and drainage video from Clinical Key, time stamp-- :20. Overview of local anesthetics with punch biopsy (Dr. Amy Moore), time stamp 6:14.
204
6. Question: Mrs. Robertson, a 60 year old patient, that recently lost her husband, states that lately she has been having a hard time sleeping and nothing that she used to love doing draws her attention anymore. When and how long must these symptoms be present in order for Mrs. Robertson to be diagnosed with depression? a. Nearly every day for 2 weeks b. Nearly every day for 1 month c. Nearly every day for 2 months d. Nearly every day for 6 months
Answer: A Rationale for why the answer is correct: In order to diagnose a patient with depression, the patient must experience the symptoms of depression nearly every day for 2 weeks. Site for question/rationale: Mental Health Disorders video by Dr. Day, time stamp 7:58.
205
7. Question: Postpartum depression may begin up to how long after giving birth? a. 2 months b. 4 months c. 6 months d. 3 months
Answer: C Rationale for why the answer is correct: Postpartum depression usually starts within weeks after giving birth, but the symptoms can begin up to 6 months after birth. Site for question/rationale: Mental Health Disorders video by Dr. Day, time stamp 12:05.
206
8. Question: All of the following are headache red flags EXCEPT? a. Photosensitivity b. New onset of headache for a 55 year old with no history of headache c. Patient stating “this is the worse headache I’ve ever had” d. Patient is slurring words
Answer: A Rationale for why the answer is correct: Photosensitivity is usually associated with migraine headaches and is not necessarily a red flag. Site for question/rationale: Dr. Moore and Dr. Hilliard video on headache differentials, time stamp: 1:46, 7:02
207
9. Question: A mother brings her 18 month old daughter in because she has been fussy, running fever and pulling at her right ear. She states that she knew to bring her in because this is the 4th time that this happens this year and all of the other times, her daughter had an ear infection. You look in her ears and see a bulging, erythematous tympanic membrane. You diagnose her with otitis media. As the NP, what is your plan for this patient? a. Prescribe antibiotics and have them follow up in 3 days. b. Prescribe antibiotics, educate mom on alternating Tylenol and Motrin for fever and pain and refer her to an ENT for evaluation for placement of tympanostomy tubes. c. Refer her to an ENT for evaluation for placement of tympanostomy tubes. d. Inform the mom that she can alternate Tylenol and Motrin for fever and pain
Answer: B Rationale for why the answer is correct: The patient is in pain and you have diagnosed her with otitis media. It would be prudent to treat the otitis media in addition to referring her to an ENT due to the 4th occurrence of otitis media in less than a year. Site for question/rationale: Clinical Key: ENT Referral for tympanostomy tubes in children—Guidelines: Myringotomy and tympanostomy tube placement for recurrent acute otitis media
208
10. Question: A 23 year old female comes in because she woke up with her eyes “crusted shut.” She is congested and reveals to you that she has been having cold like symptoms for a few days, including a low grade fever. What is your most likely diagnosis? a. Bacterial conjunctivitis b. Allergies c. Corneal abrasion d. Viral conjunctivitis
Answer: D Rationale for why the answer is correct: Generally, viral conjunctivitis is usually preceded by cold symptoms. Site for question/rationale: HEENT presentation by Dr. Sun and Dr. Moore, time stamp: 1:26
209
1. Point of care ultrasounds (POCUS): select all that apply a. Allow for rapid screening b. Replace a diagnostic ultrasound c. Heighten diagnostic accuracy d. Minimize exposure to ionizing radiation
Answer: A, C, and D. Rationale for why the answer is correct: POCUS allows for rapid screening of certain issues like aneurysms, heightens diagnostic accuracy, and minimizes ionizing radiation by being able to visualize an area of the body without the use of x-rays or CT scans. It does not, however, replace a diagnostic ultrasound. Site for question/rationale: Dr. Branham’s lecture “Principles of Ultrasound for the ACNP Student,” slides 5-6.
210
2. Which x-ray view is best to determine the size of the heart? a. Posteroanterior (PA) b. Anteroposterior (AP) c. Lateral (Lat) d. Oblique
Answer: A posteroanterior (PA) view Rationale for why the answer is correct: The PA view provides the best view to determine the size of the heart because, in the PA view, the heart is closer to the film. Since the heart is further from the film in the AP view, it can appear distorted and larger than it is. Site for question/rationale: Dr. Rita Dello Stritto’s “Understanding Radiology” lecture, slide 21.
211
3. With what infectious agent is Kaposi Sarcoma most commonly associated? a. Bordetella pertussis b. Mycobacterium tuberculosis c. Herpes virus d. Varicella-zoster virus
Answer: C Herpes virus Rationale for why the answer is correct: Kaposi sarcoma requires infection with human Herpes virus 8 and is also known as Kaposi sarcoma-associated Herpes virus (KSHV). Site for question/rationale: Dr. Mercedes Day’s “Dermatology” lecture, minute 10:31.
212
4. What clinical characteristics are indicative of melanoma? Select all that apply: a. Diameter <6mm b. Asymmetry c. Color variegation d. Distinct/regular border
Answer: B and C Rationale for why the answer is correct: The ABCDE’s of melanoma include asymmetry, border irregularities, color variegation, diameter >6mm, and evolution. Therefore, answer choices A and D would be excluded. Site for question rationale: Dr. Mercedes Day’s “Dermatology” lecture, minutes 2:28 and 24:16.
213
5. A 50-year-old female patient comes to see you with a chief complaint of a shiny, pearly lesion to their nose. Which condition should rank high on your differential diagnosis based on the clinical features? a. Squamous cell carcinoma b. Basal cell carcinoma c. Merkel cell carcinoma d. Melanoma
Answer: B Basal cell carcinoma Rationale for why the answer is correct: The shiny, pearly characteristics are a classic presentation for basal cell carcinoma. Site for question/rationale: Dr. Amy Moore and Dr. Grace Sun’s “Skin Lesion” presentation, minute 3:05.
214
6. A distinguishing characteristic between Bipolar I versus Bipolar II disorder is: a. The timing of the manic episode b. The presence of a manic episode c. The timing of a depressive episode d. The presence of a depressive episode
Answer: A The timing of the manic episode Rationale for why the answer is correct: In both Bipolar I and Bipolar II disorders, mania is observed. However, a manic episode (Bipolar I) lasts for at least a week and is a distinct change to a person’s usual behavior, whereas a hypomanic episode (Bipolar II) lasts about 4 days. Site for question/rationale: Dr. Mercedes Day’s “Common Mental Health Disorders” lecture, minute 5:29.
215
7. If you want to assess for potential problems with alcohol, what tool would you use. a. PHQ-9 b. PHQ-2 c. CAGE d. asQ
Answer: C CAGE screening tool. Rationale for why the answer is correct: CAGE is an acronym for Cut down, Annoyed, Guilty, and Eye-opener and is used as a tool to screen for alcohol abuse. Site for question/rationale: CAGE Alcohol Questionnaire handout.
216
8. You have a patient who presents with a fever that started 3 days ago, unilateral sinus pain, coryza, postnasal drainage, and purulent drainage from the right nostril. What diagnosis should rank high on your differentials? a. Acute viral sinusitis b. Acute bacterial sinusitis c. Allergic rhinitis d. Nonallergic rhinitis
Answer: B Acute bacterial sinusitis Rationale for why the answer is correct: Acute bacterial sinusitis typically appears with a fever lasting more than 48 hours, unilateral purulent drainage, postnasal drainage, and unilateral sinus pain. Site for question/rationale: Drs. Sun and Moore’s “Ear, Nose, and Throat” lecture, minute 23:24.
217
9. A 22-year-old female patient presents to the clinic with sudden onset of sore throat 9/10 on pain scale, fever of 101°F, fatigue, body aches, mild headache, and cough. She denies nasal congestion, vomiting, and rash. Her rapid strep test comes back negative. For what organisms may you consider running a throat culture? Select all that apply: a. Streptococcus b. Candida albicans c. Bordetella pertussis d. Naegleria fowleri
Answer: A, B, and C Rationale for why the answer is correct: Although her rapid strep test was negative, you cannot rule out a strep infection. Rapid strep tests have a specificity of about 95% to 98%, but a sensitivity of 75% to 85%. This means that you can be more confident that a positive result will in fact be positive, but a negative result does not necessarily rule out the infection. A positive throat culture will mean that a fungus or bacteria is growing in that culture, therefore, one would be able to detect Candida albicans, streptococcus, or Bordetella pertussis. Naegleria fowleri is an amoeba, and, thus will not grow in the culture. Site for question/rationale: Rhoads & Jensen, 2015.
218
10. What are some red flag headache symptoms that would warrant the patient to either be sent to the Emergency Department or for a CT scan? Select all that apply: a. Experiencing an aura with a unilateral headache b. New onset headaches with nausea in patients under the age of 55 years c. Thunderclap headaches d. History of headaches with new onset difficulties in concentration
Answer: C and D Rationale for why the answer is correct: Headaches that are described as a “thunderclap headache” or “the worst headache of my life” should warrant the provider to send the patient immediately to the ED. This could be a sign of a hemorrhage in the brain or a clot. If a patient already has a history of headaches like migraines or cluster headaches and complains of new onset difficulties in concentration or has a change in their headache symptoms, this warrants a CT scan to assess for hemorrhage. Site for question/rationale: Drs. Moore and Hilliard’s “Headaches” lecture, minute 7:35.
219
1. Question: When viewing a chest x-ray for lower lobe pneumonia, what characteristics would the APRN observe? A. Loss of cardiac border B. Loss of diaphragm border C. Loss of cardiac and diaphragm borders D. Loss of cervical spine at lateral view
Answer: B. Loss of diaphragm Rationale for why the answer is correct: The loss of a cardiac border would be indicative of upper or middle lobe pneumonia. With lower lobe pneumonia, the thoracic spine is lost in the lateral view, not the cervical spine. Site for question/rationale: Radiology presentation part 1 with Dr. Dello Stritto, slides 43 to 49.
220
``` 2. Question: When observing an abdominal x-ray, which would the APRN identify with a pneumoperitoneum? A. Sail sign B. Elevated diaphragm border C. Depressed diaphragm border D. Salter Harris II ```
Answer: B. Elevated diaphragm border. Rationale for why the answer is correct: a sale sign is an abnormal observation with bone structure. A Salter-Harris II is a bone defect above the metastasis. A depressed diaphragm border is not indicative of a pneumoperitoneum. Site for question/rationale: Radiology presentation part 2 with Dr. Dello Stritto, slides 30, 41, and 48.
221
3. Question: Which of the following is false regarding melanoma? A. Common in white males > 50 years old B. High risk of occurrence in significant childhood sun exposure C. Arises from deep tumors from the dermal layer D. Almost always cured through surgical excision.
Answer: C. Arises from deep tumors from the dermal layer Rationale for why the answer is correct: melanomas arises from superficial tumors confined to the epidermal layer. All other choices are true regarding melanoma. Site for question/rationale: Dermatology presentation with Mercedes Day, at 0:56 to 2:12.
222
``` 4. Question: Which of the following stages of Kaposi sarcoma is a solid tumor that is well-defined? A. Patch B. Plaque C. Pouch D. Nodular ```
Answer: D. Nodular Rationale for why the answer is correct: the patch stage describes a thin walled vascular space. The plaque stage describes increased density. Pouch is not a stage of Kaposi sarcoma. Site for question/rationale: Dermatology presentation with Mercedes Day, at 10:23 to 12:04.
223
``` 5. Question: Which of the following descriptors does not follow the presentation of scarlatina? A. Sand paper feeling B. Slapped cheek rash C. Fine, red rash D. Desquamation ```
Answer: B. Slapped cheek rash Rationale for why the answer is correct: Slapped cheek rash is a presentation for 5th disease from parvovirus. The remaining options following the typical presentation for scarlatina. Site for question/rationale: Dermatology presentation with Mercedes Day, at 26:54 to 27:56
224
6. Question: Which of the following would an APRN identify as not part of PTSD? A. Experiencing a traumatic event. B. Engaging in avoidance C. Significant decrease in arousal D. Experiencing a death, serious injury or sexual violence
Answer: C. Significant decrease in arousal Rationale for why the answer is correct: PTSD is characterized by experiencing a traumatic event, engaging in avoidance, increase in arousal symptoms and experiencing an event with death, serious injury or sexual violence. Site for question/rationale: Mental health presentation with Mercedes Day at 13:27 to 15:25
225
7. Question: Which of the following is a component of CAGE? A. Have you ever felt the need to drink excessively? B. Have you ever asked yourself why you drink alcohol? C. Have you ever used alcohol to increase sleep? D. Have you ever felt guilty about drinking?
Answer: D. Have you ever felt guilty about drinking? Rationale for why the answer is correct: option D is the only choice that is a component of CAGE screening for alcohol use. Site for question/rationale: The CAGE alcohol screening tool from psychology-tools.com, posted on the Module page.
226
``` 8. Question: Which of the following is not a diagnostic test or procedures the APRN would utilize for eye exams? A. Ophthalmoscope exam B. Fluorescein staining C. Retinal tomography D. Snellen eye chart ```
Answer: C. Retinal tomography Rationale for why the answer is correct: retinal tomography is a digital scan that gives detailed pictures of a retina compromised by macular degeneration or retinal detachment. All other options are commonly used by the APRN in assessing the eyes. Site for question/rationale: HEENT presentation with Moore and Sun at 1:45 to 3:10.
227
``` 9. Question: Which of the following is not a differential diagnosis for sore throat? A. Strep pharyngitis B. Gonorrhea C. Mononucleosis D. Conjunctivitis ```
Answer: D. Conjunctivitis Rationale for why the answer is correct: conjunctivitis is a condition of the eye, causing redness, swelling and itchiness. The remaining options are all appropriate differential diagnoses for a sore throat. Site for question/rationale: HEENT presentation with Moore and Sun at 13:06 to 15:54
228
``` 10. Question: J.W. is a 22 year old male, presenting today with “headache and my allergies are bad.” He states the headache started about 3 days ago when it work him up at night, along with excess lacrimation. J.W. states these headaches having been coming and going for 3-4 months. He denies any significant acute or chronic health problems. What of the following headache differentials seems most likely for this patient? A. Tension B. Migraine C. Cluster D. Temporal arteritis ```
Answer: C. Cluster Rationale for why the answer is correct: cluster headache characteristics include pain that will wake patient up in the middle of the night, lacrimation on the effecting side and is more common in males. Site for question/rationale: Headache presentation will Moore and Hilliard at 3:15 to 3:46
229
1. Which should the eFAST tool NOT be used for? a. To assess for solid organ injury b. To quickly assess an unstable patient c. To assess for fluid in the chest and abdomen d. To assess for a pneumothorax
Answer: A Rationale for why the answer is correct: The eFAST tool is used to quickly assess whether the patient has fluid in the chest or abdomen. It is not used to identify a solid organ injury (a CT will be needed for that). Site for question/rationale: Principles of US for the ACNP Student PP, slide 36
230
2. Which is TRUE about a pneumothorax? a. It will appear fluffy on XR b. The side of the pneumothorax will be black c. The side of the pneumothorax will be white d. It will have a barrel like appearance
Answer: B Rationale for why the answer is correct: The side of the pneumothorax will be black because it is full of air. Air is less dense, making it appear black on film. The other side will be white due to both lungs and heart being pushed over. The more dense the film is, the whiter it is. A fluffy or cotton candy like XR aligns with CHF while a barrel chest aligns with COPD. Site for question/rationale: Understanding Radiology PP, slide 72, 80, and 82
231
3. Which of the following is the patient likely experiencing based on this description? “Red scaly patches or plaques on knees and elbows that the patient describes as itchy” a. Eczema b. Impetigo c. Dermatitis d. Psoriasis
Answer: D Rationale for why the answer is correct: All of these are itchy, however only psoriasis has red, scaly patches on elbows, knees and sometimes palms and soles. Site for question/rationale: Skin Lesion Presentation, minute 8
232
4. A patient arrives to clinic with a small abscess on his thigh- which steps are appropriate? a. Applying a warm compress, draining it, and prescribing antibiotics b. Injecting lidocaine IN and around the lesion c. Applying a warm compress, incising it if it doesn’t drain naturally d. Applying a warm compress, draining it and packing it tightly for 48 hours
Answer: C Rationale for why the answer is correct: Antibiotics are usually unnecessary unless there are comorbidities like diabetes. Do not inject medicine into lesion and pack loosely for 48 hours Site for question/rationale: Overview of Local Anesthetics with Punch Biopsy Presentation, minute 5:30
233
5. Which would be a priority diagnosis to rule out first? As in, which is most concerning? a. Fifth disease b. Roseola c. Molluscum Contagiosum d. Pityriasis Rosea
Answer: A Rationale for why the answer is correct: Fifth disease can be benign or life threatening. All of the others are self-limiting. Site for question/rationale: Dermatology Presentation, slide 32-39
234
6. A patient answers YES to 2 of the 4 CAGE questions. What should the provider do next? a. Send to specialist b. Nothing. A score of 3 or more is needed. c. Further assess the patient’s depression d. Further assess the patient’s drinking behaviors
Answer: D Rationale for why the answer is correct: A score of 2 or more is needed to further assess the drinking behaviors since CAGE is just a screening tool. Site for question/rationale: CAGE Scale Alcohol Screening Tool Handout
235
7. A 16-year-old female presents to the clinic with her mother. She states she is always tired but has trouble sleeping. She has been irritable with her friends and family and has trouble concentrating in school. What would be a priority question for this patient? a. Have you had additional stress in your life lately? b. In the past few weeks, have you wished you were dead? c. Have you lost interest in previous enjoyable activities? d. Do you have a history of depression?
Answer: B Rationale for why the answer is correct: While all of these questions are pertinent, asking about suicide is a priority to ensure the safety of the patient. Site for question/rationale: Mental Health Disorders Presentation, slide 4 and 8. Depression Screening Information for Youth Handout
236
8. Which patient are you going to send to the ER for further workup of a headache? a. A 65-year-old male that has a history of headaches that wake him up at night and last for hours. b. A 22-year-old female that gets headaches that force her to stay in bed for days with nausea and light sensitivity. c. A 55-year-old male that presents to the clinic with a new headache that is the worst he’s ever felt. d. A 30-year-old female that presents to clinic for the 6th time this year rocking back and forth because her head hurts so bad.
Answer: C Rationale for why the answer is correct: A new onset headache and one that is described as a “thunderclap” or the worst the patient has ever had is cause for concern. Other red flags would be change in symptoms, difficulty thinking, or systemic involvement. The others describe cluster, migraines, and cluster, consecutively. Site for question/rationale: Headache Differentials Presentation, minute 6
237
9. A 16-month-old presents with red eyes, cough and congestion. Mom wants to know what’s going on with his eyes- what will you tell her? a. It is likely bacterial conjunctivitis, and we will treat. b. It is likely viral conjunctivitis due to his cold. c. I will stain his eye with fluorescein to see if there’s any trauma. d. I will use the Snellen chart to see if his eye acuity is affected.
Answer: B Rationale for why the answer is correct: Due to the cough, congestion, and both eyes affected, it is likely viral conjunctivitis. Bacterial conjunctivitis typically occurs in one eye suddenly. The rest are inappropriate responses. Site for question/rationale: HEENT Presentation, minute 1:30
238
10. A patient presents to clinic with a sore throat x8 hours. There is no postnasal drip, but the patient has a fever, and headache. What is the most likely diagnosis? a. Mono b. Strep c. Viral d. Postnasal
Answer: B Rationale for why the answer is correct: Mono is a slowly onset than strep. Viral and postnasal both have congestion and postnasal drip. Site for question/rationale: HEENT Presentation, minute 15
239
1. Question: To assess a patient with cardiomegaly, which view of x-ray would be the most accurate to estimate the size of the heart? a. AP View (anteroposterior) b. PA View (posteroanterior) c. Lateral d. Oblique
Answer: B. PA View Rationale for why the answer is correct: The size of the heart is most accurate on the PA view, because of placement of the film in front of the patient, thus the heart is closer to the film. In an AP view the heart is further away and therefore distorted. Site for question/rationale: Slide 20 Radiology From Novice to Expert Part 1
240
2. Question: A 24-year-old male patient presents with dyspnea and right sided chest pain after a severe motor vehicle accident. Upon arrival, physical examination reveals decreased breath sounds over the right lung field. Thoracic ultrasound shows a barcode sign over right lung. What does this indicate? a. Pneumonia b. Pulmonary embolism c. Aortic dissection d. Pneumothorax
Answer: D. Pneumothorax Rationale for why the answer is correct: Normal findings on ultrasound of the lung window is a seashore sign. A barcode sign indicates a pneumothorax. Site for question/rationale: Ultrasound Presentation - Dr. Steve Branham @27:40 Slide 39
241
``` 3. Question: A six-year-old patient presents with sore throat, enlarged tonsils with exudate, fever, a “strawberry like” tongue, and a bright red rash. His mother states the rash seemed spread outward from his trunk to extremities. Upon examination you note a fine, erythematous papular rash, that feels like sandpaper and covers most of his body. What is your leading differential diagnosis? A. Eczema B. Roseola C. Scarlatina D. Staphylococcal scalded skin syndrome ```
Answer: C. Scarlatina Rationale for why the answer is correct: The rash is called Scarlatina and is a rash that appears with Group streptococcus A pharyngitis. The patient’s symptoms of sore throat, enlarged tonsils with exudate, fever, and a strawberry like tongue indicate streptococcal infection. The other choices are less likely to present without such symptoms. Site for question/rationale: Dermatology - Dr. Mercedes Day @27mins
242
4. Question: Which skin condition would you LEAST expect pruritus? a. Impetigo b. Eczema c. Psoriasis d. Actinic keratosis
Answer: D. Actinic keratosis Rationale for why the answer is correct: Actinic keratosis is a thick, scaly, or crusty area caused by sun exposure. It is usually present in the elderly in sun-exposed areas, does not itch and can be dark/light, tan, pink, red, or a combination. Site for question/rationale: Differential Diagnosis: Practice Nurse, Rhoads pg. 267
243
5. Question: A patient with a history of atopic dermatitis presents to your clinic with vesicles, pustules, and “punched-out” erosions with a hemorrhagic crust. Based on this presentation what is your priority action? a. Send immediately to the ER b. Punch biopsy c. Continued observation d. Prescribe topical steroids
Answer: A. Send immediately to the ER Rationale for why the answer is correct: The most likely differential diagnosis is eczema herpeticum also known as Kaposi varicelliform eruption, which is an infection of a pre-existing skin condition with HSV 1 or 2. Eczema herpeticum is considered a dermatologic emergency and patients need to have proper treatment with antivirals from an emergency department. Site for question/rationale: Dermatology - Dr. Mercedes Day @36mins
244
6. Question: A nurse practitioner uses a patient health questionnaire (PHQ-9) to monitor the patient’s response to treatment for a specific mental disorder. Which mental health disorder does the patient have? a. Schizophrenia b. Substance abuse disorder c. Depression d. Generalized Anxiety disorder
Answer: C. Depression Rationale for why the answer is correct: The Patient Health Questionnaire (PHQ-9) is a self-administered questionnaire which is used to monitor the severity of depression and response to treatment. It is not a screening tool. Site for question/rationale: Patient Health Questionnaire (PHQ-9) handout
245
7. Question: A father brings in his 7-year-old son with concerns for his behavior. He explains his son has been having frequent temper tantrums, is easily annoyed, argues frequently, and has been rude to other children in his class. He is concerned because his other children never had behavior like this and grew out of temper tantrums at a much earlier age. What is you leading differential diagnosis? a. Schizophrenia b. Oppositional Defiant Disorder c. Conduct Disorder d. Attention-Deficit/Hyperactivity Disorder (ADHD)
Answer: B. Oppositional Defiant Disorder Rationale for why the answer is correct: The patient’s symptoms suggest oppositional defiant disorder, although oppositional defiant disorder is more common in patients with conduct disorder and ADHD, these would present with different symptoms. ADHD presents with hyperactivity and easy distractibility. Conduct disorder requires certain criteria such as destruction of property or theft. Symptoms of schizophrenia include delusions, hallucinations, and catatonic behavior. Site for question/rationale: Mental Health Disorders Presentation – Dr. Day @11mins
246
8. Question: A patient presents with pain, tearing, and redness to the right eye. The patient also complains that it feels like something is stuck in her eye. You suspect a corneal abrasion. What would be the best way to confirm this diagnosis? a. Examination with Ophthalmoscope b. Test extraocular movements c. Instillation of fluorescein dye d. Measure intraocular pressure
Answer: C. Instillation of fluorescein dye Rationale for why the answer is correct: The instillation of fluorescein dye will illuminate damage to the cornea, under blue light. Site for question/rationale: Instilling Fluorescein Dye (youtube video under HEENT module)
247
9. Question: A patient presents with complaints of excruciating headaches that wake him up at night. He also states the pain is usually periorbital on one side. How would you classify this type of headache? a. Cluster b. Tension c. Migraine d. Acute headache
Answer: A. Cluster headache Rationale for why the answer is correct: Cluster headaches occur in cyclical patterns or clusters and are one of the most painful types of headache. A cluster headache commonly awakens patients in the middle of the night with intense pain in or around one eye on one side of your head. Site for question/rationale: Headache Differentials video @ 3mins and Headaches handout
248
10. Question: A patient presents with the complaint of headache. Of the following symptoms what would be LEAST likely to make you order a scan of the brain, such as a CT or MRI? a. New onset headache in a patient over 50 b. Headache with personality change c. Headache with photophobia d. Headache that worsens with coughing, straining, or sneezing
Answer: C. Headache with photophobia Rationale for why the answer is correct: Choices A, B, and D are all red flags that suggest that the headache is secondary to another diagnosis. While headache with photophobia is common in migraines. Site for question/rationale: To Scan or Not to Scan in Headaches (Goadsby in Clinical Key)
249
1. Question: A patient has been in cardiac arrest in the emergency department for approximately 30 minutes. On the last two pulse checks, the patient was found to be in asystole. Prior to the third pulse check, you ask for the ultrasound so that you may assess for any cardiac activity. Which probe is preferred so that proper cardiac activity can be assessed? a. Linear probe b. Curve linear probe c. Phased array probe d. Intracavitary probe
Answer: phased array probe Rationale for why the answer is correct: the depth of the phased array probe is 35cm and is adequate for assessing the heart. Site for question/rationale: Ultrasound presentation handout, slide 17
250
2. Question: A patient has been rushed into the emergency department status post MVC. The patient had blunt trauma to the abdomen and you, the provider, immediately begin performing a FAST exam on the patient. Which of the following are some of the items you are assessing for in your FAST abdominal exam? (select all that apply) a. Splenic injury b. Free Fluid c. Pelvic/Spinal bone abnormalities d. Pneumothorax/Hemothorax
Answer: Splenic injury, Free Fluid, Pneumothorax/Hemothorax Rationale for why the answer is correct: A FAST exam is used to rapidly assess for abdominal abnormalities such as free or misplaced fluid, abnormal organ size and/or location, pneumothorax, hemothorax, kidney injury, liver or splenic injury, and bladder injury. Site for question/rationale: Ultrasound presentation handout, slide 22-28
251
3. Question: A 58-year-old Caucasian male presents to the clinic one day after his wife found a mole that look “odd” on his back. The patient states he is out in the sun often due to occupation as a construction work. The patient admits that in the hot Texas summer, he removes his shirt to assist in cooling himself down and he never wears sunscreen. On your physical examination, you find the “mole” in question and it is as follows: irregularly shaped, asymmetric, various shades of red and black, and is approximately 7mm in diameter. The provider’s immediate and leading differential diagnosis is: a. Basal cell carcinoma b. Squamous cell carcinoma c. Melanoma d. Merkel Cell Carcinoma
Answer: Melanoma Rationale for why the answer is correct: This skin abnormality follows the ‘ABCDE’ acronym used to rapidly identify melanoma. The patient is also in the high risk category due to being Caucasian, over 50 years of age, and intense and prolonged sun exposure without photoprotection. Site for question/rationale: Dermatology presentation by Dr. Mercedes Day, time 01:24 (m/s).
252
4. Question: Which of the following skin conditions would you use a shave biopsy as your primary biopsy choice? (select all that apply) a. Skin Tags b. Pigmented Lesions c. Superficial Basal Cell Carcinomas d. Superficial Squamous Cell Carcinomas
Answer: Skin tags, Superficial Basal Cell Carcinomas, Superficial Squamous Cell Carcinomas Rationale for why the answer is correct: Skin tags, Superficial Basal Cell Carcinomas, Superficial Squamous Cell Carcinomas can all be appropriately biopsied using the shave biopsy method. You do not use shave biopsy for pigmented lesions. Site for question/rationale: Dermatology presentation by Dr. Mercedes Day, time 14:54 (m/s).
253
5. Question: Which biopsy would you use to completely remove large skin tumors? a. Punch Biopsy b. Excisional Biopsy c. Shave Biopsy d. Total Biopsy
Answer: Excisional Biopsy Rationale for why the answer is correct: Excisional biopsies are used on lesions that require complete removal for diagnostic or therapeutic purposes. They require the greatest expertise and time and always require sutures for closure of the wound. Examples of when to use excisional biopsies include large skin tumors or inflammatory disorders involving the panniculus. Site for question/rationale: Dermatology presentation by Dr. Mercedes Day, time 19:40 (m/s).
254
6. Question: Emma, a 19-year-old college student, comes into the clinic complaining of increasing restlessness for the last 7 months. When asked about associated symptoms, Emma states, “I don’t know if it’s related, but I can’t seem to fall asleep like I used to, and my mom says I’ve been very irritable since I started nursing school several months ago.” When questioned further, she agrees that she feels “wound up” and “worrisome” every day and more than normal. Which common mental health disorder is Emma likely suffering from? a. Substance Abuse Disorder b. Generalized Anxiety Disorder c. Obsessive Compulsive Disorder d. ADHD
Answer: Generalized Anxiety Disorder Rationale for why the answer is correct: To be diagnosed with generalized anxiety disorder, one must have some of the following symptoms for at least six months: restlessness/on-edge/wound-up, easily fatigued, difficulty concentrating or mind going blank, irritable, muscle tension, difficulty controlling feelings of worry, and sleeping problems. Site for question/rationale: Common Mental Health Disorders presentation by Dr. Mercedes Day, time: 01:55 (mm/ss).
255
7. Question: In order to be diagnosed with bipolar disorder, what is the minimum number of “highs” (mania) and “lows” (depression) must a person experience during their lifetime? a. One episode of each b. Two episodes of each c. Three episodes of each d. Four or more episodes of each
Answer: One episode of each Rationale for why the answer is correct: To be diagnosed with bipolar disorder, a person must have experienced at least one manic episode and one depressive episode during their lifetime. Site for question/rationale: Common Mental Health Disorders presentation by Dr. Mercedes Day, time: 04:45 (mm/ss).
256
8. Question: Leah, a 13-year-old female, comes into the clinic with a chief complaint of sudden right eye redness and irritation. Leah’s father states that she has also been having discharge from that eye and “crusty parts” but denies cough or cold-like symptoms. What is the differential that best fits this case just with the information provided? a. Viral Conjunctivitis b. Bacterial Conjunctivitis c. Corneal Abrasion d. Eye Floaters
Answer: Bacterial Conjunctivitis Rationale for why the answer is correct: Bacterial conjunctivitis has a sudden onset, whereas viral conjunctivitis may develop after a few days and patients often complain of “cold-like” symptoms and cough. Site for question/rationale: Ears, Nose and Throat presentation by Dr Amy Moore and Dr Grace Sun, time: 01:25 (mm/ss).
257
9. Question: Sarra, a 27-year-old female, presents to clinic with ear pain. After speaking with Sarra, the provider notes that she has the following symptoms: gradual onset of bilateral ear pain, sore throat, nasal congestion with postnasal drip, and a slight, nonproductive cough. Which of the following most closely encompasses the etiology of Sarra’s ear pain? a. Bacterial b. Allergic c. Neuropathic d. Trauma e. Referred Pain
Answer: Allergic Rationale for why the answer is correct: Allergic origins of ear pain include: gradual onset, +/- sore throat, +/- cough, +/-fatigue, +/-headache, +/- nodes in the neck, + nasal congestion/discharge, + postnasal drip. Site for question/rationale: Ears, Nose and Throat presentation by Dr Amy Moore and Dr Grace Sun, time: 09:25 (mm/ss).
258
10. Question: 16-year-old Andrew, accompanied by his mother, comes to the clinic for generalized complaints of pain in his throat that increases when swallowing, fatigue, and headache. Andrew’s mother states that she think he has “just caught some bug” because he has been out a lot and spending time with his new girlfriend. When asking Andrew more about his associated symptoms, he states that he has been alternating between fevers and chills, but denies nasal congestion, or cough. On exam, the provider notes posterior nodes in the neck. Which of the following illnesses should the provider have at the top of their differential list? a. Strep Throat b. Mononucleosis c. Postnasal Drip d. Tonsillar Abscess
Answer: Mononucleosis Rationale for why the answer is correct: Mono is an illness that often causes a sore throat (abrupt or gradual onset), pain when swallowing, fever/chills, headache, and posterior nodes in the neck. Andrew has all of these symptoms, plus he has a possible exposure source with his new girlfriend (mono is often called the “kissing disease”). Site for question/rationale: Ears, Nose and Throat presentation by Dr Amy Moore and Dr Grace Sun, time: 15:35 (mm/ss).
259
1. Question: The size of the heart will appear larger on x-ray film due to the hearts distance from the film in which view? A. PA (posteroanterior) B. AP (Anteroposterior) C. Lateral D. Oblique
Answer: B. AP (Anteroposterior) Rationale for why the answer is correct: In the AP view the heart is further away from the film and appears distorted. Therefore, you are unable to estimate heart size reliably. Site for question/rationale: Dr. Rito Dello Stritto Understanding Radiology lecture. Slide 7
260
2. Question: Which finding on a PA x-ray would be indicative of cardiomegaly? A. Heart exceeds ½ the width of the chest. B. Heart is not visualized on the right side. C. Heart has a straight-line border D. Kerley B lines are present
Answer: A. Heart exceeds ½ the width of the chest Rationale for why the answer is correct: The heart size exceeds 1/2 the width of the chest on PA view is correct. The heart not visualized on the right could indicate a right sided pneumonia. A straight-line border is seen with a tension pneumothorax, and Kerley B lines are seen in COPD. Site for question/rationale: Dr. Rito Dello Stritto Understanding Radiology lecture. Slide 29
261
3. Question: The ACNP is utilizing an ultrasound to check for suspected renal calculi. What color finding when visualizing the kidney would lead the diagnosis of kidney stones? A. Black B. Grey C. Blue or Red D. White
Answer: D. White Rationale for why the answer is correct: Hyperechoic items appear white on ultrasound. They reflect most of soundwaves and indicate dense structures such as calcium deposits. Site for question/rationale: Dr. Steve Branham Ultrasound handouts, slide 11.
262
``` 4. Question: An ACNP finds a growth on a patient during assessment. Which of the following findings would be concerning for melanoma? A. Solid Brown color B. Round and regular in shape C. Diameter greater than 6mm D. Symmetrical shape ```
Answer: C. Diameter greater than 6mm Rationale for why the answer is correct: The ABCDE of checking for melanoma include, asymmetry, border irregularities, color variations, diameter > 6mm, and evolution of the growth. Site for question/rationale: Dr. Mercedes Day, Dermatology lecture, Timestamp 1:00.
263
5. Question: When performing a punch biopsy, what is the smallest amount of the tissue that must be collected to ensure an accurate diagnosis? A. 3 mm B. The entire lesion C. 1mm D. 0.5 mm
Answer: A. 3mm Rationale for why the answer is correct: 3 mm is the smallest size sample that should be collected to give an accurate diagnosis. Removing the entire lesion is not necessary for diagnosis. Both 1mm and 0.5 mm are not large enough to diagnosis. Site for question/rationale: Dr. Mercedes Day, Dermatology lecture, Timestamp 16:40
264
6. Question: The ACNP is conducting a CAGE questionnaire to assess for potential alcohol abuse. Which of the following responses from the patient could indicate alcohol abuse is present? A. I have had a drink in the morning (eye-opener) to get rid of my hang over, I feel guilty about my drinking B. I drink a few drinks most weekend. C. I have had a hangover due to my drinking D. My family annoys me when criticizing my drinking.
Answer: A. I have had a drink in the morning (eye-opener) to get rid of my hang over, I feel guilty about my drinking. Rationale for why the answer is correct: C.A.G.E. questionnaire consists of 4 questions to evaluate if the patient feels the need to Cut down on drinking, if you feel Annoyed you when others criticize your drinking, if the patient has felt Guilty about their drinking, and have you ever had the need for an Eye-opener in the morning to calm your nerves or help a hangover. To indicate an alcohol abuse issue two or more of the questions must be answered yes. B. is not an indication for abuse. C. Is not a question covered by cage but it may warrant more questions. D. is appropriate but it requires two positive responses to indicate potential abuse. Site for question/rationale: Cage Scale Alcohol Screening Tool Questionnaire
265
7. Question: Which of the following symptoms would be consistent with a diagnosis of Bipolar disorder? A. Excessive cleaning, rearranging items, repetitive activities, hoarding B. Easily distracted, lack of focus, inability to sit still and C. Serious and significant mood swings with both mania and depression D. Hallucinations, delusions, and incoherent speech
Answer: C. Serious and significant mood swings with both mania and depression Rationale for why the answer is correct: Mood swings, mania and depression are hallmarks of bipolar disorder. The patient experiences high, highs and low, lows. Answer A. is consistent with OCD. Answer B is consistent of ADHD and answer D. is consistent with schizophrenia. Site for question/rationale: Dr. Mercedes Day, Mental Health Disorders lecture, Timestamp 4:30.
266
8. Question: A patient presents with complaints of severe throbbing pain on one side of the head. nausea, vomiting, and extreme sensitivity to light and sound. Which of the following types of headaches are your lead differential diagnosis? A. Sinusitis B. Cluster C. Tension D. Migraine
Answer: D. Migraine Rationale for why the answer is correct: Migraines are classical manifest with symptoms of severe throbbing pain or a pulsing sensation, usually on just one side of the head. It is often accompanied by nausea, vomiting, and extreme sensitivity to light and sound. Site for question/rationale: Module 4, HEENT, Headache handout pdf.
267
9. Question: A patients presents to the ER with complaint of the “worst headache that I have ever had.” What diagnostic test should be completed to rule out a possible bleed? A. P.E.T Scan B. CT Scan C. X-RAY D. M.R.I
Answer: B. CT Scan Rationale for why the answer is correct: A CT scan is the gold standard for diagnosis of a brain bleed. Site for question/rationale: Module 4, Dr. Amy Moore and Dr. Tara Hilliard presentation on Headache differentials. Timestamp 5:19
268
10. Question: A patient is seen by the ACNP with complaints of an acute onset sore throat with very painful swallowing, fever, and chills. On exam you note the tonsillar lymph nodes are inflamed. What is your leading differential diagnosis at this point of the exam? A. Viral Infection B. Strep Throat C. Mononucleosis D. Allergic Rhinitis
Answer: B. Strep Throat. Rationale for why the answer is correct: All of the symptoms listed are consistent with Strep throat. A viral infection usually has a gradual onset and may have a mild sore throat but the tonsillar lymph nodes are more specific to Strep. Mono may have many of the same symptoms with more posterior inflamed lymph nodes. Allergic Rhinitis would not include a fever or chills. Site for question/rationale: Module 4, Dr. Amy Moore and Dr. Grace Shun presentation on HEENT. Timestamp 15:15.
269
1. Question: Which statement is true about a chest x-ray? A. You can estimate the size of the heart from an AP view. B. You can estimate the size of the heart from a PA view. C. Patient must be able to stand for a chest x-ray. D. On a normal chest x-ray you should be able to count 10 or more spaces between ribs.
Answer: B – You can estimate the size of the heart from a PA view. Rationale for why the answer is correct: On the PA view, the heart is closer to the film, so the size is more accurate. On the AP view, the heart is further away from the film, so size is distorted. Can do AP chest x-ray if patient is unable to stand. On a normal chest x-ray, 8-10 rib spaces show normal lung expansion; 10 or more shows hyperinflation. Site for question/rationale: Understanding Radiology – Stritto ppt, slides 18-30
270
``` 2. Question: On ultrasound, what does a barcode sign indicate? A. Gall bladder obstruction B. Hydronephrosis C. Pneumothorax D. DVT ```
Answer: C - Pneumothorax Rationale for why the answer is correct: Gall bladder will appear as thin-walled, pear shaped structure beneath liver; sludge, gallstones, or polyps may be visible. Hydronephrosis on ultrasound appears as dilation extending from renal pelvis to calyces. Pneumothorax on ultrasound in M-mode will have a barcode sign – no motion of lung sliding. DVT on ultrasound is identified by direct visualization of thrombotic material in the examined vessel. Site for question/rationale: Ultrasound presentation – Dr. Steve Branham
271
``` 3. Question: What is the most common malignancy in Caucasians and usually occur on the face? A. Melanoma B. Squamous Cell Carcinoma C. Merkel Cell Carcinoma D. Basal Cell Carcinoma ```
Answer: D – Basal Cell Carcinoma Rationale for why the answer is correct: BCC most common, SCC 2nd most common, Melanoma is the most deadly, but not the most common, Merkel Cell Carcinoma is rare. Site for question/rationale: Dermatology Video – Mercedes Day
272
``` 4. Question: Which childhood skin disease will appear in a classic Christmas Tree distribution? A. Molluscum Contagiosum B. Scarlatina C. Pityriasis Rosea D. Roseola ```
Answer: C – Pityriasis Rosea Rationale for why the answer is correct: Molluscum Contagiosum is known for flesh, colored, dome-shaped papules with an indentation area in the middle. Scarlatina is known for its sandpaper-like exanthem and “strawberry-like” tongue. Roseola has a classic exanthem, rose pink macular popular rash beginning on trunk and may spread to neck and extremities. Pityriasis Rosea is a scaly, pink salmon, plaque that develops along skinfolds on trunk into a Christmas tree distribution. Site for question/rationale: Dermatology Video – Mercedes Day
273
5. Question: What does A, B, C, D, and E stand for when assessing skin lesions? A. Asymmetry, Border, Color, Diameter, and Evolution B. Asymmetry, Bullous, Character, Diameter, and Evolution C. Appearance, Border, Color, Diameter, and Effects D. Appearance, Bullous, Color, Description, and Evolution
Answer: A – Asymmetry, Border, Color, Diameter, and Evolution. Rationale for why the answer is correct: These characteristics are often symptoms of skin lesions that would be concerning. Site for question/rationale: Skin Lesion presentation – Dr. Amy Moore and Dr. Grace Sun
274
``` 6. Question: You are seeing a 16-year-old female for a well visit. During your interview, she stated that she “drinks at parties” with her friends. Which questionnaire would you consider using to get more information about her drinking? A. GAD-7 B. SCARED C. M-CHAT D. CAGE ```
Answer: D- CAGE Rationale for why the answer is correct: GAD-7 and SCARED are screening tools for anxiety, M-CHAT is a screening tool for autism. CAGE is a screening tool for alcohol use. Site for question/rationale: Mental Health Module, CAGE Scale – Alcohol Screening Tool
275
7. Question: Signs and symptoms that may be indicative of ADHD? A. Inattention, hyperactivity, and impulsivity B. Excessive anxiety or worry that causes problems in social interactions, school, or work. C. Significant mood swings, Highs and lows D. Feelings of sadness or loneliness, lack of energy, thoughts of death or suicide
Answer: A – Inattention, hyperactivity, and impulsivity Rationale for why the answer is correct: A are signs and symptoms of ADHD; B are signs and symptoms of Generalized Anxiety Disorder; C are signs of Bipolar Disorder; D are signs of Depression. Site for question/rationale: Mental Health Disorders Video – Mercedes Day
276
``` 8. Question: A 25-year-old female presents your clinic with a complaint of headaches. When taking a history, the patient explained that her headaches occur approximately twice per month, are usually on one side, last 2 days, and she is sensitive to light. She states that laying down in a dark room is the only thing that helps her pain. Her pain is 8/10. With this limited information, what is her most likely diagnosis? A. Tension headache B. Migraine headache C. Cluster headache D. Brain tumor ```
Answer: B – Migraine headache Rationale for why the answer is correct: Tension headaches are usually described as a band around the head. Migraine headaches are moderate to severe intensity, often unilateral and associated photosensitivity, causing an avoidance of activity. Cluster headaches are episodic, unilateral, often centered around one eye, and often associated with restlessness or agitation and runny nose or congestion. Brain tumor headache may or may not be preceded by a history of cancer and would be a concern if there were associated vision or hearing problems, change in headache pattern, nausea or vomiting. Site for question/rationale: Video – Headache Differentials – Dr. Moore and Dr. Hilliard
277
9. Question: Fluorescein Dye can be used to? A. Highlight defects in corneal epithelium. B. Diagnose periorbital cellulitis. C. Diagnose cataracts. D. Screen for Ocular hypertension.
Answer: A – highlight defects in corneal epithelium Rationale for why the answer is correct: Fluorescein is an organic compound and dye that will stain the epithelium of the eye, highlighting corneal abrasions, ulcers, or foreign objects in the eye. Fluorescein is not used in the diagnosis of periorbital cellulitis, cataracts, or ocular hypertension. Site for question/rationale: Instilling Fluorescein Dye video.
278
10. Question: Which condition is a contraindication for cerumen removal? A. Less than 4 years of age B. Patient uses hearing aids. C. Patient has sensation of ear fullness. D. Patient has otitis externa.
Answer: D – Patient has otitis externa Rationale for why the answer is correct: Otitis externa is the only answer listed that is a contraindication to cerumen removal. Site for question/rationale: Removal of Impacted Cerumen Clinical Key Todd Thomsen, MD
279
2. While examining a patient’s posteroanterior (PA) view chest x-ray, you notice the cardiac silhouette is >½ the total width of the chest. You must consider: a. Hyperinflation of the lungs b. Hemothorax c. Congestive heart failure d. An anteroposterior (AP) view x-ray
Answer: Congestive heart failure Rationale for why the answer is correct: When examining a PA view chest x-ray, the cardiac silhouette should not exceed >1/2 of the total width of the chest. If so, consider, pericardial effusion, cardiomyopathy or CHF. You can only estimate the size of the heart on the PA view. Site for question/rationale: Rita A. Dello Stritto, Basic Radiology power point, Slide #34 and #35. (page 12, slide 1 &2).
280
3. You are assessing a 73-year-old Caucasian patient who comes in to get a “mole checked out”. He mentions that he has had this mole for about 4 years and notices that it “has slowly been getting bigger” He also reports significant sun exposure as a kid working in tobacco fields, “I’d spend hours out in the sun and would get terrible sunburns! But as a kid you don’t know any better, and I had to work!” You note a pink colored papule on the cheek with rolled borders and a telangiectatic vessel. You know this lesion is most characteristic of: a. Kaposi’s sarcoma b. Squamous cell carcinoma c. Merkel cell carcinoma d. Basal cell carcinoma
Answer: d. Basal cell carcinoma Rationale for why the answer is correct: Basal cell carcinoma lesions are best characterized by slow growth, most common malignancy in Caucasians, significant UV exposure as children, present usually on the face and head. Papules are pink, flesh colored, with rolled borders, have a pearly or translucent quality and have telangiectatic vessels and/or ulcerations. Site for question/rationale: Mercedes Day dermatology lecture, 3:11 – 5:09. Picture obtained from Mercedes Day dermatology lecture, 3:11-5:09.
281
4. A patient arrives to the clinic to have a skin tag removed. The skin tag is confined to the epidermis, therefore, the best biopsy technique to be used should be a: a. Punch biopsy b. Shave biopsy c. Excisional biopsy d. Debridement
Answer: b. shave biopsy Rationale for why the answer is correct: Lesions elevated above the skin or confined to the epidermis are best suitable for shave biopsies. Some examples of these include seborrheic keratosis, actinic keratosis, skin tags, warts or superficial basal cell or squamous cell carcinomas Site for question/rationale: Mercedes Day Dermatology video lecture, 14:23 – 14:53.
282
5. This skin disorder is caused by Parvovirus B19 and is characterized by a malar rash with circumoral pallor, known as “slapped cheek rash” in children or presents as acute arthritis in adults. Clinical manifestations may be benign or life threatening depending on age, hematological and immunologic status. This condition is known as: a. Scarlatina b. Roseola c. 5th disease d. 6th disease
Answer: c. Rationale for why the answer is correct: 5th disease is caused by Parvovirus B19, clinical presentations vary from benign to life threatening and depend on age, hematological and immunologic status. Erythema infectiosum is a mild febrile illness characterized by an erythematous malar rash with circumoral pallor known as “slapped cheek rash”. Most commonly occurs in children but may also occur in adults and may present with acute arthritis with or without a rash. Site for question/rationale: Mercedes Day Dermatology lecture 29:36 – 30:52
283
6. A 20-year-old female presents to the clinic complaining of fatigue, difficulty concentrating, tension headaches and insomnia for the last 3 years. She states she’s had trouble at work and difficulty making friends. She reports she’s been told before that she’s very self-conscious and admits to frequently replaying conversations/scenarios in her head worried about the way she might have come across, making her lose sleep. She said she’s always been like this, but realized it wasn’t normal to feel this stressed all the time and decided to finally get some help. As she’s explaining, you note she’s biting her nails, and has a slightly disheveled appearance. You associate these symptoms most closely resemble: a. ADHD b. Bipolar disorder 2 c. Conduct disorder d. Generalized anxiety disorder
Answer: d. Generalized anxiety disorder Rationale for why the answer is correct: Generalized anxiety disorder are people who have excessive anxiety or worry about a number of things, including everyday life routines, social interactions, work and school which cause problems. Symptoms include feeling wound up or on-edge, easily fatigued, difficulty concentrating, irritable, muscle tension, difficulty controlling feelings of worry and sleeping problems. Site for question/rationale: Mercedes Day Common Mental Health Disorders lecture, 1:51 – 2:34.
284
7. A mother brings in her 12-year-old boy to get checked out. The mother states she is concerned about her child’s behavior and doesn’t know what else to do. She states he has been suspended from school three times this year for aggression towards his peers and teachers, trying to start a fire in the boys’ restroom, stealing the teacher’s cell phone and wallet and this last time (a week ago), for carrying a knife in school. These behaviors closely resemble: a. Schizophrenia b. Post-partum depression c. Conduct Disorder d. PTSD
Answer: C. Conduct disorder Rationale for why the answer is correct: Conduct disorder is characterized by a repetitive pattern of behaviors in which societal norms are violated. Criteria include 3 or more within the past 12 months and at least one within the past six months: aggression to people and/or animals, intentional destruction of property, deceitfulness and theft and serious violations of rules. Site for question/rationale: Mercedes Day Common Mental Health Disorders Lecture, 6:13 – 7:45.
285
8. A patient presents to the clinic complaining of a unilateral headache behind and around the eye. “I woke up last night around 3 am and the headache was pretty intense.” This type of headache is known as a: a. Tension headache b. Migraine with aura c. Tension headache d. Cluster headache
Answer: d. Cluster headache Rationale for why the answer is correct: One of the most common primary headaches are cluster headaches which occur in cyclical patters or clusters and are one of the most painful types of headache. It commonly awakens you in the middle of the night with intense pain around one eye on one side of your head. Site for question/rationale: Module 4, HEENT, Headaches pdf, page #1
286
9. A patient presents to the clinic stating he’s still having trouble seeing without his glasses even though he had Lasik surgery a year ago. You assess his eye with the ophthalmoscope, and you note the patient has keratoconus. This means: a. The patient has a lazy eye b. The patient has keratin build up in the cornea c. The patient has a keloid scar in his cornea d. The patient’s cornea is thin, and cone shaped
Answer: d. The patient’s cornea is thin, and cone shaped Rationale for why the answer is correct: Keratoconus: the cornea, which is normally round, becomes thin and cone shaped. Site for question/rationale: Module 4, HEENT, Eyes pdf, page #1
287
10. A 45-year-old male presents to the clinic complaining of nasal congestion with clear drainage. He states these symptoms come and go but has noticed his symptoms get worse around cats or in places where there are carpets. He denies sinus pain, fever or malaise. Upon assessment, you note increased ocular lacrimation, swollen nasal turbinates and postnasal drip. Judging by the presenting symptoms and by the physical assessment your leading diagnosis is: a. Strep throat b. Otitis media c. Perennial allergic rhinitis d. Sinusitis
Answer: c. Rationale for why the answer is correct: Perennial allergic rhinitis may result from sensitivity to pet hair, mold on wallpaper, houseplants, carpeting, upholstery and automobile pollution. Clinical presentations of allergic rhinitis: afebrile, clear purulent nasal drainage, congestion, nasal pruritus, may or may not have postnasal drip, has triggers, may or may not have sinus pain, nasal mucosa may be swollen, may or may not have increased lacrimation and will have nasal pruritus. Site for question/rationale: Module 4 HEENT, Nose pdf, page #1 and HEENT presentation by Dr. Amy Moore and Dr. Grace Sun 16:35 – 21:56.
288
``` 1. Question: An 18-year-old patient walks into to the emergency room after an accident on the soccer field. He is complaining of pain on his left side. You suspect rib fracture and wish to order an X-ray. You order: A. AP view B. Portable x-ray C. AP, lateral, and oblique views D. PA and lateral view ```
Answer: D. PA and lateral view Rationale for why the answer is correct: If the patient is able to stand, a PA and lateral view should be ordered. Site for question/rationale: Radiology lecture by Dr. Rita A. Dello Stritto, Radiology: From Novice to Expert, Part 1, slide 18.
289
``` 2. Question: Which radiology view is most reliable for detecting a patient’s heart size? A. AP view B. PA and lateral view C. PA view D. Oblique view ```
Answer: C. PA view Rationale for why the answer is correct: AP view demonstrates a distorted and enlarged view of the heart. PA and lateral view are not both necessary. Oblique view also not necessary. PA view is closest to the film, making it the most reliable method for determining the size of the patient’s heart. Site for question/rationale: Radiology lecture by Dr. Rita A. Dello Stritto, Radiology: From Novice to Expert, Part 1, slides 19 and 20.
290
``` 3. Question: A 71-year-old Caucasian male patient presents to your office with a concern for a papule that is pink in color on his left cheek. He describes no pain. On closer examination, you note the papule to have a shiny appearance with rolled borders. A differential diagnosis you most consider may be: A. Squamous cell carcinoma B. Merkel cell carcinoma C. Melanoma D. Basal cell carcinoma ```
Answer: D. Basal cell carcinoma Rationale for why the answer is correct: Classic appearance of basal cell carcinomas include a pink, flesh colored papule that may have a shiny appearance with rolled borders. This type of lesion occurs commonly in Caucasian persons who have had exposure to ultraviolet sun exposure, especially during childhood. Site for question/rationale: Module 3, Dermatology lecture by Mercedes Day, FNP starting at 3:09 of 39:21 minutes.
291
``` 4. Question: You are working as an FNP in a clinic, preparing to numb a patient’s finger before biopsy on a lesion. You most likely will choose which anesthetic? A. Lidocaine 1% plus epinephrine B. Lidocaine 0.5% plus epinephrine C. Lidocaine 1% - 2% alone D. Lidocaine 2% plus epinephrine ```
Answer: C. Lidocaine 1% - 2% alone Rationale for why the answer is correct: Utilizing 1%-2% is appropriate for anesthetizing small areas. The use of epinephrine on a digit is not recommended because it is a debated topic in its use. Site for question/rationale: Module 3, Dermatology, Overview of local anesthetics with punch biopsy lecture by Dr. Amy Moore, starting at 0:42 of 11:26 minutes.
292
5. Question: You have a patient who appears in your office with several small warts on the bottom of his foot. You discuss the benefits, risks, and contraindications for treating him with cryotherapy. The following are contraindications (select all that apply): A. Compromised circulation in the area B. Patient has keloids in the area C. Patient has severe active ulcerative colitis D. Patient is being treated for malignant melanoma
Answer: A, C, D Rationale for why the answer is correct: Compromised circulation in the area, severe active ulcerative colitis, and malignant melanoma are all contraindications for cryotherapy. Keloids is an indication for cryotherapy. Site for question/rationale: Module 3, Dermatology, Cryotherapy on Warts by Tuggy & Garcia
293
``` 6. Question: Autism Spectrum Disorder (ASD) is a developmental disorder that can be diagnosed at any age. The American Academy of Pediatrics recommends specific testing for autism at which well checks (select all that apply): A. 12-month well-check B. 18-month well-check C. 24-month well check D. 30-month well check ```
Answer: B and C, 18-month and 24-month well checks Rationale for why the answer is correct: The recommendation from the American Academy of Pediatrics is that all children be screened for developmental delays at 9-months, 18-months, 24-months, and 30-months. However, children should be specifically screened for autism at their 18 and 24-month well checks. Site for question/rationale: Module 3, Mental Health, Autism-NIH Screening, National Institute of Mental Health https://www.nimh.nih.gov/health/topics/autism-spectrum-disorders-asd/index.shtml
294
``` 7. Question: You are seeing a 20-year-old patient in the clinic who has completed the ASQ questionnaire. Your goal is to screen the patient for: A. ADHD B. Alcohol dependency C. Anxiety D. Suicidal thoughts ```
Answer: D. Suicidal thoughts Rationale for why the answer is correct: ASQ stands for Ask Suicide-screening Questions is a 4-item screening tool that can be utilized to screen pediatric and adult patients for suicide risks. Site for question/rationale: Module 3, Mental Health, Suicidal Screenings & Helpful Information – NIH https://www.nimh.nih.gov/research/research-conducted-at-nimh/asq-toolkit-materials/index.shtml
295
8. Question: A 23-year-old female patient arrives to your clinic, stating she is very worried because she has been awakened in the middle of the night by severe unilateral pain near her left eye. This has happened to her now three times in the past 3 weeks. You suspect: A. Trigeminal autonomic cephalalgia (TAC), a primary headache B. Cluster headache, a primary headache C. Chiari malformation, a secondary headache D. Migraine, a primary headache
Answer: B. Cluster headache, a primary headache Rationale for why the answer is correct: Cluster headaches are considered one of the most painful primary headaches, which happen in a cyclical pattern and a common feature is the headache awakens the person in the middle of the night with intense pain in the eye or on one side of the person’s head. Site for question/rationale: Module 4, HEENT, Headaches: file:///C:/Users/Admin/Downloads/Headaches.pdf
296
``` 9. Question: As an APRN, you are examining a 53-year-old patient who has a history of Acquired Immune Deficiency Syndrome (AIDS), hypertension, hypercholesteremia, and depression who presents with bilateral eye irritation and redness. You do not observe drainage. You suspect which of the following conditions: A. Keratoconus B. Uveitis C. Conjunctivitis D. CMV Retinitis ```
Answer: D. CMV Retinitis Rationale for why the answer is correct: People with immune disorders, including AIDS are more likely to be affected by CMV retinitis, a serious infection that affects the retina. Site for question/rationale: Module 4, HEENT, Eyes: file:///C:/Users/Admin/Downloads/Eyes.pdf
297
10. Question: You are an APRN practicing in a family health clinic, when your previously established 35-year-old female patient presents to your office. The patient has a history of hypertension, Type 2 diabetes, and tension headaches. As you enter the room, the patient is holding her temples, crying, and stating “This is the worst headache I’ve ever had! I feel like thunder crashed in my head!” Your first plan of action is: A. Send her to the ER immediately for evaluation B. Prescribe new medication for headaches C. Perform a neurological exam D. Determine her hydration status
Answer: A. Send her to the ER immediately for evaluation Rationale for why the answer is correct: The patient describing the worst headache she has ever had is a red flag for a possible brain bleed or a blood clot, so time is critical, and she needs to have a CT scan at the hospital. Site for question/rationale: Module 4, HEENT, Headache differentials video by Dr. Amy Moore and Dr. Tara Hilliard, starting at 3:50 of 8:52 minutes.
298
1. Question: A MVC trauma patient comes to the emergency department, you need to examine the patient quickly for any internal bleeding in the chest and abdomen. What is the ultrasound exam for this procedure? a) Abdominal trauma scan b) Extended focused abdominal scan for trauma c) Focused extremity scan d) Abdominal series radiographic films
Answer: Focused abdominal scan for trauma Rationale for why the answer is correct: eFAST, also known as extended focused abdominal scan for trauma checks for blood in the abdomen, its quick, easy, and can be done at the bedside. Site for question/rationale: Branham, S. (n.d.). Ultrasound Presentation. Video presentation time 18:12.
299
2. Question: You just received a female trauma patient in the emergency room who was in a motorcycle accident, heart rate is 145 bpm, and is moaning and guarding abdomen. You do eFAST exam, it is positive, which one of the following comprises a positive exam? a) Air bubble in the colon b) Fluid visible in the pouch of douglas c) Normal seashore sign, no barcode sign d) Moderate stool in the sigmoid
Answer: B Rationale for why the answer is correct: eFAST answers simple clinical questions from the ultrasound for any significant free fluid in the pericardial, peritoneal, chest cavity, or if there is a pneumothorax. A positive finding in any fluid visible in the potential spaces is abnormal such as the pouch of douglas in the rectouterine pouch. Site for question/rationale: Branham, S. (n.d.). Ultrasound Presentation. Video presentation time 18:58.
300
3. Question: John is a 55-year-old male comes to your clinic with a lesion that started about 6 months ago to his right upper forehead and noticed that it is quickly growing. He says that he works in the sun all day as a farmer for the last 30 years. You evaluate the lesion using what acronym? a) AFPC (asymmetry, fullness, pallor, color) b) ABCDE (asymmetry, border, color, direction, evolution) c) ABCDE (asymmetry, border, color, diameter, evolution) d) ABEF (asymmetry, border, evolution, fullness)
Answer: C Rationale for why the answer is correct: By using the acronym ABCDE helps the provider look for asymmetry, border irregularities, color varigation from red, blue, or gray, diameter that is greater 6 mm, and evolution which means a lesion is changing in size, shape, or color. Site for question/rationale: Day, M. (n.d.). Dermatology. Video time presentation 00:55.
301
4. Question: Malignant melanoma is a serious form of skin cancer which has high risk factors. Which is NOT considered a risk factor for malignant melanoma? a) White males b) Family history of melanoma c) Blistering or painful sunburn <30 years of age d) Dark hair phenotype
Answer: D Rationale for why the answer is correct: Malignant melanoma is a serious skin cancer and high risk patients include white males, >50 years of age, family history, blistering or painful sunburn less than 30 years old or as a child, chronic outdoor activities without adequate protection, indoor tanning beds, immunocompromised, and red hair phenotype. Site for question/rationale: Day, M. (n.d.). Dermatology. Video time presentation 1:40.
302
5. Question: A 15-year-old male comes to your clinic with his mother who says he developed rash two days ago and is becoming worse where its spreading to his arms and legs. You assess the rash and you run your hand over the rash and feels “sand-papery.” You suspect the patient has: a) Mononucleosis b) Group Streptococcal A c) Herpes zoster d) Fifth Disease
Answer: B Rationale for why the answer is correct: Scarlatina is caused by Streptococcus Group A which is a rash that is an erythematous popular which starts in groin/axilla and spreads to extremities. This rash has the sandpaper like exanthem and are 1-2 mm in diameter. This also may affect the tongue with a strawberry appearance. Site for question/rationale: Day, M. (n.d.). Dermatology. Video time presentation 26:55.
303
6. Question: Jennifer is a 24-year-old female who is currently in college studying to be a registered nurse. She has recently been having excessive worries, fear of being in a crowd or at a school party, feeling on edge, and difficulty concentrating for the last 8 months which is becoming worse in the last few weeks. What mental disorder is Jennifer experiencing at this time? a) Attention deficit-Hyperactivity Disorder b) Generalized Anxiety c) Bipolar disorder d) Substance Use disorder
Answer: B Rationale for why the answer is correct: To be diagnosed with generalized anxiety disorder, a person must have symptoms for at least 6 months. The symptoms include having excessive anxiety or worry, constant fear in social interactions, school, or work, restless, on edge, easily fatigued, irritable, difficulty concentrating, mind going blank, muscle tension, difficulty controlling feelings of worry, and sleeping problems. Site for question/rationale: Day, M. (n.d.). Common Mental Health Disorders. Video time presentation 01:52.
304
7. Question: Mary is at her 2-week newborn visit with her midwife. She says that she has been taking care of the baby by herself, her mother is not able to help, and her husband is out on deployment. She reports that she has had excessive crying, difficulty bonding with her baby, has loss of energy, and having thoughts of hurting herself the last two nights. You know this is postpartum depression, what is your next priority? a) Send her home and tell her she is going to be fine. b) Tell her she needs to follow up with her primary doctor. c) Tell her it is okay to feel depressed and we will get you some help right now. d) Get her admitted in a psychiatric facility in two weeks.
Answer: C Rationale for why the answer is correct: Postpartum depression can occur during pregnancy or after delivery. Symptoms include depressed moods, severe mood swings, excessive crying, loss of energy, difficulty bonding with baby, withdrawing, problems with sleeping, anxiety or panic attacks, and thoughts of harming self or baby. This must meet criteria for a major depressive episode and last greater than 2 weeks. These women need to be screened at newborn or postpartum visits and needs to start intervention right away. Site for question/rationale: Day, M. (n.d.). Common Mental Health Disorders. Video time presentation 11:40.
305
8. Question: A patient presents to the emergency department having painful swallowing, fevers, fatigue, and started suddenly today. You feel cervical anterior lymphadenopathy which is most consistent with this type of infection: a) Rhinosinusitis b) Otitis media c) Meningitis d) Streptococcus pharyngitis
Answer: D Rationale for why the answer is correct: Streptococcus pyogenes can cause painful swallowing due to enlarged tonsils, usually is abrupt, develops fever, fatigue, no cough, and has anterior lymph nodes in the neck. Site for question/rationale: Moore, A. & Sun, G. (n.d.). Ears, Nose and Throat. Video presentation time 12:16.
306
``` 9. Question: Sara, 56-year-old female says she started developing fever 3 days ago, unilateral sinus pain, symptoms started about 12 days ago, and her congestion is becoming worse. You diagnose this patient with: A) Allergic rhinitis B) Acute viral pharyngitis C) Acute bacterial rhinosinusitis D) Nasal polyps ```
Answer: C Rationale for why the answer is correct: Acute bacterial rhinosinusitis is most consistent with fever, congestion, post-nasal drainage, duration is greater than 10 days, unilateral, and has red and purulent drainage. Allergic rhinitis and nasal polyps has no fever and not unilateral. Acute viral pharyngitis is in the throat. Site for question/rationale: Moore, A. & Sun, G. (n.d.). Ears, Nose and Throat. Video presentation time 15:55.
307
10. Question: A patient comes to the emergency department complaining of blurred vision and right eye pain. You know that this patient will need a visual acuity using: a) Tumbling E chart b) Rosenbaum chart c) DMV eye chart d) Snellen eye chart
Answer: D Rationale for why the answer is correct: The Snellen chart is used to measure visual acuity in the medical setting for anyone who can read letters. Site for question/rationale: Moore, A. & Sun, G. (n.d.). Ears, Nose and Throat. Video presentation time 02:48.
308
1. Question: According to the radiodensity scale air is? a. black on films b. white on films c. grey on films d. light on films
Answer: a. black on films Rationale for why the answer is correct: The radiodensity chart displays air as being black on imaging films. Site for question/rationale: Rita Dello Strito Understanding Radiology Presentation page #2, slide #4.
309
2. Question: Which diagnostic exam is equivalent to 90 chest x-rays? a. Ultrasound (US) b. HIDA scan c. Magnetic resonance imaging (MRI) d. Computerized tomography (CT scan)
Answer: d. computerized tomography (CT scan) Rationale for why the answer is correct: CT verses ultrasound, CT generates ionizing radiation 90x that of chest x-rays. Site for question/rationale: Steve Branham Ultrasound Presentation minute 5:12.
310
3. Question: Which one of these is NOT consistent with ABCDE assessment of melanoma? a. Asymmetrical b. Color variation c. Dry patches d. Evolution
Answer: c. dry patches Rationale for why the answer is correct: A – asymmetrical, b – border irregularities, c- color variation, d – diameter >6mm, e- evolution. Site for question/rationale: Mercedes Day Dermatology Presentation minute 1:05
311
4. Question: Basal cell carcinoma is the most common malignancy of which race? a. Black b. Hispanic c. Caucasian d. Asian
Answer: c. Caucasian Rationale for why the answer is correct: Basal cell carcinoma is the most common malignancy among Caucasians. Site for question/rationale: Mercedes Day Dermatology Presentation minute 4:19
312
5. Question: When considering your differentials, you always want to consider the most concerning differential. Which differential is most concerning? a. Keratoacanthomas b. Dermal Nevi c. Sebaceous hyperplasia d. Keratoses
Answer: a. Keratoacanthomas Rationale for why the answer is correct: When considering your differentials, you always want to consider the most concerning differential. Most concerning differentials versus benign slide. Site for question/rationale: Mercedes Day Dermatology Presentation minute 23:34.
313
6. Question: Characterization of this disorder include experiencing mania and depression. a. Substance abuse disorder b. Bipolar disorder c. Depression d. Schizophrenia
Answer: b. Bipolar disorder Rationale for why the answer is correct: Bipolar disorder experience significant mood swings and are characterized as “highs” (mania) and “lows” (depression). Site for question/rationale: Mercedes Day Mental Health Disorders Presentation minute 4:40
314
7. Question: What statement is true about post-partum depression. a. Post-partum depression has to last longer than 1 week to be diagnosed b. Post-partum depression only occurs during pregnancy c. Post-partum depression may be diagnosed up to 6 months after birth d. Post-partum depression is not a serious debilitating illness
Answer: c. Post-partum depression may be diagnosed up to 6 months after birth Rationale for why the answer is correct: a – last longer than 2 weeks; b – can occur during or after pregnancy; c – true; d – post-partum depression IS a serious debilitating illness Site for question/rationale: Mercedes Day Mental Health Disorders Presentation minute 12:11
315
8. Question: A patient presents to clinic with a chief complaint red painful eyes that started 1 week ago and has progressively gotten worse. He reports nasal congestion and postnasal drip. On exam lymph nodes in neck are non-palpable. What is the patients most likely diagnosis? a. Bacterial infection b. Allergies c. Foreign body in eye d. Referred pain
Answer: b. allergies Rationale for why the answer is correct: Illness script. Bacterial infection and foreign body (trauma) typically have abrupt onset; and postnasal drip is typically not associated with referred pain. Site for question/rationale: Amy Moore and Grace Sun HEENT Presentation minute 9:45
316
9. Question: A patient presents with a chief complaint of painful swallowing that started early upon waking up this morning. On exam posterior lymph nodes are firm and palpable. What is the most likely diagnosis? a. Strep b. Mono c. Viral d. Postnasal
Answer: b. mono Rationale for why the answer is correct: Illness script. Viral and Postnasal typically have a gradual onset. On exam anterior lymph nodes are palpable in viral. Site for question/rationale: Amy Moore and Grace Sun HEENT Presentation minute 15:22
317
10. Question: Patient presents with chief complaint of fever for the last 3 days. He reports left sided face pain and left sided nasal drainage. On exam nasal mucosa is red and purulent. What is the most likely diagnosis? a. acute viral rhinosinusitis b. acute bacterial rhinosinusitis c. allergic rhinitis d. nonallergic rhinitis
Answer: b. acute bacterial rhinosinusitis Rationale for why the answer is correct: Illness script. Fever >48 hours, unilateral pain and drainage, and red purulent mucosa are all pertinent positives for acute bacterial rhinosinusitis. Site for question/rationale: Amy Moore and Grace Sun HEENT Presentation minute 20:34
318
1. Question: When doing an abdominal FAST exam, what are you primarily screening for? Answer: A) Tendon/ligament injuries B) Bladder C) Ruptured organ D) Fluid
D Rationale for why the answer is correct: You are screening for intra-abdominal bleeding. Site for question/rationale: Principles of Ultrasound For the ACNP Student by Steve Branham PhD, MSN, ACNP-C, FNP-C, CCRN, FAANP at 16:47
319
2. Question: What is the difference between PA and AP view? Answer: A) There is no difference B) The heart appears larger on the AP view C) The heart appears larger in PA view D) The clavicles can only be seen from the PA view
B Rationale for why the answer is correct: In the AP view the heart is further away and therefore appears distorted and bigger. Site for question/rationale: Understanding Radiology by Rita A. Dello Stritto RN, PhD, CNS, ENP slide 19/153.
320
3. Question: Where is basal cell carcinoma most commonly found? Answer: A) Face and head B) Back C) Hands and arms D) Neck
A Rationale for why the answer is correct: Face and head because this is the area that is most exposed to UV light. Site for question/rationale: Dermatology by Mercedes Day DNP, RN, FNP-C at 4:14
321
4. Question: Why should one avoid removing the punch to see progress during a punch biopsy? Answer: A) Causes for incomplete biopsy sample B) Results in a ragged wound and shredded biopsy sample C) Results in wound that cannot be sutured closed D) Causes areas to bleed
B Rationale for why the answer is correct: Removing the punch biopsy to see progress results in a ragged wound and shredded biopsy sample that cannot be used. Site for question/rationale: Dermatology by Mercedes Day DNP, RN, FNP-C at 18:03
322
5. Question: Pityriasis Rosea can usually starts on what body part? Answer: A) Trunk B) Hands and feet C) Face and neck D) Skin folds
A Rationale for why the answer is correct: The “Herald patch” starts off on the trunk and after several days to weeks spreads in the “Christmas tree” like distribution. Site for question/rationale: Dermatology by Mercedes Day DNP, RN, FNP-C at 34:25
323
6. Question: What is the difference between Bipolar I and Bipolar II disorder? Answer: A) Bipolar I experiences mania more than depression B) Bipolar II experiences mania more than depression C) A manic episode in Bipolar I must last at least a week D) A manic episode in Bipolar II can must last at least one month
C Rationale for why the answer is correct: In a manic episode (Bipolar I) a person experiences extreme happiness, extreme irritability, hyperactivity, and feeling of being able to do anything for at least one week. Site for question/rationale: Common Mental Health Disorders by Mercedes Day DNP, RN, FNP-C at 4:55
324
7. Question: The range of postpartum depression onset is from pregnancy to - Answer: A) immediately after birth B) 6 months after birth C) one week after birth D) 1 month after birth
B Rationale for why the answer is correct: Can begin up to six months after birth which is why screening for postpartum depression at newborn visits, subsequent well-child visits, and postpartum visits is extremely important. Site for question/rationale: Common Mental Health Disorders by Mercedes Day DNP, RN, FNP-C at 11:39
325
8. Question: What is the difference between viral and bacterial conjunctivitis? Answer: A) Viral will have cough/cold symptoms prior to eye redness B) Viral is very sudden C) Bacterial most commonly involves both eyes D) It is very unlikely to have viral conjunctivitis cross contaminate and affect both eyes.
A Rationale for why the answer is correct: Viral conjunctivitis will have cough/cold symptoms for a couple of days and then will get red eyes whereas bacterial is very sudden and most commonly just in one eye. Site for question/rationale: Ears, Nose and Throat by Amy Moore DNP, APRN, FNP-c and Grace Sun DNP, APRN, FNP-BC at 0:55
326
9. Question: With what pathophysiology would a patient experience postnasal drip? Answer: A) Bacterial B) Allergic C) Trauma D) Referred pain
B Rationale for why the answer is correct: The only one with a postnasal drip is allergic. Site for question/rationale: Ears, Nose and Throat by Amy Moore DNP, APRN, FNP-c and Grace Sun DNP, APRN, FNP-BC at 08:12
327
10. Question: What symptom is experienced in acute viral rhinosinusitis, acute bacterial rhinosinusitis, allergic rhinitis, and nonallergic rhinitis? Answer: A) Fever B) Purulent drainage C) Nasal pruritis D) Congestion
D Rationale for why the answer is correct: As stated and emphasized by Dr. Moore in the lecture “everyone may complain of some congestion”. Site for question/rationale: Ears, Nose and Throat by Amy Moore DNP, APRN, FNP-c and Grace Sun DNP, APRN, FNP-BC at 17:46
328
1. Question: What ultrasound probe is used to assess arteries and veins? a. Linear probe b. Curve Linear probe c. Phased array probe d. Intracavitary Probe
Answer: A Rationale for why the answer is correct: Linear probes are used to assess arteries/veins, procedures, pleura, skin/soft tissues, musculoskeletal, testicle/hernia, and breast. Site for question/rationale: Ultrasound presentation/Handouts-Dr. Steve Branham. Slide 14 and video at 11:46
329
2. Question: What are the three ultrasound modes? a. Two-dimensional, M-Mode, Color=Flow b. One dimensional, M-Mirror, B-Black c. Two-dimensional, M-Magnetic, Color=flow d. Two-dimensional, M-Mode, C-Contrast
Answer: A Rationale for why the answer is correct: There are three ultrasound modes Two-dimensional (2D or brightness) used for POCUS. M-Mode used for Motion and Color=Flow that builds a color map of red, blue or white. Site for question/rationale: Ultrasound presentation/Handouts-Dr. Steve Branham. Slide 9 and video at 8:51.
330
3. Question: What does the acronym ABCDE stand for when assessing for Melanoma? a. Atypical, Border irregularities, Color variegation, Diameter >6, Evolution b. Asymmetry, Black in color, Color variegation, Diameter >5, Evolution c. Asymmetry, Border irregularities, Color variegation, Diameter >6, Evolution d. Atypical, Border irregularities, Color variegation, Diameter >5, Evolution
Answer: C Rationale for why the answer is correct: ABCDE stands for Asymmetry, Border irregularities, Color variegation, Diameter >6, Evolution Site for question/rationale: Dermatology presentation/video-Dr. Mercedes Day at 1:08
331
4. Question: A 75-year-old women comes into the clinic presenting with a hyperkeratotic nodule on her nose. She admits that she used tanning beds frequently when younger. A biopsy is done to confirm diagnosis. What type of carcinoma can you suspect? a. Basal Cell Carcinoma b. Squamous Cell Carcinoma c. Merkel Cell carcinoma d. Non-Hodgkin Lymphoma
Answer: B Rationale for why the answer is correct: Squamous Cell Carcinoma is a malignant proliferation of epidermal keratinocytes. Risk factors include UV exposure, age, and ethnicity. Papules, plaque or nodule can be smooth, hyperkeratotic, or ulcerative lesions. Most commonly in the neck and face. A biopsy should be done to confirm diagnosis. Site for question/rationale: Dermatology presentation/video-Dr. Mercedes Day at 6:37.
332
5. Question: A patient comes into the clinic complaining of a skin rash. Upon examination, the patient has red scaly patches on his elbows, knees, and the palms of his hands. He reports that these areas are “very itchy”. What type of skin disorder does this patient have? a. Impetigo b. Herpes Zoster c. Eczema d. Psoriasis
Answer: D Rationale for why the answer is correct: Psoriasis is characterized as scaly patches, plaques, or papules that are red in color. They are found most commonly on the elbows, knees, palms of hands, and the soles of the feet. Pruritis is also common with eczema. Site for question/rationale: Skin Disorders Presentation- Dr. Amy Moore & Dr. Grace Sun at 9:45
333
6. Question: Bipolar I disorder can be characterized as a. A hypomanic disorder b. A depressive episode c. A manic disorder d. A substance abuse disorder
Answer: C Rationale for why the answer is correct: Bipolar I disorder is a manic episode that is characterized as extreme happiness, extreme irritability, hyperactivity, and they feel like they can do anything. Site for question/rationale: Common Mental Health Disorders Presentation- Dr. Mercedes Day at 5:44
334
7. Question: According to the American Psychiatric Association, Posttraumatic Stress disorder is composed of 5 main components. Which component is NOT correct? a. Experiencing a traumatic event b. Re-experiencing the traumatic event c. Engaging in avoidance d. Suffering from the experience e. Decrease in arousal symptoms
Answer: E Rationale for why the answer is correct: The 5 main components of PTSD are experiencing a traumatic event, re-experiencing the traumatic event, engaging in avoidance, suffering from the experience and an increase in arousal symptoms. Site for question/rationale: Common Mental Health Disorders Presentation- Dr. Mercedes Day at 13:40
335
8. Question: A 30-year-old women presents to the clinic with a chief complaint of a “terrible headache” that has lasted for 48 hours. She describes her headache as unilateral. She is sensitive to light and she has been experiencing nausea and vomiting. Based on this patients’ symptoms, what type of headache does she have? a. Tension b. Cluster c. Migraine d. Acute
Answer: C Rationale for why the answer is correct: A migraine headache is characterized by unilateral pain that can last anywhere between 4-72 hours. Symptoms can consist of an aura, sensitivity to light and smells, and nausea/vomiting. Migraine headaches occur more commonly in women and can also be hormone related. Site for question/rationale: Headache differentials video- Dr. Amy Moore & Dr. Hilliard at 1:47
336
9. Question: What type of headache is a tension headache classified as? a. Vascular b. Muscle Contraction c. Traction d. Inflammatory
Answer: B Rationale for why the answer is correct: Muscle Contraction=Tension Headache. Vascular=Migraine and/or Cluster Headache. Mixed=Muscle and Vascular. Traction or inflammatory=Acute Headache Site for question/rationale: Headache differentials video- Dr. Amy Moore & Dr. Hilliard at 1:00
337
10. Question: A 12-year-old comes into the clinic and his mother is concerned he has “pink eye”. Pink eye also known as Conjunctivitis is a. A degenerative form of eye disease in which the lens gradually becomes opaque b. Where the eye protrudes from the eye socket due to swelling of the muscles, fat, and tissue behind the eye c. An inflammation or infection of the transparent membrane that lines the eyelid and covers the white part of the eyeball d. Caused by fluid accumulation in the macula and blurred vision can occur
Answer: C Rationale for why the answer is correct: Conjunctivitis is the inflammation or infection of the transparent membrane that lines the eyelid and covers the white part of the eyeball. Bulging eyes or proptosis occurs when eye protrudes from the eye socket due to swelling of the muscles, fat, and tissue behind the eye. Cataracts are a degenerative form of eye disease in which the lens gradually becomes opaque and vision mists over. Diabetic Macular edema is caused by fluid accumulation in the macula and blurred vision can occur. Site for question/rationale: Eyes PDF page 1.
338
``` 1. Question: You are viewing a chest radiograph of your 3 year old patient. Which of the following findings can have a healthy, normal etiology? A: a flattened diaphragm B: fluid filled costophrenic angles C: a widened mediastinum D: RML air trapping ```
Answer: C Rationale for why the answer is correct: A widened mediastinum at this age is likely present due to the remaining thymus gland tissue. Other causes should be ruled out, but pediatric patients can have a thymus gland on x ray up to the approximate age of 8. Site for question/rationale: (Mrs. Dello Stritto, Page 23 of slide notes)
339
2. Question: You order a chest radiograph for your 16 year old male patient for concern of pleural effusion; when considering pleural effusions you know that: A: they are often caused by a bleb rupture B: your needle decompression will be in the mid clavicular line and 2nd intercostal space C: the x-ray will likely have more than 10 ribs visible D: 250 ml of fluid may need to accumulate before visualization can be possible on a PA film
Answer: D Rationale for why the answer is correct: radiographs taken too early with suspected pleural effusion may not show the fluid. 250-300 ml of fluid may be required prior to visualization (PA view) Site for question/rationale: (Mrs. Dello Stritto, Page 25 of slide notes)
340
``` 3. Question: These common skin problems are all usually pruritic except: A: Actinic Keratosis B: Dermatitis C: Psoriasis D: Eczema ```
Answer: A Rationale for why the answer is correct: Actinic Keratosis is not a pruritic lesion. It is seen with age and sun exposure. Site for question/rationale: (Dr. Moore’s skin disorder lecture, minute 5:36)
341
``` 4. Question: Psoriasis may present with all the symptoms below except: A: Redness B: dermatome orientation C: Scaly plaques D: Pruritis ```
Answer: B Rationale for why the answer is correct: B, Herpes Zoster is a rash that is found to travel/present along dermatomes. Psoriasis is usually found spread throughout different areas of the body, often elbow and knees are involved. Site for question/rationale: (Dr. Moore’s skin disorder lecture, minutes 10:30)
342
``` 5. Question: Your patient, a healthy term newborn, has the presence of well demarcated plaques that are greasy in appearance with yellowing scales on their head. What is the most likely cause? A: Mongolian spots B: Meningococcal Purpura C: Seborrheic Dermatitis D: Erythema Toxicum Neonatorum ```
Answer: C Rationale for why the answer is correct: Seborrheic dermatitis or “cradle cap” appears as greasy plaques on the neonate’s head. The other lesions are possibly age appropriate, but would be described differently. Site for question/rationale: (Dr. Day’s Dermatology lecture, minute 12:56)
343
6. Question: All are part of the CAGE questionnaire except: A: Have you ever been told you get angry when you drink? B: Have you ever considered cutting down on your drinking? C: Have you ever felt guilty about your drinking? D: Do you ever drink an eye-opener in the morning to steady your nerves?
Answer: A Rationale for why the answer is correct: Similar, but the question in the CAGE does not ask about anger specifically. The question asks if you have ever been “annoyed” when others have criticized your drinking habits. Site for question/rationale: (CAGE questionnaire PDF)
344
7. Question: You are caring for a mother of a 5 month old infant who displays symptoms of loss of energy, withdrawing from family and friends and new panic attacks, you are concerned for postpartum depression. You know that postpartum depression: A: presents before the new baby is 1 month old B: Does not increase the chance of harm for the baby C: Meets the clinical diagnostic criteria for depression D: Only happens to Mothers with advanced maternal age
Answer: C Rationale for why the answer is correct: Postpartum depression is a serious illness that affects many women. It meets the diagnostic criteria of depression and lasts longer than 2 weeks, can occur up to 6 months after delivery of the baby and does increase the risk of harm to the newborn and the new mother. Site for question/rationale: (Mercedes Day lecture, minute 11:51)
345
8. Question: The pneumatic bulb on the otoscope is used, primarily in pediatrics, to help with: A: diagnosing bacterial conjunctivitis B: the diagnosis of otitis media by testing the movement of the eardrum C: creating a suction in the nares during examination D: distraction during difficult examination of toddlers
Answer: B Rationale for why the answer is correct: I have seen it used for D as well, but that is not the primary purpose of the pneumatic bulb. Pediatric guidelines recommend tympanometry as an accurate way to confirm a suspected diagnosis of otitis media. Site for question/rationale: (HEENT Dr. Moore lecture, Minute 3:50)
346
``` 9. Question: Headaches that occur in a cyclical pattern and are very painful (often to one side of the head) are likely to be: A: tension headaches B: migraine headaches C: cluster headaches D: secondary to an acute sinusitis ```
Answer: C Rationale for why the answer is correct: Definition of cluster headaches describes HA that come in a cyclical pattern and are very painful, often affect one side of the head or near one eye. Tension, migraine and HA associated with sinusitis can have similar symptoms, but do not fit the description as specifically. Site for question/rationale: (Headache PDF)
347
``` 10. Question: This eye pathology involves the light sensitive portion of the eye as it is separated from the nerves and blood supply it needs. What problem does this describe? A: retinal detachment B: keratoconus C: uveitis D: macular degeneration ```
Answer: A Rationale for why the answer is correct: This is the definition of retinal detachment, it is important to know what all four of these eye problems are. Site for question/rationale: (Eye handout, PDF)
348
``` 1. Question: The heart appears larger in what type of view? A. PA view B. Lateral view C. AP view D. It appears the same in all views ```
Answer: C Rationale for why the answer is correct: The heart appears larger in the AP view, the heart is further away, therefore distorted. Site for question/rationale: Understanding radiology by Dr. Rita Dello Stritto slide 7
349
``` 2. Question: What are benefits of POCUS? A. Prompt screening B. Heightened diagnostic accuracy (early narrowing of differential) C. No ionizing radiation D. All of the above ```
Answer: D Rationale for why the answer is correct: Prompt screening, no ionizing radiation, and heighted diagnostic accuracy are all benefits of POCUS. Site for question/rationale: Principles of ultrasound for the ACNP student slide 6
350
``` 3. Question: Match definition to term. Non palpable lesions that vary in pigmentation form the surrounding skin. There are no elevations or depressions of the skin. A. Macules B. Papules C. Nodules D. Vesicles ```
Answer: A Rationale for why the answer is correct: Macules are non-palpable lesions that vary in pigmentation form the surrounding skin. There are no elevations or depressions of the skin. Site for question/rationale: Differential Diagnosis for the advance practice nurse: Terminology for documenting rashes page 263
351
4. Question: Fifth Disease is caused by what type of virus? a. Epstein- Barr virus b. Parvovirus B19 c. Human Herpes Virus 8 d. poxvirus
Answer: B Rationale for why the answer is correct: Fifth disease is caused by Parvovirus B19. Site for question/rationale: Dermatology presentation by Dr. Day minute 29:52
352
5. Question: 1% Lidocaine is the most common type of local anesthetic used in a punch biopsy. What is the max dose of lidocaine? a. 4.5mg/kg b. 2.5mg/kg c. 6.5mg/kg d. This does not need to be dosed kg/mg
Answer: A Rationale for why the answer is correct: The max dose of lidocaine is 4.5mg/kg Site for question/rationale: Overview of local anesthetic with punch biopsy minute 1:08
353
6. Question: A score of 9 on the PHQ-9 would indicate what? a. No depression b. Mild depression c. Minimal depression d. Severe depression
``` Answer: B Rationale for why the answer is correct: A score of 9 would indicate the patient has mild depression. Total score Depression severity 1-4 Minimal depression 5-9 Mild depression 10-14 Moderate depression 15-19 Moderately severe depression 20-27 Severe depression ``` Site for question/rationale: Patient Health Questionnaire 9 page 2
354
7. Question: The CAGE screening is used for? a. Depression screen b. Alcohol abuse c. Drug abuse d. Suicide assessment
Answer: B Rationale for why the answer is correct: The CAGE scale is used to indicate potential problems with alcohol abuse. The CAGE acronym is a 4-item questionnaire that stands for Cut, Annoyed, Guilty, Eye opener all referring to alcohol intake. Site for question/rationale: CAGE Alcohol Questionnaire tool
355
8. Question: Which type of headache would have a periorbital pain, tearing and nasal drainage on affected side? a. Cluster headache b. Migraine headache c. Tension headache d. All of them
Answer: A Rationale for why the answer is correct: In a cluster headache, the patient will wake up from there sleep with a periorbital pain. Cluster headaches are also more common in males. Site for question/rationale: Headache differential presentation minute 2:47
356
``` `9. Question: A 24-year-old female presents to the clinic with a fever of 100.9 for the past 3 days along with unilateral sinus pain and purulent drainage. She denies nasal pruritis, triggers and eye tearing. What is your leading diagnosis on this patient? A. Acute viral Rhinosinusitis B. Acute bacterial rhinosinusitis C. Allergic rhinitis D. Non allergic rhinitis ```
Answer: B Rationale for why the answer is correct: Patients with acute bacterial rhinosinusitis present with a fever for greater than 48 hrs. along with unilateral sinus pain and unilateral purulent drainage. They also have congestion and their duration of symptoms is greater than 10 days. They do not have nasal itching, triggers or eye tearing. Site for question/rationale: HEENT presentation minute 21:39
357
10. Question: What type of tool can you use to measure visual acuity? a. Ophthalmoscope b. Snellen Eye chart c. Fluorescein staining d. None of them
Rationale for why the answer is correct: Snellen Eye chart is used to measure visual acuity. Site for question/rationale: HEENT presentation minute 2:57
358
1. Question: A chest xray was done on an 80 year old male presenting with increasing shortness of breath and cough, with a history of smoking. PA and lateral views were done showing more than 10 ribs counted, diaphragms appear flattened, and increased AP diameter on lateral film. What condition can these findings confirm? a. Asthma b. COPD c. Pneumonia d. CHF
Answer: b. COPD Rationale for why the answer is correct: Destruction of the distal airways enlarge the amount of airspace within the lungs causing outflow obstruction which leads to overinflation. Site for question/rationale: Part 2, slide 15, Dr. Dello Stritto’s radiology presentation
359
2. Question: Which of the following indicates a positive eFAST finding? a. An ultrasound of the gallbladder reveals a gallstone after a pt presents with increasing abdominal pain b. A pt presents to the ER with increased work of breathing after a car accident showing a seashore sign with lung sliding during a chest ultrasound c. A pt presents to the ER with a penetrating abdominal wound to the RUQ revealing fluid in Morison’s pouch d. Decreased compressibility of femoral vein during DVT screening
Answer: c. A pt presents to the ER with a penetrating abdominal wound to the RUQ revealing fluid in Morison’s pouch Rationale for why the answer is correct: Any fluid visible in any of the potential spaces is abnormal. Morison’s pouch is the potential space separating the abdominal cavity from the kidney and fluid that accumulates in this space could indicate collection of blood. Site for question/rationale: Dr. Branham’s ultrasound presentation, 21:25 minutes
360
3. Question: What type of lesions are palpable discrete lesions that measure less than 5 mm and can present isolated or grouped? a. Macules b. Papules c. Nodules d. Plaques
Answer: b. papules Rationale for why the answer is correct: papules are palpable discrete lesions that measure less than 55 mm, presenting as isolated or grouped. Site for question/rationale: Ch 33, pg 263 in Differential Diagnosis for the Advanced Practice Nurse, Rhoads
361
4. Question: An 18 y/o male patient complains of an itchy and scaly rash that has become more widespread over the past week. Flesh colored plaque appear on his back, and he states before the rash appeared, he experienced a low-grade fever with a sore throat and headache. He seeks advice and treatment for his rash. What would be an appropriate response to tell this patient? a. This is an emergency and the patient needs to go to the ER to seek appropriate care b. A serologic test is needed to detect a positive parvovirus B19 specific IgM antibody c. An antibiotic will be prescribed for strep pharyngitis d. No treatment is necessary, the rash will resolve within 8 weeks and recurrence is usually rare.
Answer: d. no treatment is necessary, the rash will resolve within 8 weeks and recurrence is usually rare. Rationale for why the answer is correct: Pityriasis Rosea is a common and self-limiting skin condition that presents with scaly, pink, salmon or flesh colored plaque which appears often on the trunk. After several days or weeks, new patches or plaques may form and preceding symptoms may occur including fever, headache, cough, and arthralgia. The condition usually resolves in about 8 weeks and recurrence is rare. Site for question/rationale: Dr. Day’s dermatology presentation, 34:20 minutes
362
5. Question: What type of malignancy is seen with a rapidly growing, painless, firm nontender shiny, flesh-colored or bluish-red intracutaneous nodule most commonly found on the head and neck region? a. Melanoma b. Squamous cell carcinoma c. Merkel cell carcinoma d. Basal cell carcinoma
Answer: c. merkel cell carcinoma Rationale for why the answer is correct: Merkel cell carcinoma is a rare, aggressive cutaneous malignancy and presents with rapidly growing, painless, firm, nontender shiny, flesh-colored or bluish-red, intracutaneous nodule and common on head and neck region Site for question/rationale: Dr. Day’s dermatology presentation, 7:19 minutes
363
6. Question: After administrating Suicide-Screening Questions to an adolescent, what steps must be taken for a positive screen with the patient having thoughts of suicide? Select all that apply. a. Patient requires a STAT safety/full mental health evaluation b. Patient requires a brief suicide safety assessment to determine if a full mental health evaluation is needed c. Remove all dangerous objects from room. d. Patient is only able to leave after a brief suicide safety assessment is completed
Answer: a and c Rationale for why the answer is correct: if a patient has a positive screen and has suicidal thoughts, the patient requires a STAT safety/full mental health evaluation, the patient cannot leave until evaluated for safety, the patient must be kept in sight, and remove all dangerous objects from room. The physician or clinician responsible for patient’s care must also be alerted. Site for question/rationale: Depression screening for youth
364
7. Question: Which of the following questions are included when screening for potential problems with alcohol abuse? a. Have you ever felt you needed to cut down on your drinking? b. Have people annoyed you by criticizing your drinking? c. Have you ever felt guilty about drinking? d. Have you ever felt you needed a drink first thing in the morning to steady your nerves or to get rid of a hangover? e. All of the above
Answer: e. all of the above Rationale for why the answer is correct: the CAGE alcohol questionnaire is a 4-item questionnaire that can indicate potential problems with alcohol abuse. Answering 2 “yes” out of 4 questions may indicate a problem with alcohol. Site for question/rationale: CAGE Scale – Alcohol Screening Tool
365
8. Question: What type of headache is generally a diffuse, mild to moderate pain in the head that is often described like a tight band around the head, and usually with an unknown etiology? a. Migraine b. Tension c. Cluster d. Acute sinusitis
Answer: b. tension Rationale for why the answer is correct: tension headaches are described as a diffuse, mild to moderate pain in your head that is often described as feeling like a tight band around the head. A tension headache is the most common type of headache yet causes have an unknown etiology. Site for question/rationale: Headaches handout
366
9. Question: What would you order for a patient who complains of afebrile recurring rhinitis with clear discharge and frequent congestion, bilateral sinus pain, and nasal pruritis? a. Amoxicillin every 8 hours for 10 days b. Referral to ENT c. CBC to rule out infection d. Allergy skin testing or referral to allergist
Answer: d. allergy skin testing or referral to allergist Rationale for why the answer is correct: This patient is most likely experiencing allergic rhinitis due to its recurring nature, along with signs and symptoms of being afebrile, clear nasal discharge, bilateral sinus pain, and nasal pruritis. Since it is recurrent, the patient would benefit from having allergy testing done to rule out allergens that may be causing these symptoms. Site for question/rationale: Dr. Moore and Dr. Sun’s ENT presentation, 24:30 minutes
367
10. Question: This condition is caused by fluid accumulation in the macula with possible blurred vision. a. Cataracts b. Glaucoma c. Diabetic macular edema d. Uveitis
Answer: c. diabetic macular edema Rationale for why the answer is correct: diabetic macular edema is caused by fluid accumulation in the macula. Blurred vision can occur and may be severe. Site for question/rationale: Eyes handout
368
1. Which of the following appear radiolucent on x-ray? a. Bone b. Barium c. Air d. Metal
Answer: C – Air. Rationale for why the answer is correct: Radiolucent objects are low density and appear black on x-ray, and include items such as air and glass (Dr. Stritto’s Radiology powerpoint, slides 3 and 4)
369
2. Presence of ten or more ribs spaces on a PA chest x-ray is indicative of what? a. Barotrauma b. Pleural effusion c. Pnemothorax d. Hyperinflation
Answer: D. hyperinflation Rationale for why the answer is correct: 8-10 rib spaces on a PA view is normal, 10 or more is indicative of hyperinflation (Dr. Stritto’s Radiology powerpoint, slide 28).
370
3. An x-ray which penetrates from the back to the front is called what view? a. AP (anteroposterior) b. PA (posteroanterior) c. Lateral d. Oblique
Answer: B. PA (posteroanteror) Rationale for why the answer is correct: The first word of the x-ray view is where the beam enters and the second is where the x-ray exits (Dr. Stritto’s Radiology powerpoint, slides 7 and 8)
371
4. Flesh-colored, dome shaped papules on the skin which are caused by the pox virus is which common childhood skin disorder? a. Molluscum contagiosum b. Fifth’s disease c. Roseola d. Scarlatina
Answer: A – molluscum contagiosum Rationale for why the answer is correct: Molluscum contagiosum is a common childhood skin disorder characterized by flesh-colored, dome shaped papules which may be anywhere except the soles and palms, and may be transmitted via sexual contact, contact sports, and can be itchy or inflamed. Site for question/rationale: Dr. Mercedes Day’s video on Dermatology, Time Mark 32:44
372
5. The most common malignancy in Caucasians is a. Melanoma b. Basal cell carcinoma c. Squamous cell carcinoma d. Breast cancer
Answer: B Rationale for why the answer is correct: basal cell carcinoma is the most common malignancy in Caucasians Site for question/rationale: Dr. Mercedes Day’s video on Dermatology, Time Mark 4:04
373
6. A score of 2 out of 4 on the CAGE questionnaire may indicate what? a. Depression b. Anxiety c. A problem with alcohol abuse d. A problem with drug abuse
Answer: C Rationale for why the answer is correct: The CAGE questionnaire is a self-administered 4-item questionnaire which may indicate a potential problem with alcohol. Site for question/rationale: https://psychology-tools.com/cage-alcohol-questionnaire/
374
7. Difficulty falling or staying asleep, restlessness, fatigue, difficulty concentrating, and excessive worry are all symptoms of a. Depression b. ADHD c. Generalized anxiety disorder d. Bipolar disorder
Answer: C Rationale for why the answer is correct: sleeping difficulties, restlessness or feeling ‘wound up’, fatigue, difficulty concentrating, irritability, muscle tension, and difficulty controlling feelings of worry are all symptoms of GAD. Site for question/rationale: Dr. Mercedes Day lecture on Common Mental Health Disorders, Time Mark 2:14 – 2:33
375
8. A 4 year old patient who presents to the clinic with a three day history of cough and runny nose now has redness and irritation of both eyes. What is a possible differential diagnosis for this patient? a. Bacterial conjunctivitis b. Influenza c. Viral conjunctivitis d. Rhinovirus
Answer: C Rationale for why the answer is correct: Conjunctivitis may be bacterial or viral. Bacterial conjunctivitis is often sudden onset of redness, irritation, and drainage to one eye, whereas viral conjunctivitis is often preceded by upper respiratory symptoms and affects both eyes. Site for question/rationale: Dr. Amy Moore and Dr. Grace Sun’s ENT lecture, time mark 1:17-1:40
376
9. A primary headache is caused by which of the following: a. Exercise b. Acute sinusitis c. Brain tumor d. Concussion
Answer: A Rationale for why the answer is correct: A primary headache is caused by overactivity of or problems with pain-sensitive structures in your head. Lifestyle choices such as wine or lack of sleep, cough, exercise, sex or stress can cause primary headaches. Tension headaches, cluster headaches, and migraines are also primary headaches. Site for question/rationale: Module 4, HEENT Headaches handout (https://ttuhsc.instructure.com/courses/6100/files/784184/download?wrap=1)
377
10. Sneezing, runny or itchy nose, irritation and congestions which are accompanied by the presence of IgE is a presentation of: a. Sinusitis b. Allergic rhinitis c. Non-allergic rhinitis d. Deviated septum
Answer: B Rationale for why the answer is correct: Allergic rhinitis occurs when the body over-responds to a specific non-infectious particle by releasing antibodies (IgE) into the lungs, mucus membranes, and skin. IgE attaches to mast cells and releases histamines, producing the symptoms of redness, inflammation, sneezing and congestion. Site for question/rationale: Module 4, HEENT Nose handout (https://ttuhsc.instructure.com/courses/6100/files/784200/download?wrap=1)
378
``` 1. Question: What is the most common mode used for Point of Care Ultrasound (POCUS)? A. Two-Dimensional (2-D or brightness) B. M-Mode C. Color = Flow D. Three-Dimensional (3-D or brightness) ```
Answer: A Rationale for why the answer is correct: It produces grey to black images, whereas M-Mode is most often used to view cardiac structures or view vessel flow. Color = Flow is like a pulse wave doppler or a spectral doppler that builds a color map to allow the provider to visual flow toward or away from the probe. 3-D is not a mode. Site for question/rationale: Principles of ultrasound for the ACNP Student PPT Slide #8
379
2. Question: What can an eFAST tell the provider? A. Significant free fluid is present in the pericardial cavity. B. Free fluid in the peritoneal cavity. C. Free fluid in the chest cavity. D. All of the above.
Answer: D Rationale for why the answer is correct: An eFAST is an ultrasound exam designed for trauma patients that can be done at the bedside to check for blood in the chest and abdomen. Site for question/rationale: Principles of ultrasound for the ACNP Student PPT Slide #27
380
3. Question: Identify the benefits of POCUS. A. Does not require ionizing radiation. B. Heightened diagnostic accuracy. C. Allows fast screening of patients both inpatient and outpatient. D. Directly visible findings.
Answer: A. B. C. D. Rationale for why the answer is correct: These are all benefits of POCUS. Site for question/rationale: Principles of ultrasound for the ACNP Student PPT Slide #6
381
4. Question: Kaposi Sarcoma is an infection with human herpes infection eight. What are the types of Kaposi Sarcoma? A. Classic. B. Endemic. C. Iatrogenic. D. Acquired Immunodeficiency syndrome (AIDS) associated.
Answer: A, B, C, D Rationale for why the answer is correct: Site for question/rationale: Dr. Mercedes Day - Dermatology Video 10 minutes 30 Seconds into recording.
382
``` 5. Question: The APRN collects data from the patient with a chief complaint of severe itching on the face and groin area. The skin is red and the APRN notes there are some weeping patches near the groin and some crusting to areas on the face. What is the most likely skin disorder this patient is experiencing? A. Psoriasis B. Dermatitis C. Eczema D. Impetigo ```
Answer: C. Rationale for why the answer is correct: Site for question/rationale: Dr. Amy Moore and Dr. Grace Sun - Skin Disorder video 9 minutes and 10 seconds into recording for complete table of skin disorders discussed.
383
``` 6. Question: After collecting a history and from subjective information, the APRN suspects the patient may be suffering from depression. The APRN asks the patient to complete a PHQ-9. After totaling, the patient scored a 13. The APRN recognizes the severity of depression to be: A. Severe B. Mild C. Moderate D. Moderate to Severe ```
``` Answer: C Rationale for why the answer is correct: Total Score Depression Severity 1-4 Minimal Depression 5-9 Mild Depression 10-14 Moderate Depression 15-19 Moderately Severe Depression 20-27 Severe Depression ```
384
7. Question: An adult patient diagnosed with ADHD is seeking treatment with medication and attending therapy sessions to help manage the adult patient organize their life with tools such as: A. Using a calendar for scheduling events. B. Keeping routines. C. To avoid using reminder notes to avoid overstimulation and loss of focus. D. Breaking down large tasks into manageable, smaller steps so that completing each part of the task provides a sense of accomplishment.
Answer: A, B, D Rationale for why the answer is correct: A, B, and D are correct. The patient with ADHD can benefit from routines, schedules and approaching smaller tasks to accomplish a big task. In addition, using reminder notes can help focus the patient on the smaller tasks that need to be completed. Site for question/rationale: Link in resources: Attention-Deficit/Hyperactivity Disorder www.nimh.nih.gov National Institute of Mental Health Page 5.
385
``` 8. Question: The American Academy of Pediatrics recommends that every child should be screened for developmental delays at their 9-, 18, and 24- or 30-month well child visit. At what month is Autism specifically screened for? A. 18- and 24-month well child visit. B. 9- and 18-month well child visit. C. 24- or 30-month well child visit. D. 18-month well child visit. ```
Answer: A Rationale for why the answer is correct: Autism can be diagnosed as early as 18-24 months. The earlier it is detected, the better the outcome for the child not only socially, but also in language and cognitive skills also. Site for question/rationale: Link in resources: Autism Spectrum Disorder www.nimh.nih.gov National Institute of Mental Health Page 2.
386
``` 9. Question: A patient presents to the clinic with a headache and a fever. After the APRN collects a history from the patient, the APRN asks the patient to flex their head and bend their knees. The patient reports an increase in pain to the neck when doing so. What sign is the patient positive for? A. Murphy’s sign. B. Valgus and Varus sign. C. Bruzinski and Kernig signs. D. Phalen’s sign. ```
Answer: C Rationale for why the answer is correct: When a patient presents with a headache and fever, one test that is performed is the Bruzinski and Kernig sign. If the patient experiences an increase in pain in the neck, this is an indication of meningeal signs. The most common reason with a headache accompanied by fever is infectious meningitis. Site for question/rationale: Rhoades & Jenson, 2015, Differential Diagnosis for the Advanced Practice Nurse, Page141.
387
``` 10. Question: During an allergic attack, which immunoglobin primarily attaches to mast cells in the lungs, skin, and mucous membranes? A. IgA B. IgE C. IgM D. IgG ```
Answer: B Rationale for why the answer is correct: It is IgE that is primarily responsible for the release of histamine. After IgE attaches to mast cell, a chemical (histamine) is released and causes swollen membranes, skin redness, sneezing and congestion. Site for question/rationale: Resources: Nose Handout
388
``` 1. Question: Which one is not true about POCUS? A- Can be used inpatient or outpatient B- Increases procedural accuracy C- It uses less radiation than CT D- It can only be used on women ```
Answer: D Rationale for why the answer is correct: The first 3 answers are all benefits of POCUS. It can be used on both men and women and people of all ages. Site for question/rationale: Ultrasound lecture by Steve Branaham, 5 mins 40 secs
389
2. Question: What is the purpose of the FAST exam? A- To look at detailed vessels of the heart B- To look for blood in the chest and abdomen C- To look for DVTs in extremities D- To look for blood in the brain
Answer: B Rationale for why the answer is correct: The FAST exam is used to look for fluid in the abdominal and chest cavities. It is also used to recognize pneumothorax. This is helpful with trauma patients entering the ER to help rule in or out emergent diagnoses. Site for question/rationale: Ultrasound lecture by Steve Branaham, 18 mins 12 secs
390
``` 3. Question: Who is most at risk for developing melanomas? A- White adult men B- Black adult men C- Latino adult men D- Black adult women ```
Answer: A Rationale for why the answer is correct: White men older than 50 are at higher risk for developing a melanoma due the fairness of their skin and the time exposed to sun. Other factors include greater than 25 nevi, and red hair. Site for question/rationale: Dermatology lecture by Mercedes Day, 1 min 20 secs
391
``` 4. Question: Which is not a stage of Kaposi sarcoma? A- patch B- plaque C- lymphatic D- nodular ```
Answer: C Rationale for why the answer is correct: The 3 stages of Kaposi sarcoma are patch, plaque, and nodular stages. Site for question/rationale: Dermatology lecture by Mercedes Day, 11 mins 20 secs
392
``` 5. Question: Which biopsy type is best for cosmetic results? A- Shave method B- Punch method C- Blade method D- Excision ```
Answer: B Rationale for why the answer is correct: Punch method for biopsy is quick, easy, and has less scarring than the other options listed. It may only need 1 to 2 stitches to close. Site for question/rationale: Dermatology lecture by Mercedes Day, 16 mins 57 secs
393
``` 6. Question: Which of the following is not a characteristic of conduct disorder? A- Aggression towards animals B- theft C- verbal arguing with parents D- violation of the law, arrests ```
Answer: C Rationale for why the answer is correct: The other 3 answers are all characteristics of conduct disorder along with aggression towards people and destruction of property. Site for question/rationale: Mental Health lecture by Mercedes Day, 6 mins 20 secs
394
``` 7. Question: What is ADHD? A- Constant worrying B- Ongoing inattention and/or hyperactivity C- A made-up issue D- Feeling down for 6 months or more ```
Answer: B Rationale for why the answer is correct: The definition for ADHD is ongoing inattention and/or hyperactivity, usually diagnosed in school aged children. Site for question/rationale: Mental Health lecture by Mercedes Day, 2 mins 40 secs
395
8. Question: What is the most helpful when diagnosing an ear problem? A- Thorough history and physical assessment B- Culturing discharge C- CT scan D- CBC
Answer: A Rationale for why the answer is correct: By performing a thorough history and physical assessment, the provider should be able to rule in or out diagnoses based off reported symptoms and physical findings. It is not useful to culture discharge, a CT scan is expensive and only warranted after a referral to EENT specialist for non-resolving symptoms, and a CBC is not useful for diagnosing ear problems. Site for question/rationale: EENT lecture by Grace Sun and Amy Moore, 10 mins 32 secs
396
``` 9. Question: A symptom of congestion is a definitive diagnosis for which nasal problem? A- Bacterial sinitis B- Allergic rhinitis C- Non allergic rhinitis D- None of the above ```
Answer: D Rationale for why the answer is correct: When diagnosing problems of EENT, it is important to get a thorough HPI, as well as PMI and physical assessment of the patient. It is not helpful nor correct to ask a few questions and then start ordering diagnostics. Site for question/rationale: EENT lecture by Grace Sun and Amy Moore, 23 mins 9 secs
397
10. Question: A 60 y/o male with a history of cluster headaches is being seen today. Which statement by him would be considered a “red flag” and is cause for immediate intervention (sent to the ER)? A- “I have pain in my right eye only” B- “My pain woke me from my sleep” C- “I’ve had this headache for 3 days” D- “This is the worst pain I’ve ever had”
Answer: D Rationale for why the answer is correct: The first 3 answers are typical of cluster headaches. Red flags for headaches include the worst pain a patient has experienced, thunderclap headaches, difficulty thinking and talking, and a sudden change from previous headaches. All of these red flags should be sent for an immediate CT scan to rule out a head bleed. This is considered a medical emergency. Site for question/rationale: Headache lecture by Amy Moore and Tara Hillard, 4 mins